matematİk olİmpİyati ÇaliŞma kİtapÇiĞi · c»˜oz um.˜ bu soruda »c˜oz ume daha kolay...

183
MATEMATİK OLİMPİYATI ÇALIŞMA KİTAPÇIĞI www.sbelian.wordpress.com

Upload: others

Post on 03-Aug-2020

3 views

Category:

Documents


0 download

TRANSCRIPT

Page 1: MATEMATİK OLİMPİYATI ÇALIŞMA KİTAPÇIĞI · C»˜oz um.˜ Bu soruda »c˜oz ume daha kolay i»slemlerle ula»smak i»cin baz‡ de‚gi»s-˜ ken e‚gi»stirmeler yapmak yerinde

MATEMATİK

OLİMPİYATI ÇALIŞMA

KİTAPÇIĞI www.sbelian.wordpress.com

Page 2: MATEMATİK OLİMPİYATI ÇALIŞMA KİTAPÇIĞI · C»˜oz um.˜ Bu soruda »c˜oz ume daha kolay i»slemlerle ula»smak i»cin baz‡ de‚gi»s-˜ ken e‚gi»stirmeler yapmak yerinde

Matematık OlımpıyatlarıCalısma Kıtapcıgı

www.sbelian.wordpress.com

6 Temmuz 2010

Page 3: MATEMATİK OLİMPİYATI ÇALIŞMA KİTAPÇIĞI · C»˜oz um.˜ Bu soruda »c˜oz ume daha kolay i»slemlerle ula»smak i»cin baz‡ de‚gi»s-˜ ken e‚gi»stirmeler yapmak yerinde

Icindekiler

1 Giris 5

1.1 Ilksoz . . . . . . . . . . . . . . . . . . . . . . . . . . . . . . . . 5

2 KONULAR 6

2.1 Denklem Sıstemlerı . . . . . . . . . . . . . . . . . . . . . . . . . 6

2.1.1 Calısma Soruları . . . . . . . . . . . . . . . . . . . . . . 9

2.1.2 Cozumler . . . . . . . . . . . . . . . . . . . . . . . . . . 11

2.2 Repunıtler . . . . . . . . . . . . . . . . . . . . . . . . . . . . . . 16

2.2.1 Calısma Soruları . . . . . . . . . . . . . . . . . . . . . . 19

2.2.2 Cozumler . . . . . . . . . . . . . . . . . . . . . . . . . . 20

2.3 Sophıe Germaın Ozdeslıgı . . . . . . . . . . . . . . . . . . . . . 23

2.3.1 Calısma Soruları . . . . . . . . . . . . . . . . . . . . . . 24

2.3.2 Cozumler . . . . . . . . . . . . . . . . . . . . . . . . . . 26

2.4 Tamkareler Pozıtıftır . . . . . . . . . . . . . . . . . . . . . . . . 30

2.4.1 Calısma Soruları . . . . . . . . . . . . . . . . . . . . . . 31

2.4.2 Cozumler . . . . . . . . . . . . . . . . . . . . . . . . . . 34

2.5 Esıtsızlıkler I . . . . . . . . . . . . . . . . . . . . . . . . . . . . 40

2.5.1 Calısma Soruları . . . . . . . . . . . . . . . . . . . . . . 49

2.6 Esıtsızlıkler II . . . . . . . . . . . . . . . . . . . . . . . . . . . . 50

2.6.1 Calısma Soruları . . . . . . . . . . . . . . . . . . . . . . 58

2.7 Indırgemelı Dızıler . . . . . . . . . . . . . . . . . . . . . . . . . 59

2.7.1 Birinci Dereceden Indirgemeler . . . . . . . . . . . . . . 59

2

Page 4: MATEMATİK OLİMPİYATI ÇALIŞMA KİTAPÇIĞI · C»˜oz um.˜ Bu soruda »c˜oz ume daha kolay i»slemlerle ula»smak i»cin baz‡ de‚gi»s-˜ ken e‚gi»stirmeler yapmak yerinde

2.7.2 Ikinci Dereceden Indirgemeler . . . . . . . . . . . . . . . 63

2.7.3 Alıstırmalar . . . . . . . . . . . . . . . . . . . . . . . . . 65

2.8 Yenıden Duzenleme [Rearrangement] Esıtsızlıgı . . . . . . . . . 66

2.9 Trıgonometrık Degısken Degıstırme . . . . . . . . . . . . . . . . 75

2.9.1 Calısma Soruları . . . . . . . . . . . . . . . . . . . . . . 78

2.9.2 Cozumler . . . . . . . . . . . . . . . . . . . . . . . . . . 80

2.10 Cebırde Teleskopık Toplamlar ve Carpımlar . . . . . . . . . . . 90

2.10.1 Calısma Soruları . . . . . . . . . . . . . . . . . . . . . . 93

2.10.2 Cozumler . . . . . . . . . . . . . . . . . . . . . . . . . . 96

2.11 Tamdeger Fonksıyon Problemlerı . . . . . . . . . . . . . . . . . 102

2.11.1 Calısma Soruları . . . . . . . . . . . . . . . . . . . . . . 107

2.11.2 Cozumler . . . . . . . . . . . . . . . . . . . . . . . . . . 108

2.12 Bolunebılme ve Asal Sayılar . . . . . . . . . . . . . . . . . . . . 110

2.12.1 Calısma Soruları . . . . . . . . . . . . . . . . . . . . . . 114

2.12.2 Cozumler . . . . . . . . . . . . . . . . . . . . . . . . . . 116

2.13 Trıgonometrıde Sonsuz Toplam ve Farklar . . . . . . . . . . . . 122

2.13.1 Calısma Soruları . . . . . . . . . . . . . . . . . . . . . . 124

2.13.2 Cozumler . . . . . . . . . . . . . . . . . . . . . . . . . . 126

2.14 Kus, Guvercın, Yuva... . . . . . . . . . . . . . . . . . . . . . . . 133

2.14.1 Calısma Soruları . . . . . . . . . . . . . . . . . . . . . . 135

2.14.2 Cozumler . . . . . . . . . . . . . . . . . . . . . . . . . . 136

2.15 Ustel Dıyofant Denklemlerı . . . . . . . . . . . . . . . . . . . . 137

2.15.1 Calısma Soruları . . . . . . . . . . . . . . . . . . . . . . 139

2.15.2 Cozumler . . . . . . . . . . . . . . . . . . . . . . . . . . 140

2.16 Kalan Sınıfları [Resıdues] . . . . . . . . . . . . . . . . . . . . . 143

2.16.1 Calısma Soruları . . . . . . . . . . . . . . . . . . . . . . 146

2.16.2 Cozumler . . . . . . . . . . . . . . . . . . . . . . . . . . 147

2.17 Vıete Teoremi . . . . . . . . . . . . . . . . . . . . . . . . . . . . 151

2.17.1 Newton-Girard Formulæ . . . . . . . . . . . . . . . . . . 152

2.17.2 Lagrange Interpolasyon Teknıgı . . . . . . . . . . . . . . 155

2.17.3 Calısma Soruları . . . . . . . . . . . . . . . . . . . . . . 157

3

Page 5: MATEMATİK OLİMPİYATI ÇALIŞMA KİTAPÇIĞI · C»˜oz um.˜ Bu soruda »c˜oz ume daha kolay i»slemlerle ula»smak i»cin baz‡ de‚gi»s-˜ ken e‚gi»stirmeler yapmak yerinde

2.17.4 Cozumler . . . . . . . . . . . . . . . . . . . . . . . . . . 159

2.18 Bagıntı Sayıları . . . . . . . . . . . . . . . . . . . . . . . . . . . 163

2.18.1 Calısma Soruları . . . . . . . . . . . . . . . . . . . . . . 167

2.18.2 Cozumler . . . . . . . . . . . . . . . . . . . . . . . . . . 167

2.19 Lıneer Denklemlerın Tamsayı Cozumlerı . . . . . . . . . . . . . 168

2.19.1 Calısma Soruları . . . . . . . . . . . . . . . . . . . . . . 172

2.19.2 Cozumler . . . . . . . . . . . . . . . . . . . . . . . . . . 173

2.20 Fonksıyonel Denklemler . . . . . . . . . . . . . . . . . . . . . . 173

2.20.1 Tek Degiskenliler - Temel Teknikler . . . . . . . . . . . 173

2.20.2 Cok Degiskenliler . . . . . . . . . . . . . . . . . . . . . . 176

2.20.3 Calısma Soruları . . . . . . . . . . . . . . . . . . . . . . 180

2.20.4 Cozumler . . . . . . . . . . . . . . . . . . . . . . . . . . 180

4

Page 6: MATEMATİK OLİMPİYATI ÇALIŞMA KİTAPÇIĞI · C»˜oz um.˜ Bu soruda »c˜oz ume daha kolay i»slemlerle ula»smak i»cin baz‡ de‚gi»s-˜ ken e‚gi»stirmeler yapmak yerinde

Bolum 1

Giris

1.1 Ilksoz

Belkide internetin hayatımıza kattıgı en buyuk artı deger legal paylasımlaryapmak ve bilgiyi birbirimizle paylasmaya ortam hazırlamak olmustur. Bizdebu paylasımın bir parcası olarak bu zamana kadar bir cok ders notunu vecozum paketini sizlerle paylasmıstık. Simdi de, tum bu ders notlarının birle-simi olan bu mini kitabı sizlerin paylasımına acıyoruz. Sizlerde cekinmedenbu kitapcıgı gerek fotokopi ile gerekse elektronik yollarla birbirinizle paylasın.Yararlanabildiginiz kadar yararlanın.Bu calısmanın hazırlanmasında oncelikle www.sbelian.wordpress. com sayfa-mıza teveccuh gosteren tum dostlarımıza ve faydalı olabilmek icin calısmala-rımıza yardım eden LATEX2ε dostlarına tesekkur ederiz.

Sbelian Σ

Haziran 2010

5

Page 7: MATEMATİK OLİMPİYATI ÇALIŞMA KİTAPÇIĞI · C»˜oz um.˜ Bu soruda »c˜oz ume daha kolay i»slemlerle ula»smak i»cin baz‡ de‚gi»s-˜ ken e‚gi»stirmeler yapmak yerinde

Bolum 2

KONULAR

2.1 Denklem Sıstemlerı

Bu bolumde bazı standart olmayan denklem sistemlerinin cozumlerini ya-pacagız. Ancak ilerleyen orneklerde sizinde farkedeceginiz uzere, kullanacagı-mız yontemler genelde bazı cebirsel manipulasyonlardan olusuyor. Bu yon-temleri kullanarak hem cozumlere daha kolay ulasacagız hemde her bir sorudafarklı bir teknigi ogrenmis olacagız.

Ornek.

x+ y2 + z2 = 3

y + z2 + x3 = 3

z + x2 + y3 = 3

denklem sisteminin pozitif cozum uclusunun yanlızca (1, 1, 1) oldugunu kanıt-layınız.

Cozum. Eger ilk iki denklemin farkını alırsak

x(1− x2) + y(y − 1) + z2(z − 1) = 0

denklemini elde ederiz. Benzer sekilde ikinci ve ucuncu denklemlerin farkınıalırsak

y(1− y2) + z(z − 1) + x2(x− 1) = 0

6

Page 8: MATEMATİK OLİMPİYATI ÇALIŞMA KİTAPÇIĞI · C»˜oz um.˜ Bu soruda »c˜oz ume daha kolay i»slemlerle ula»smak i»cin baz‡ de‚gi»s-˜ ken e‚gi»stirmeler yapmak yerinde

2.1. DENKLEM SISTEMLERI 7

denklemini elde ederiz. Eger bu denklemi z ile carpıp bir onceki denklemdencıkarırsak

x(x− 1)(1 + x+ xz) = y(y − 1)(1 + z + yz)

esitligini elde ederiz. Benzer islemleri yaparak

y(y − 1)(1 + y + yx) = z(z − 1)(1 + x+ xz)

esitliginede ulasırız. Son yazdıgımız iki esitlikte eger x, y, z pozitifse x = y =z = 1, x, y, z < 1 veya x, y, z > 1 olacaktır. Son iki yazdıgımız olasılıgın

x+ y2 + z3 = 3

esitligini saglayamayacagı acıktır. Demek ki tek cozum

(x, y, z) = (1, 1, 1)

olacaktır.

Ornek.

a+ b+ c+ d = 12

abcd = 27 + ab+ ac+ ad+ bc+ bd+ cd

denklem sistemini saglayan tum (a, b, c, d) pozitif sayı dortlulerini bulunuz.

Cozum. Eger soruda verilen sistemdeki ikinci esitlige Aritmetik Orta -Geometrik Orta esitsizligini uygularsak

abcd ≥ 27 + 6√abcd

olacaktır. Eger bu esitsizligi duzenlersek yeni denklemimiz√abcd degiskenine

baglı

(√abcd+ 3)(

√abcd− 9) ≥ 0

esitsizligini elde ederiz. Burada

√abcd ≥ 9

Page 9: MATEMATİK OLİMPİYATI ÇALIŞMA KİTAPÇIĞI · C»˜oz um.˜ Bu soruda »c˜oz ume daha kolay i»slemlerle ula»smak i»cin baz‡ de‚gi»s-˜ ken e‚gi»stirmeler yapmak yerinde

8 BOLUM 2. KONULAR

oldugu acıktır. Eger bu durumu sistemdeki ilk denklemle beraber kullanırsak

a+ b+ c+ d

4≤ 4

√abcd ⇒ 3 ≤ 3

oldugundan esitlik durumu soz konusudur. Buna gore tek cozum

a = b = c = d = 3

olacaktır.

Ornek.

√3x

(1 +

1

x+ y

)= 2

√7y

(1− 1

x+ y

)= 4

√2

denklem sistemini pozitif reel sayılarda cozunuz.

Cozum. Bu soruda cozume daha kolay islemlerle ulasmak icin bazı degis-ken egistirmeler yapmak yerinde olacaktır. Buna gore eger

√x = u ve

√y = v

alırsak sistemimiz

u

(1 +

1

u2 + v2

)=

2√3

v

(1− 1

u2 + v2

)=

4√2√7

seklini alır. Burada u2+v2 aslında z = u+iv karmasık sayısının normunun ka-residir. Buna gore ikinci denklemi i karmasık sayısı ile carpıp birinci denklemeeklersek

u+ iv +u− iv

u2 + v2=

2√

3 + i4√2√7

esitligini elde ederiz. Buradau− iv

u2 + v2

Page 10: MATEMATİK OLİMPİYATI ÇALIŞMA KİTAPÇIĞI · C»˜oz um.˜ Bu soruda »c˜oz ume daha kolay i»slemlerle ula»smak i»cin baz‡ de‚gi»s-˜ ken e‚gi»stirmeler yapmak yerinde

2.1. DENKLEM SISTEMLERI 9

ifadesiz

|z|2 =z

(zz)=

1

z

olacaktır. Yani

z +1

z=

(2√3+ i

4√2√7

)

esitligi elde edilir. Eger bu denklemi duzenlersek

z2 −(

2√3+ i

4√2√7

)z + 1 = 0

denklemini ve bu denklemin cozumu olan

(1√3± 2√

21

)+ i

(2√2√7

±√2

)

sayısını elde ederiz. Buna gore soruda verilen sistemin cozumu

x =

(1√3± 2√

21

)2

ve y =

(2√2√7

±√2

)2

olacaktır.

2.1.1 Calısma Soruları

1.

x+2

x= 2y

y +2

y= 2z

z +2

z= 2x

denklem sistemini cozunuz.

Page 11: MATEMATİK OLİMPİYATI ÇALIŞMA KİTAPÇIĞI · C»˜oz um.˜ Bu soruda »c˜oz ume daha kolay i»slemlerle ula»smak i»cin baz‡ de‚gi»s-˜ ken e‚gi»stirmeler yapmak yerinde

10 BOLUM 2. KONULAR

2. [x], x sayısının tam kısmını, {x} ise ondalık kısmını temsil etmek uzerex = [x] + {x} olarak veriliyor. Buna gore

x+ [y] + {z} = 1, 1

z + [x] + {y} = 2, 2

y + {x}+ [z] = 3, 3

denklem sistemini saglayan x, y, z degiskenlerini bulunuz.

3. Asagıdaki denklem sistemini pozitif reel sayılarda cozunuz.

xy + yz + zx = 12

xyz = 2 + x+ y + z

4.

4x2

4x2 + 1= y

4y2

4y2 + 1= z

4z2

4z2 + 1= x

Denklem sisteminin tum reel cozumlerini bulunuz.

5.

3 = x+ y + z = x3 + y3 + z3

esitligini saglayan tum tamsayı uclulerini bulunuz.

6.

6(x− y−1) = 3(y − z−1) = 2(z − x−1) = xyz − (xyz)−1

esitligini saglayan tum sıfıran farklı x, y, z reel sayılarını bulunuz.

Page 12: MATEMATİK OLİMPİYATI ÇALIŞMA KİTAPÇIĞI · C»˜oz um.˜ Bu soruda »c˜oz ume daha kolay i»slemlerle ula»smak i»cin baz‡ de‚gi»s-˜ ken e‚gi»stirmeler yapmak yerinde

2.1. DENKLEM SISTEMLERI 11

7. a, b, c birbirinden ve sıfırdan farklı reel sayılar olmak uzere verilen

x2 − yz = a

y2 − zx = b

z2 − xy = c

denklem sisteminin tum (x, y, z) reel uclulerini bulunuz.

8.

(x+ y)3 = z

(y + z)3 = x

(z + x)3 = y

denklem sistemini saglayan tum (x, y, z) reel sayı ucluleini bulunuz.

9.

x3 − 9(y2 − 3y + 3) = 0

y3 − 9(z2 − 3z + 3) = 0

z3 − 9(x2 − 3x+ 3) = 0

denklem sisteminin tum cozumlerini bulunuz.

10. a, b, x, y reel sayılar olmak uzere

ax+ by = 3

ax2 + by2 = 7

ax3 + by3 = 16

ax4 + by4 = 42

olarak veriliyor. Buna gore, ax5 + by5 toplamının esitini bulunuz.

2.1.2 Cozumler

1. Eger cozumde aritmetik orta - geometrik orta esitsizligini kullanırsak

x+1

x≥ 2

√x · 2

x= 2

√2

Page 13: MATEMATİK OLİMPİYATI ÇALIŞMA KİTAPÇIĞI · C»˜oz um.˜ Bu soruda »c˜oz ume daha kolay i»slemlerle ula»smak i»cin baz‡ de‚gi»s-˜ ken e‚gi»stirmeler yapmak yerinde

12 BOLUM 2. KONULAR

ise 2y ≥ 2√2 ve y ≥ √

2 olacaktır. Benzer sekilde y ≥ √2 ve z ≥ √

2olacaktır. Eger sistemdeki 3 denklemi −1 ile carpıp toplarsak

x+ y + z = 2

(1

x+

1

y+

1

z

)

esitligi elde edilir. Bu denklemin sol kısmı

x+ y + x ≥ 3√2

iken sag kısım

2

(1

x+

1

y+

1

z

)≤ 3

√2

olacaktır. Demek ki

x+ y + z = 2

(1

x+

1

y+

1

z

)= 3

√2

olmalıdır. Buradan istenen cozumler

x = y = z = ±√2

olarak bulunur.

2. Eger sistemdeki 3 esitsizligi taraf tarafa toplarsak

2x+ 2y + 2z = 6.6

oldugundan

x+ y + z = 3.3

denklemini elde ederiz. Sistemdeki her bir denklemi son buldugumuzesitlikten cıkarırsak yeni sistemimiz

{y}+ [z] = 2.2

{x}+ [y] = 1.1

{z}+ [x] = 0

Page 14: MATEMATİK OLİMPİYATI ÇALIŞMA KİTAPÇIĞI · C»˜oz um.˜ Bu soruda »c˜oz ume daha kolay i»slemlerle ula»smak i»cin baz‡ de‚gi»s-˜ ken e‚gi»stirmeler yapmak yerinde

2.1. DENKLEM SISTEMLERI 13

olacaktır. Bu sistemde ilk denklemden {y} = 0.2 ve [z] = 2 bulunur.Ikinci denklemden [y] = 1, {0.1} ve ucuncu denklemden {z} = 0, [x] = 0bulunur. Buna gore, denklem sisteminin cozumu

x = 0.1, y = 1.2, z = 2

olacaktır.

3. Varsayalım 3√xyz = a olsun. Aritmetik orta - geometrik orta esitsizligin-

den12 = xy + yz + zx ≥ 3a2

vea3 = 2 + x+ y + z ≥ 2 + 3a

olacaktır. ilk esitsizlikten 12 ≥ 3a2 ise 4 ≥ a2 bulunur. Ikinci esitsizlik-tense a ≥ 2 bulunur. Dolayısıyla a = 2 ve x = y = z olacaktır. Demekki tek cozum

(x, y, z) = (2, 2, 2)

olur.

4. Cozume, denklem sistemimizdeki her bir denklemi karsılayan bir fonk-siyon bularak baslayalım. Varsayalım f fonksiyonu f : [0,∞) → [0,∞)olmak uzere

f(t) =4t2

4t2 + 1

olarak verilsin. f fonksiyonun artan oldugu acıktır. Dolayısıyla eger x < yise f(x) < f(y) dolayısıyla y < z olacaktır. Eger bu argumanı tekraredersek z < x elde ederiz. Oyleyse x < y < z < x olacaktır ki bu durumimkansızdır. Benzer bicimde x > y ile baslarsak yine celiski elde ederiz.Demek ki x = y = z olmalıdır. Buna gore,

4t2

4t2 + 1= t

denklemini cozersek t = 0 veya t = 1/2 olacaktır. Oyleyse sistemincozumleri sadece

(0, 0, 0) ve (1

2,1

2,1

2)

olarak bulunur.

Page 15: MATEMATİK OLİMPİYATI ÇALIŞMA KİTAPÇIĞI · C»˜oz um.˜ Bu soruda »c˜oz ume daha kolay i»slemlerle ula»smak i»cin baz‡ de‚gi»s-˜ ken e‚gi»stirmeler yapmak yerinde

14 BOLUM 2. KONULAR

5. Eger soruda verilen esitlik uzerinde biraz oynarsak

24 = (x+ y + z)3 − (x3 + y3 + z3) = 3∑

x2y + 6xyz

esitligine ulasırız. Eger sadelestirme yapıp carpanlarına ayırırsak

8 = (x+ y)(x+ z)(y + z)

olacagından

8 = (3− x)(3− y)(x+ y)

esitligi elde edilir. 8 sayısının carpanlarını kontrol edersek cozumleri

(1, 1, 1), (4, 4,−5), (4,−5, 4), (−5, 4, 4)

olarak bulabiliriz.

6. Soruda verilen esitligi kullanarak

(x− y−1) + (y − z−1) + (z − x−1) = xyz − (xyz)−1

yazabiliriz. Eger bu esitligi yeniden duzenlersek de

(x− y−1)(y − z−1)(z − x−1) = 0

denklemini elde ederiz. Demek ki carpanlardan biri sıfır olmalıdır. Ancaksorudaki esitlikten hepsinin sıfır oldugu ortaya cıkar. Dolayısıyla

xy = yz = zx = 1 ve x = y = z = ±1

bulunur.

7. Soruda verilen sistemdeki herbir esitligin karesini alıp diger iki esitligincarpımından cıkarırsak yeni sistemimimiz

a2 − bc = x(x3 + y3 + z3 − 3xyz)

b2 − ac = y(x3 + y3 + z3 − 3xyz)

c2 − ab = z(x3 + y3 + z3 − 3xyz)

Page 16: MATEMATİK OLİMPİYATI ÇALIŞMA KİTAPÇIĞI · C»˜oz um.˜ Bu soruda »c˜oz ume daha kolay i»slemlerle ula»smak i»cin baz‡ de‚gi»s-˜ ken e‚gi»stirmeler yapmak yerinde

2.1. DENKLEM SISTEMLERI 15

olacaktır. Buradax3 + y3 + z3 − 3xyz = k

olarak alırsak

(a2 − bc)2 − (b2 − ac)2(c2 − ab) = k2(x2 − yz) = k2a

olur. Burada esitligin sol tarafını acıp duzenlersek

k = ±√

a3 + b3 + c3 − 3abc

olacaktır. Buna gore sistemin cozumleri

x =a2 − bc

k, y =

b2 − ac

k,

c2 − ab

k

olacaktır.

8. Eger ikinci denklemi birinciden cıkarırsak

(x− z)((x+ y)2 + (x+ y)(y + z) + (y + z)2

)= z − x

esitligini elde ederiz. Burada

(x+ y)2 + (x+ y)(y + z) + (y + z)2 > 0

olduguna gore, x = z olacaktır. Simetriden dolayı y = z ve elimizde8x3 = x denklemi olusacaktır. Bu denklemin kokleri x = 0 ve x = ± 2

2√2

olacaktır. Buna gore sistemin cozumleri

x = y = z = 0, x = y = z = ± 1

2√2

olur.

9. Soruda verilen sistemi duzenlersek, yeni sistemimiz

(y − 3)3 = y3 − x3

(z − 3)3 = z3 − x3

(x− 3)3 = x3 − z3

Page 17: MATEMATİK OLİMPİYATI ÇALIŞMA KİTAPÇIĞI · C»˜oz um.˜ Bu soruda »c˜oz ume daha kolay i»slemlerle ula»smak i»cin baz‡ de‚gi»s-˜ ken e‚gi»stirmeler yapmak yerinde

16 BOLUM 2. KONULAR

olacaktır. Bu esitligi toplarsak

(x− 3)3 + (y − 3)3 + (z − 3)3 = 0

esitligi elde edilir. Genelligi bozmadan varsayalım x ≥ 3 olsun. Sistem-deki ucuncu denklemden

z3 − 27 = 9x(x− 3)

olacaktır, dolayısıyla z ≥ 3 olur. Benzer bicimde y ≥ 3 olacaktır. Ancakucu birden 3’ten buyuk olamaz. Buna gore tek cozum x = y = z = 3olacaktır.

10. n = 2 ve n = 3 icin

(axn + byn)(x+ y)− (axn−1 + byn−1)xy = axn+1 + byn+1

esitliginin saglandıgı gorulebilir. Buna gore,

7(x+ y)− 3xy = 16

ve

16(x+ y)− 7xy = 42

olacaktır. Bu iki denklemi cozersek x+y = −14 ve xy = −38 olur. n = 4icin basta belirledigimiz rekuransı yeniden uygularsak

ax5 + by5 = (42)(−14)− (16)(−38) = −588 + 608 = 20

olacaktır.

2.2 Repunıtler

Basamaklarını olusturan rakamların hepsi 1 olan dogal sayılara repunitdiyecegiz. Oyleki

111 · · · 111︸ ︷︷ ︸n basamaklı

Page 18: MATEMATİK OLİMPİYATI ÇALIŞMA KİTAPÇIĞI · C»˜oz um.˜ Bu soruda »c˜oz ume daha kolay i»slemlerle ula»smak i»cin baz‡ de‚gi»s-˜ ken e‚gi»stirmeler yapmak yerinde

2.2. REPUNITLER 17

sayısı bir repunit sayı olarak sorularımızda yerini alacaktır. Simdi ilk orne-gimizle baslayalım. Sorumuz 2005 Bulgaristan Matematik Olimpiyatları’ndasorulmustur.

Ornek. x, y, z ∈ Z olmak uzere verilen

x2 + 2y2 + 98z2 = 111 · · · 111︸ ︷︷ ︸666 tane 1

esitligini saglayan tamsayı uclulerinin bulunmadıgını kanıtlayınız.

Cozum I. Cozume celiski ile ulasmaya calısalım. varsayalım soruda ve-rilen denklemi saglayan (x, y, z) tamsayı uclusu varolsun. Buna gore esitliginsol tarafındaki repuniti carpanlarına ayırırsak

111111 · (1 + 106 + · · ·+ 106·110) =106 − 1

9(1 + 106 + · · ·+ 106·110)

olacaktır. Kucuk Fermat teoremine gore, 106 − 1 sayısı 7 ile bolunebilir. Bunagore esitligimize (mod7) altında bakabiliriz. Tamkare bir sayının (mod7) al-tındaki kalan sınıfı {0, 1, 2, 3, 4} ve 98 sayısı 7 ile kalansız bolunebildigine gore,esitligin sag tarafındaki ifademizde x ve y sayıları 7 ile kalansız bolunmelidir.Bu durumda esitligin sol tarafı 72 ile tam bolunecektir. Ancak esitligin sagkısmındaki ifademiz

7 · 15873 · (1 + 106 + · · ·+ 106·110)

oldugundan bu ifadenin (mod7) altındaki ikinci carpanı 4 olacaktır. Ucuncucarpanı (mod7) altında 0 olmadıgına gore, esitligin sag tarafı 49 ile bolune-mez. Celiski vardır. Demekki (x, y, z) tamsayı ucluleri yoktur.

Cozum II. Eger soruda verilen esitligin sag tarafına (mod) altında ba-karsak

x2 + 2y2 + 2x2 ≡ 111 (mod 8)

veya

x2 + 2y2 + 2z2 ≡ 7 (mod 8)

Page 19: MATEMATİK OLİMPİYATI ÇALIŞMA KİTAPÇIĞI · C»˜oz um.˜ Bu soruda »c˜oz ume daha kolay i»slemlerle ula»smak i»cin baz‡ de‚gi»s-˜ ken e‚gi»stirmeler yapmak yerinde

18 BOLUM 2. KONULAR

oldugunu gorebiliriz. Bir tamsayının karesinin ( mod 8) altındaki kalanları {0, 1, 4}olabilir. Eger iki katlarını alırsak kalan sınıfı {0, 2} olacaktır. Ancak elde edilen{0, 1, 4} kumesinin elemanlarını {0, 2} ve {0, 2} kumesinin elemanlarına ekleye-rek (mod8) altında 7 sayısına ulasamayız. Demek ki, soruda verilen denklemisaglayan (x, y, z) tamsayıları yoktur.

Sıradaki sorumuzda Rusyada yayınlanan Potansia Magazine isimli dergi-den alıntıdır.

Ornek. Repunitleri yine repunitlere goturen tum ikinci dereceden tam-sayı katsayılı polinomları bulunuz.

Cozum. varsayalım ikinci dereceden polinomumuz f(x) = ax2 + bx+ colsun. Soruda verilen sartlara gore,

f(111 · · · 1︸ ︷︷ ︸m tane

) = 111 · · · 1︸ ︷︷ ︸n tane

yani

f

(10m − 1

9

)=

10n − 1

9

olacaktır. Eger

g(x) = 9f

(x− 1

9

)+ 1

olarak alırsak,

9

(a

(x− 1

9

)2

+ b

(x− 1

9

)+ 1

)= 9

(a

92(x2 − 2x+ 1) +

b

9(x− 1) + c

)+ 1

=a

9(x2 − 2x+ 1) + b(x− 1) + c+ 1

=a

9x2 +

(b− 2a

9

)x+ (9c+ 1− b+

a

9)

olacaktır. Demek ki

g(x) =a

9x2 +

(b− 2a

9

)x+ (9c+ 1− b+

a

9)

Page 20: MATEMATİK OLİMPİYATI ÇALIŞMA KİTAPÇIĞI · C»˜oz um.˜ Bu soruda »c˜oz ume daha kolay i»slemlerle ula»smak i»cin baz‡ de‚gi»s-˜ ken e‚gi»stirmeler yapmak yerinde

2.2. REPUNITLER 19

olacaktır. Buna gore, aslında g(10m) = 10n olacaktır. yani g polinomu 10’unkuvvetlerini yine 10’un kuvvetlerine goturecektir. Buna gore,

10−2m · g(10m) = 10n−2m =a

9+

(b− 2a

9

)10−m +

(9c+ 1− b+

a

9

)10−2m

olacaktır. Ilk esitlikten goruldugu uzere 10−2m ·g(10m) sayısı 10’un kuvvetidir.Eger m degerini yeterince buyuk alırsak 10−2mg(10m) ifadesi a/9 degerineyakınsak. Buradan da, a/9 = 10k ve 10−2mg(10m) = 10k olacaktır. Dolayısıyla

(b− 2a

9

)10−m +

(9c+ 1− b+

a

9

)10−2m = 0

olacagından

b− 2a

9= 9c+ 1− b+

a

9= 0

olur. Buradan b = 2 · 10k ve c = 10k−19 olacagından istenen polinomlar

f(x) = 9 · 10k · x2 + 2 · 10k · x+10k − 1

9

olur.

2.2.1 Calısma Soruları

1. (a.) Cift sayıda basamagı olan, bes tabanındaki repunitlerin ardısık po-zitif iki tamsayının carpımı oldugunu gosteriniz.

(b.) Dokuz tabanındaki tum repunitlerin Ucgensel Sayı1 oldugunu ka-nıtlayınız.

2.111 · · · 1︸ ︷︷ ︸2n tane

= 222 · · · 2︸ ︷︷ ︸n tane

+(333 · · · 3︸ ︷︷ ︸n tane

)2

esitligini kanıtlayınız.

1Bir ak ucgensel sayısı, 1’den k’ya kadar olan tamsayıların toplamıdır. Ucgensel sayıdenilmesinin temel sebebi, eskenar bir ucgenin uzerine esit aralıklarla yerlestirilibilecek esityarıcaplı kurelerin sayısını vermesidir. Oyleki, ilk bir kac ucgensel sayı 1, 3, 6, 10, 15, 21, 28olarak kolaylıkla bulunabilir.

an = 1+2+3+ · · ·+(n− 1)+n = n(n+1)2

= n2+n2

=(n+12

)genel formuda n. ucgensel sayıyı

vermektedir.

Page 21: MATEMATİK OLİMPİYATI ÇALIŞMA KİTAPÇIĞI · C»˜oz um.˜ Bu soruda »c˜oz ume daha kolay i»slemlerle ula»smak i»cin baz‡ de‚gi»s-˜ ken e‚gi»stirmeler yapmak yerinde

20 BOLUM 2. KONULAR

3. 19 ile bolunebilen ve tum basamakları 1 olan en kucuk pozitif tamsayıyıbulunuz.

4. Asal bir repunit sayının basamak sayısınında asal oldugunu kanıtlayınız.Ayrıca, bu onermenin tersi dogrumudur?

5. Tum basamakları 1 olan, 81 basamaklı bir sayı 81 ile kalansız bolunebilirmi?

6. (a.) 111 · · · 1︸ ︷︷ ︸n basamaklı

sayının 41 ile ancak ve ancak 5|n oldugunda, bolunebile-

cegini kanıtlayınız.

(b.) 111 · · · 1︸ ︷︷ ︸n basamaklı

sayının 91 ile ancak ve ancak 6|n oldugunda, bolunebile-

cegini kanıtlayınız.

7. 1’den buyuk hic bir repunitin tam kare olamayacagını kanıtlayınız.

8. Son basamagı 1, 3, 7 veya 9 olan her n tamsayısı icin, n ile bolunebilenbir repunit bulundugunu kanıtlayınız.

9. Herhangi iki elemanı aralarında asal olan sonsuz buyuklukte bir repu-nitler dizisi oldugunu kanıtlayınız.

10. Sonsuz coklukta n degeri icin, basamaklarında sıfır bulunmayan n basa-maklı bir sayının basamakları toplamına bolunebilecegini kanıtlayınız.

2.2.2 Cozumler

1. (a.)

(111 · · · 1︸ ︷︷ ︸2n tane

)5 = 1 + 5 + · · ·+ 52n−1

=52n − 1

4=

5n − 1

2· 5

n + 1

2

esitligine gore, 5n−12 ve 5n+1

2 ardısık tamsayılardır.

Page 22: MATEMATİK OLİMPİYATI ÇALIŞMA KİTAPÇIĞI · C»˜oz um.˜ Bu soruda »c˜oz ume daha kolay i»slemlerle ula»smak i»cin baz‡ de‚gi»s-˜ ken e‚gi»stirmeler yapmak yerinde

2.2. REPUNITLER 21

(b.)

(111 · · · 1︸ ︷︷ ︸n tane

)9 = 1 + 9 + · · ·+ 9n−1

=9n − 1

8=

1

2· 3

n − 1

2· 3

n + 1

2

olduguna gore bu sayı bir ucgensel sayıdır. Cunku, 3n−12 ve 3n+1

2ardısık tamsayılardır.

2. Soruda verilen esitligi kullanalım. Buna gore,

111 · · · 1︸ ︷︷ ︸2n tane

− 222 · · · 2︸ ︷︷ ︸n tane

= 111 · · · 1︸ ︷︷ ︸n tane

000 · · · 0︸ ︷︷ ︸n tane

− 111 · · · 1︸ ︷︷ ︸n tane

= 111 · · · 1︸ ︷︷ ︸n tane

·(10n − 1) = 111 · · · 1︸ ︷︷ ︸n tane

· 999 · 9︸ ︷︷ ︸n tane

= 333 · · · 3︸ ︷︷ ︸n tane

· 333 · · · 3︸ ︷︷ ︸n tane

= (333 · 3︸ ︷︷ ︸n tane

)2

olur.

3. Varsayalım A sorudaki sartları saglayan sayımız olsun. Eger basamaksayısını n olarak alırsak A = 10n−1

9 olacaktır. 9 ve 19 sayıları aralarındaasal olduklarına gore, en kucuk n degeri icin, 10n − 1 sayısının 19 ilebolunmesi gerekmektedir. Kucuk Fermat teoremine gore, 1018 − 1 sayısı19 ile bolunebilir. Eger daha kuck bir n degeri varsa, 10n ≡ 1 (mod19) ven|18 olmalıdır. Bu durumda 2, 3, 6 ve 9 degerlerini kontrol etmemiz gere-kir. Ancak bu sayılardan hicbirisinin denkligi saglamadıgı gorulecektir.Demek ki en kucuk sayımız 19 basamaklıdır.

4. m,n > 1 olmak uzere repunit sayımız m× n basamaklıysa, bu sayıyı

111 · · · 1︸ ︷︷ ︸n tane

×1 000 · · · 0︸ ︷︷ ︸n−1 tane

1 000 · · · 0︸ ︷︷ ︸n−1 tane

1 · · · 1 000 · · · 0︸ ︷︷ ︸n−1 tane

1

seklinde carpanlara ayırabiliriz. Burada m − 1 tane sıfırlardan olusmusgrup vardır. Dolayısıyla bu sekildekii bir repunit asal olamaz. Demek ki,

Page 23: MATEMATİK OLİMPİYATI ÇALIŞMA KİTAPÇIĞI · C»˜oz um.˜ Bu soruda »c˜oz ume daha kolay i»slemlerle ula»smak i»cin baz‡ de‚gi»s-˜ ken e‚gi»stirmeler yapmak yerinde

22 BOLUM 2. KONULAR

asal bir repunit icin, basamak sayısıda asal olmalıdır. Ancak tersi dogrudegildir. Mesela 111 = 3× 37 ve

111 · · · 1︸ ︷︷ ︸11 tane

= 21649× 513239

sayıları asal degildirler.

5. Soruda verilen sayıyı carpanlarına ayırmaya calısalım. Buna gore,

111 · · · 1︸ ︷︷ ︸81 tane

= 111 · · · 1︸ ︷︷ ︸9 tane

× 100 · · · 0100 · · · 010 · · · 01︸ ︷︷ ︸9 tane 1 64 tane 0

olur. Carpanların her ikiside 9 ile bolunebilir. Dolayısıyla sayımız 81 ilebolunebilir.

6. (a.) Varsayalım n = 5k + r, r ∈ {0, 1, 2, 3, 4} olsun. Buna gore,

111 · · · 1︸ ︷︷ ︸n tane

= 111 · · · 1︸ ︷︷ ︸5k tane

000 · · · 0︸ ︷︷ ︸r tane

+111 · · · 1︸ ︷︷ ︸r tane

= 11111×100001 · · · 00001+111 · · · 1︸ ︷︷ ︸r tane

olacaktır. 11111 = 41 × 271 olduguna gore, ifade mod41 altında111 · · · 1︸ ︷︷ ︸r tane

’e esittir. Fakat 1, 11, 111, 1111 sayıları 41 ile bolunemez.

Dolayısıyla 111 · · · 1︸ ︷︷ ︸n tane

sayısının 41 ile bolunebilmesi icin ancak ve an-

cak r = 0 yani 5|n olmalıdır.

(b.) Onceki sıkta kullanılan aynı yontemle cozume gidilir. Sadece 111111 =91× 1221 olacaktır.

7. Varsayalım sayımız

A = 111 · · · 1︸ ︷︷ ︸n tane

=10n − 1

9

bir tamkare olsun. Buna gore, A ≡ 111 · · · 1 ≡ 11 ≡ 3 (mod4) olur.Ancak bu imkansızdır cunku bir tamkarenin (mod4) altıdaki denklerisadece {0, 1} olabilir.

Page 24: MATEMATİK OLİMPİYATI ÇALIŞMA KİTAPÇIĞI · C»˜oz um.˜ Bu soruda »c˜oz ume daha kolay i»slemlerle ula»smak i»cin baz‡ de‚gi»s-˜ ken e‚gi»stirmeler yapmak yerinde

2.3. SOPHIE GERMAIN OZDESLIGI 23

8. Eger dikkat edilirse 1, 3, 7 ve 9 sayılarının 10 ile aralarında asal olduklarıkolayca gorulebilir. Varsayalım elimizde 1, 11, 111, · · · , 111 · · · 1︸ ︷︷ ︸

n+1 tane

sayıları-

mız olsun. Guvercin Yuvası Ilkesine gore, bu (n + 1) sayı arasından ikitanesi n ile bolundugunde aynı kalanları verir. Bu sayıların farkı ise n ilebolunebilir ve fark a · b formundadır. Burada a bir repunit ve b ise 10’unkuvvetidir. (n, 10) = 1 olduguna gore, repunit a sayısı 10 ile bolunmeli-dir. ispat tamamlanır.

9. Euclide’in sosuz coklukta asalın varlıgını kanıtladıgı ispatını sorumuzaadapte edelim. Varsayalım a1 = 1 olsun ve dizinin elemanlarını an’e ka-dar secelim. Bir onceki problemden a1 · a2 · · · · · an carpımını bolen birm repunit sayısı vardır. 10m + 1 sayısıda bir repunittir ve m ile arala-rında asaldırlar. Dolayısıyla 1 ≤ k ≤ n olmak uzere ak’larda aralarındaasaldırlar. Eger an+1 = 10m+ 1 secersek kanıt tamamlanır.

10. Tumevarımla ispatlamaya calısalım. Buna gore 3n basamaklı bir repu-nitin 3n ile bolunebilecegini gostermemiz yeterli olacaktır. n = 1 icin111 = 3×37 olur. Varsayalım n basamak icinde dogru olsun. Buna gore,

111 · · · 1︸ ︷︷ ︸3n+1 tane

= 111 · · · 1︸ ︷︷ ︸3n tane

×1 000 · · · 0︸ ︷︷ ︸3n−1 tane

1 000 · · · 0︸ ︷︷ ︸3n−1 tane

1

olacaktır. Ilk carpanın 3n ile bolunebildigi acıktır. Ikinci carpanda 3 ilekalansız bolunebilecegine gore, (n+ 1) icin ispatı tamamlarız.

2.3 Sophıe Germaın Ozdeslıgı

x2 + 1 polinomunun R uzerinde indirgenemeyen yada carpanlarına ayrıla-mayan bir polinom oldugunu biliyoruz. Ancak, her nekadar benzer gibi gorun-sede, x4 + 1 polinomu icin durum aynı degildir. Oyleki,

x4 + 1 =(x4 + 2x2 + 1

)− 2x2

=(x2 + 1

)2 −(√

2x)2

=(x2 +

√2x+ 1

)(x2 −

√2x+ 1

)

Page 25: MATEMATİK OLİMPİYATI ÇALIŞMA KİTAPÇIĞI · C»˜oz um.˜ Bu soruda »c˜oz ume daha kolay i»slemlerle ula»smak i»cin baz‡ de‚gi»s-˜ ken e‚gi»stirmeler yapmak yerinde

24 BOLUM 2. KONULAR

olacaktır. Benzer bicimde

x4 + 4y4 =(x2 + 2xy + 2y2

) (x2 − 2xy + 2y2

)

ozdesligide elde edilebilir. Eger katsayıyı degitirirsek,

x4 +1

4y4 =

(x2 + xy +

1

2y2)(

x2 − xy +1

2y2)

olacaktır. Bu son yazdıgımız iki ozdeslik sorularn cozumunde oldukca kulla-nıslıdır ve literaturdeki adıda Sophie Germain Identity olarak gecer.

Ornek.n∑

k=1

4k

4k4 + 1

olarak verilen toplamı bulunuz.

Cozum. Eger bildigimiz ozdeslikleri kullanırsak,

n∑

k=1

4k

4k4 + 1=

n∑

k=1

(2k2 + 2k + 1)− (2k2 − 2k + 1)

(2k2 + 2k + 1)(2k2 − 2k + 1)

=

n∑

k=1

(1

(2k2 + 2k + 1)− 1

(2(k + 1)2 + 2(k + 1) + 1)

)

= 1− 1

2n2 + 2n+ 1

olacaktır. Yukarıda cozumlerini verdigimiz ornekler konunun anlasılması icinyeterlidir. Simdi bu bilgiler ısıgında calısma sorularını cozmeye calısınız.

2.3.1 Calısma Soruları

1. n > 2, n ∈ Z icin 22n−2 + 1 sayısının asal olmadıgını kanıtlayınız.

2.xn+1 + xn−1 =

√2xn

dizisinin periyodik oldugunu kanıtlayınız.

Page 26: MATEMATİK OLİMPİYATI ÇALIŞMA KİTAPÇIĞI · C»˜oz um.˜ Bu soruda »c˜oz ume daha kolay i»slemlerle ula»smak i»cin baz‡ de‚gi»s-˜ ken e‚gi»stirmeler yapmak yerinde

2.3. SOPHIE GERMAIN OZDESLIGI 25

3.n∑

k=1

k2 − 1/2

k4 + 1/4

toplamını hesaplayınız.

4. (14 + 1

4

) (34 + 1

4

) · · · · · ·((2n− 1)4 + 1

4

)

(24 + 1

4

) (44 + 1

4

) · · · · · ·((2n)4 + 1

4

)

ifadesinin esitini bulunuz.

5. n4 + a sayısının asal olmamasını saglayan sonsuz sayıda pozitif a degerioldugunu kanıtlayınız.

6. n4+4n sayısının ancak ve ancak n = 1 durumunda asal oldugunu kanıt-layınız.

7. P (x) = x4+6x2−4x+1 polinomu veriliyor. Buna gore, P (x4) polinomu-nun katsayıları tamsayı olan ve derecesi 1’den buyuk olan iki polinomuncarpımı seklinde yazılabilecegini kanıtlayınız.

8. n12 + 64 polinomunun her biri 1’den buyuk 4 farklı carpanın carpımıolarak yazılabilecegini kanıtlayınız.

9. m,n pozitif tamsayılardır, m bir cift sayı ise

m∑

k=0

(−4)k n4(m−k)

toplamının asal olmadıgını kanıtlayınız.

10. P (x) = xn−4 + 4n polinomunun, sabit polinomdan farklı 4 polinomuncarpımı seklinde yazılmasını saglayan en kucuk n degeri kactır?

Page 27: MATEMATİK OLİMPİYATI ÇALIŞMA KİTAPÇIĞI · C»˜oz um.˜ Bu soruda »c˜oz ume daha kolay i»slemlerle ula»smak i»cin baz‡ de‚gi»s-˜ ken e‚gi»stirmeler yapmak yerinde

26 BOLUM 2. KONULAR

2.3.2 Cozumler

1. Soruda verilen ifadeyi

22n−2 + 1 = 1 +

m4

4, m = 22

n−2

olarak yazabiliriz . Buna gore,

1 +m4

4=

(1 +m+

1

2m2

)(1−m+

1

2m2

)

olacagından sayımız asal degildir.

2. x4 + 1 polinomunu carpanlarına ayırırsak,

x4 + 1 =(x2 −

√2x+ 1

)(x2 +

√2x+ 1

)

olacaktır. Buna α ve β sayıları x2 +√2x + 1 karakteristiginin kokleri

ise bunlar aynı zamanda x4 +1 polinomunun kokleridir. Benzer bicimdex2 +

√2x + 1 icinde aynı durum soz konusu ise, xn = a · αn + b · βn

esitliginden, dizinin periyodunun 8 oldugu bulunur.

3. k4 + 14 =

(k2 − k + 1

2

) (k2 + k + 1

2

)olduguna gore,

n∑

k=1

k2 − 1/2

k4 + 1/4=

n∑

k=1

(k − 1/2

k2 − k + 1/2− k + 1/2

k2 + k + 1/2

)=

n∑

k=1

(k − 1/2

k2 − k + 1/2− (k + 1)− 1/2

(k + 1)2 − (k + 1) + 1/2

)

olacagından toplam

1− (2n+ 1)

(2n2 + 2n+ 1)

olarak bulunacaktır.

4. Soruda verilen ifadeyi genellestirirsek,

n∏

k=1

(2k − 1)4 + 1/4

(2k)4 + 1/4

Page 28: MATEMATİK OLİMPİYATI ÇALIŞMA KİTAPÇIĞI · C»˜oz um.˜ Bu soruda »c˜oz ume daha kolay i»slemlerle ula»smak i»cin baz‡ de‚gi»s-˜ ken e‚gi»stirmeler yapmak yerinde

2.3. SOPHIE GERMAIN OZDESLIGI 27

ifadesini elde ederiz. Eger bu ifadenin pay ve paydasını carpanlına ayı-rırsak,

n∏

k=1

((2k − 1)2 + (2k − 1) + 1/2

)((2k − 1)2 − (2k − 1) + 1/2

)((2k)2 + 2k + 1/2

)((2k)2 − 2k + 1/2

)

durumunu elde ederiz. Ancak

m2 −m+ 1/2 = (m− 1)2 + (m− 1) + 1/2

oldugundan pay ve paydadaki carpanlardan birer tanesi sadelesir. Sadece12−1+1/2 pay kısmında ve (2n)2+2n+1/2 paydada kalır. Dolayısıylaistenilen cevap

1

8n2 + 4n+ 1

olacaktır.

5. Eger a = 4k4, k > 1 olarak alırsak,

n4 + 4k4 =(n2 + 2nk + 2k2

) (n2 − 2nk + 2k2

)

olacaktır. Buna gore, n2 + 2nk + 2k2 > k > 1 ve n2 − 2nk + 2k2 =(n− k)2 + k2 > k2 > 1 oldugundan n4 + 4k4 asal olamaz ve sonsuzsayıda a degeri secilebilir.

6. Eger n cift bir sayı ise n4+4n cift olacaktır. Eger n tek sayı ve n = 2k+1ise,

(2k + 1)4 + 4(2k+1) = (2k + 1)4 + 4 · 42k = (2k + 1)4 + 4 (2)4k

olacaktır. Eger (2k + 1) = a ve 2k = b olarak alırsak, ifademiz

a4 + 4b4 =(a2 + 2ab+ 2b2

) (a2 − 2ab+ 2b2

)

olacaktır. Buna gore ifademiz,

((2k + 1)2 + 2 (2k + 1) 2k + 2 · 22k

)((2k + 1)2 − 2 (2k + 1) 2k + 2 · 22k

)

Page 29: MATEMATİK OLİMPİYATI ÇALIŞMA KİTAPÇIĞI · C»˜oz um.˜ Bu soruda »c˜oz ume daha kolay i»slemlerle ula»smak i»cin baz‡ de‚gi»s-˜ ken e‚gi»stirmeler yapmak yerinde

28 BOLUM 2. KONULAR

olacaktır. Buradan da,(n2 + 2k+1n+ 22k+1

)(n2 − 2k+1n+ 22k+1

)

olacaktır. Eger n > 1 ise her iki carpanda 1’den buyuk olacaktır. Dola-yısıyla n = 1 durumu tek asal oldugu durumdur.

7. P(x4

)= x16+6x8− 4x4+1 =

(x4 − 1

)4+4

(x3

)4olacaktır. Buna gore,

A = x4 − 1 ve B = x3 olarak alırsak P (x4) polinomu[(x4 − 1

)2+ 2

(x4 − 1

)x3 + 2

(x3

)2] [(x4 − 1

)2 − 2(x4 − 1

)x3 + 2

(x3

)2]

olur. Ispat tamamlanır.

8. Sophie Germain ozdesligini kullanırsak

n12 + 64 =(n6 − 4n3 + 8

) (n6 + 4n3 + 8

)

olarak yazabiliriz. Diger yandan, n12+64 ifadesi(n4 + 4

) (n8 − 4n4 + 16

)biciminde de yazılabilir. Burada

(n4 + 4

)=

(n2 − 2n+ 2

) (n2 + 2n+ 2

)olduguna gore ifademiz n12 =

(n2 − 2n+ 2

) (n2 + 2n+ 2

) (n8 − 4n4 + 16

)olacaktır. Simdi de (n6 − 4n3 + 8) ve (n6 + 4n3 + 8) carpanlarını incele-yelim. Buna gore,

n6 + 4n3 + 8 =(n2 − 2n+ 2

) (n4 + 2n3 + 2n2 + 4n+ 4

)

ven6 − 4n3 + 8 =

(n2 + 2n+ 2

) (n4 − 2n3 + 2n2 − 4n+ 4

)

olacagına gore

n12+64 =(n2 − 2n+ 2

) (n2 + 2n+ 2

) (n4 + 2n3 + 2n2 + 4n+ 4

) (n4 − 2n3 + 2n2 − 4n+ 4

)

olacaktır. Carpanlardan herbiri artan fonksiyonlar olduguna gore, hepsibirbirinden farklıdır.

9. Geometrik dizi toplamını kullanırsak,

m∑

k=0

(−4)k n4(m−k) =n4mn∑

k=0

(− 4

n

)k

=

(n4

)m+1+ 4m+1

n4 + 4=

(nm+1

)4+ 4

(2m/2

)4n4 + 4

Page 30: MATEMATİK OLİMPİYATI ÇALIŞMA KİTAPÇIĞI · C»˜oz um.˜ Bu soruda »c˜oz ume daha kolay i»slemlerle ula»smak i»cin baz‡ de‚gi»s-˜ ken e‚gi»stirmeler yapmak yerinde

2.3. SOPHIE GERMAIN OZDESLIGI 29

esitligini elde edebiliriz. Sophie Germain ozdesligini kullanarak kesrinpay kısmı carpanlarına ayrılabilir. Buna gore ifade,(n2(m+1) + 2

m2+1 · nm+1 + 2m+1

)(n2(m+1) − 2

m2+1 · nm+1 + 2m+1

)

olacaktır. m ≥ 2 oldugundan, payda kısmı paydaki iki carpandan da ku-cuk olacaktır. Sadelestirmelerden sonra bile, kalan sayı hala 1’den buyukiki sayının carpımı olacaktır. Buna gore, ispat tamamlanır.

10. Cozume baslamadan once Einstein Kriteri ’ni bir okuyalım.Einstein Kriteri. P (x) = anx

n + an−1xn−1 + · · · + a0, tamsayı kat-

sayılı bir polinom veriliyor. Varsayalım p asal sayı olmak uzere, p -an, p|ak, k = 1, 2, · · · , n − 1 ve p2 - a0 olsun. Buna gore, P (x) poli-nomu Z[x] uzerinde indirgenebilir.Biz en kucuk degerin 16 oldugunu gosterecegiz, ancak ilk 15 degerin ol-madıgını kanıtlamamız gerekiyor. Bunun icinde Einstein Kriteri’ni kul-lanacagız.n = 10, 11, 12, 13, 14, 15 icin kriterimiz 5, 11, 3, 13, 7 ve 5 asallarını uygu-layarak kontrol edebiliriz.n = 8 veya 9 durumunda, eger polinomumuz istenilen gibi carpanlarınaayrılabiliyorsa, carpanlar lineer olmalıdır. Ancak kontrol edilebilirki bupolinomların tamsayı kokleri olmayacaktır. Demek ki, n ≥ 16 olmalıdır.Bu durumda da, zaten sorumuz (8.) ormekte oldugu gibi cozulebilir.

Sophie Germain2 (1776-1831)

Babası zengin bir ipek tuccarıdır. Toplumda liberal reformların konusulupplanlandıgı bir ailenin kızı olarak 1776 yılında dogar. Daha 13 yasındaykenArsimet’in olum hikayesini okuduktan sonra matematikci olmaya karar ve-rir.Kendi kendine Latince ve Yunanca ogrenir. Ailesinin muhalefetine ragmen,anne ve babası uyduktan sonra Newton ve Euler’i okur. Felsefeye merak sarar.Bu kadar inatcı bir cocukla bas edemeyen babası sonunda Sophie’yi hayatıboyunca desteklemeye karar verir. Matematikteki zekasını ilk kez meshur ma-tematikci Lagrange kesfeder. Lagrange icin hazırladıgı bir odevi kadın oldu-gundan onem verilmeyecegi kaygısıyla “M. LeBlanc”diye sahte bir isimle verir.

2Hurriyet gazetesi yazarı, Erkan Kumcu’nun 9 Subat 2003 tarihli yazısından alınmıstır.

Page 31: MATEMATİK OLİMPİYATI ÇALIŞMA KİTAPÇIĞI · C»˜oz um.˜ Bu soruda »c˜oz ume daha kolay i»slemlerle ula»smak i»cin baz‡ de‚gi»s-˜ ken e‚gi»stirmeler yapmak yerinde

30 BOLUM 2. KONULAR

Lagrange bu dehanın Sophie Germain oldugunu daha sonra ogrenir. Sophie’ninmatematik alanında en buyuk destekcilerinden biri Lagrange olur. Sophie Ger-main’i en cok etkileyen matematikcilerden biri de cogu kesimlerin fikir birligiicinde matematigin prensi diye adlandırılan Gauss oldu. Ona da cesitli mate-matik konularında bir cok mektup yazdı. Aynı kaygıyla, mektuplarına uzunsure M. LeBlanc olarak imza attı.

Gauss, M. LeBlanc’ın Sophie Germain oldugunuFransızlar Gauss’un oturdugu sehri isgal edip Sop-hie’nin aile dostu olan bir Fransız generalden Ga-uss icin ayrıcalık istediginde ogrenir. Sophie Ger-main’in matematikteki meshur Fermat Teoremi’nincozumune yaptıgı katkılar bilinen en iyi yonudur.Yaptıgı katkıların onemi kendinden ancak 100 yılsonra Kummer tarafından bir adım ileri goturule-bildigi dusunulurse daha iyi anlasılır. Zamanın cokprestijli yarısmalarına katılmıstır. Poisson gibi mate-matik ve istatistigin onde gelen isimleriyle yarısmıs-tır. Basarılı olamamıstır. Hak ettigi dereceler hicbirzaman kendine verilmemistir. Gecmiste M. LeBlancismini kullanmakla ne kadar haklı oldugunu tum matematik dunyası adeteSophie’ye ispat etmistir.Poisson, Gaspard de Prony ve Laplace’dan olusan birjurinin seciciliginde katıldıgı bir yarısmada sundugu makale bazı teknik hatalarnedeniyle kabul dahi edilmemis ve kendisine calısmasının neden kabul edilme-digi soylenmemistir bile. Olaydan 55 yıl sonra Gaspard de Prony’nin yazdıgımakalelerinden birinin Sophie Germain’in yazdıgı makalenin duzeltilmis seklioldugu anlasılmıstır.Bir cok deha gibi, Sophie Germain de cok genc yasta oldu.

2.4 Tamkareler Pozıtıftır

Bu bolumde, cebirde oldukca basit bir esitsizlik olan

x2 ≥ 0, ∀x ∈ R

esitsizliginin bazı uygulamalarını yapacagız. Esitlik durumu yanlız ve yanlızx = 0 durumunda vardır. Simdi orneklerle konuyu kavramaya calısalım.

Page 32: MATEMATİK OLİMPİYATI ÇALIŞMA KİTAPÇIĞI · C»˜oz um.˜ Bu soruda »c˜oz ume daha kolay i»slemlerle ula»smak i»cin baz‡ de‚gi»s-˜ ken e‚gi»stirmeler yapmak yerinde

2.4. TAMKARELER POZITIFTIR 31

Ornek. 4x− x4 ≤ 3, x ∈ R esitsizligini kanıtlayınız.

Cozum. Bu ornek esitsizlikleri ogrenmeye baslayan genc bir ogrencitarafından ortaya atılmıstır. Henuz herhangi bir analiz kitabından esitsizliklerkonusunu calısmadıgını dusunursek fena bir ornek degildir. Buna gore,

x4 − 4x+ 3 > 0 ⇒ x4 − 2x2 + 1 + 2x2 − 4x+ 2 > 0

olacaktır. Buradan, (x2 − 1

)2+ 2 (x− 1)2 > 0

olacaktır ki, ispat tamamlanır.

Ornek. f(x2

)−(f (x))2 > 1/4 esitsizligini saglayan f : R → R ve birebirolan tum f fonksiyonlarını bulunuz.

Cozum. x = 0 alırsak, f(0) − (f(0))2 ≥ 1/4 ve (f (0)− 1/2)2 6 0olacaktır ki, bu durum imkansızdır ancak f(0) = 1/2 olabilir. Ancak x = 1alırsak bu seferde f(1) = 1/2 olacaktır. Ancak bu durum f fonksiyonun birebirolması durumu ile celisir.

Simdi alıstırmaları cozerek konuyu daha iyi kavramaya calısınız. Alıstır-malardaki yontemlere benzer yontemleri uygulayarak cozume gitmeye gayretediniz.

2.4.1 Calısma Soruları

1. n tane reel sayının ikiserli carpımları toplamı ve toplamları sıfırdır. Bunagore, bu sayıların kupleri toplamınında sıfır oldugunu kanıtlayınız.

2. a, b, c, d birer reel sayı olmak uzere, a−b2, b−c2, c−d2 ve d−a2 sayılarınıntumunun birden 1/4’ten buyuk olamayacagını kanıtlayınız.

3. x, y, z pozitif reel sayılarının hepsi 4 ten kucuktur. Buna gore,

1

x+

1

4− y,

1

y+

1

4− x,

1

z+

1

4− x

sayılarından en az bir tanesinin 1’den buyuk veya esit oldugunu kanıtla-yınız.

Page 33: MATEMATİK OLİMPİYATI ÇALIŞMA KİTAPÇIĞI · C»˜oz um.˜ Bu soruda »c˜oz ume daha kolay i»slemlerle ula»smak i»cin baz‡ de‚gi»s-˜ ken e‚gi»stirmeler yapmak yerinde

32 BOLUM 2. KONULAR

4. Asagıdaki denklem sisteminin tum reel cozumlerini bulunuz.

x+ y =√4z − 1

y + z =√4x− 1

z + x =√4y − 1

5. x, t ∈ (0, 1) olmak uzere, oyle bir a 6= 1 pozitif sayısı vardır ki,

logx a+ logy a = 4 logxy a

esitligi saglanmaktadır. Buna gore, x = y oldugunu kanıtlayınız.

6.

x4 + y4 + z4 − 4xyz = −1

esitligini saglayan tum (x, y, z) reel uclulerini bulunuz.

7.

2xy − z2 ≥ 1

z − |x+ y| ≥ −1

denklem sistemini saglayan tum (x, y, z) reel uclulerini bulunuz.

8. x4 + ax3 + 2x2 + bx + 1 denkleminin reel cozumu yoksa a2 + b2 ≥ 8olacagını gosteriniz.

9. a, b ve c reel sayılar olmak uzere a2+ c2 > 4b olarak veriliyor. Buna gorex ∈ R olmak uzere

x4 + ax3 + bx2 + cx+ 1 > 0

esitsizligini kanıtlayınız.

10.

x2 + y2 + z2 − xy − yz − xz > 3

4(x− y)2

esitsizliginin tum (x, y, z) reel degerleri ici saglandıgını kanıtlayınız.

Page 34: MATEMATİK OLİMPİYATI ÇALIŞMA KİTAPÇIĞI · C»˜oz um.˜ Bu soruda »c˜oz ume daha kolay i»slemlerle ula»smak i»cin baz‡ de‚gi»s-˜ ken e‚gi»stirmeler yapmak yerinde

2.4. TAMKARELER POZITIFTIR 33

11.

√x1 − 12 + 2

√x2 − 22 + · · · · · ·+ n

√xn − n2 =

1

2(x1 + x2 + · · ·+ xn)

esitligini saglayan tum (x1, x2, · · · , xn) reel sayılarını bulunuz.

12. a. a, b, c negatif olmayan reel sayılardır. Buna gore,

ab+ bc+ ca >√

3abc (a+ b+ c)

esitsizligini kanıtlayınız.

b. a, b, c negatif olmayan reel sayılardır, oyleki a+ b+ c = 1 veriliyor.Buna gore,

a2 + b2 + c2 +√12abc 6 1

esitsizligini kanıtlayınız.

13. k,m, n ∈ R olmak uzere verilen

f (km) + f (kn)− f (k) · f (mn) > 1

esitsizligini saglayan tum f : N→ R fonksiyonlarını bulunuz.

14. Bir dik paralelyuzun kenar uzunlukları a, b, c ve kosegen uzunlugu d ise

a2b2 + b2c2 + c2a2 > abcd√3

esitsizligini kanıtlayınız.

15. a1, a2, · · · , an reel sayılar olmak uzere

n∑

i=1

n∑

j=1

i · j · cos (ai − aj) > 0

esitsizligini kanıtlayınız.

Page 35: MATEMATİK OLİMPİYATI ÇALIŞMA KİTAPÇIĞI · C»˜oz um.˜ Bu soruda »c˜oz ume daha kolay i»slemlerle ula»smak i»cin baz‡ de‚gi»s-˜ ken e‚gi»stirmeler yapmak yerinde

34 BOLUM 2. KONULAR

2.4.2 Cozumler

1. Varsayalım sayılarımız a1, a2, · · · , an olsun. Soruda verilen sartlara gore,

a21+a22+· · ·+a2n = (a1 + a2 + · · ·+ an)2−2 (a1a2 + a1a3 + · · ·+ an−1an) = 0

olduguna gore, a1 = a2 = · · · = an = 0 olacagına gore,a31+a32+· · ·+a3n =0 olacaktır.

2. Varsayalım tumu birden,

a− b2 >1

4, b− c2 >

1

4, c− d2 >

1

4, d− a2 >

1

4

olsun. Eger bu dort esitsizligi toplarsak,

a+ b+ c+ d− (a2 + b2 + c2 + d2

)> 1

olacaktır. Buradan,

(1

2− a

)2

+

(1

2− b

)2

+

(1

2− c

)2

+

(1

2− d

)2

< 0

olacaktır ki, bu durum imkansızdır.

3. Soruya gore, varsayalım hepsi birden 1’den kucuk olsun. Buna gore,(1

x+

1

4− x

)+

(1

y+

1

4− y

)+

(1

z+

1

4− z

)< 3

olur. Diger taraftan(1

a+

1

4− a

)> 1, a < 4

olacagından (a− 2)2 ≥ 2 olacaktır. Ispat tamamlanır.

4. Cozum I. Bir onceki soruda oldugu gibi, uc denklemide taraf tarafatoplayıp, kareler tolamını elde etmeye calısalım. Buna gore,

2x+ 2y + 2z −√4x− 1−

√4y − 1−√

4z − 1 = 0

Page 36: MATEMATİK OLİMPİYATI ÇALIŞMA KİTAPÇIĞI · C»˜oz um.˜ Bu soruda »c˜oz ume daha kolay i»slemlerle ula»smak i»cin baz‡ de‚gi»s-˜ ken e‚gi»stirmeler yapmak yerinde

2.4. TAMKARELER POZITIFTIR 35

olacaktır. Bu esitligi oyle kareler toplamı seklinde yazalım ki, bunlardanbiri x, biri y ve biride z degiskenine baglı olsun. Eger tum denklemi 2 ilebolersek elimizde

x−√x− 1

4

olusacaktır. Eger bu denkleme 1/4 ekler cıkarırsak

x− 1

4−

√x− 1

4+

1

4=

(√x− 1

4− 1

2

)2

olusacaktır. Simdi bu yontemi digerleri ici uygularsak,

(√x− 1

4− 1

2

)2

+

(√y − 1

4− 1

2

)2

+

(√z − 1

4− 1

2

)2

= 0

olacaktır. Buna gore, x = y = z = 1/2 denklem sisteminin tek cozumuolarak bulunur.

Cozum II. Eger denklemlerin karelerini alıp toplarsak,

0 =((x+ y)2 − 4z + 1

)+

((y + z)2 − 4x+ 1

)+

((z + x)2 − 4y + 1

)

= (x+ y − 1)2 + (y + z − 1)2 + (z + x− 1)2

olacagından x+ y = y + z = z + x = 1 veya x = y = z = 1/2 bulunur.

5. a > 0 ve a 6= 1 olduguna gore esitlik

1

loga x+

1

loga y=

4

loga xy

veyaloga x+ loga y

loga x loga y=

4

loga x+ loga y

olarak yazılabilir. Eger icler dıslar carpımı yapılırsa,

(loga x+ loga y)2 = 4 loga x loga y

Page 37: MATEMATİK OLİMPİYATI ÇALIŞMA KİTAPÇIĞI · C»˜oz um.˜ Bu soruda »c˜oz ume daha kolay i»slemlerle ula»smak i»cin baz‡ de‚gi»s-˜ ken e‚gi»stirmeler yapmak yerinde

36 BOLUM 2. KONULAR

esitligi elde edilir. Buradan da,

(loga x− loga y)2 = 0

olacagından x = y oldugu acıktır.

6. Eger esitligin sol tarafını ilk iki terimi iceren bir tamkare olarak yazmayacalısırsak

−1 =(x2 − y2

)2+ 2x2y2 + z2 − 4xy

olacaktır. Bu noktadan sonra denkleme 2z2 ekler ve cıkarırsak bir tam-kare daha elde ederiz. Zaten bu basamaktan sonra yeni denklemimiz uckarenin toplamı

(x2 − y2

)2+

(z2 − 1

)2+ 2 (xy − z)2 = 0

olacaktır. Esitlik ancak hepsinin sıfır olması durumunda gecerlidir. Bunagore, z = ±1 olacagından istenen ucluler

(1, 1, 1) , (−1,−1, 1) , (−1, 1,−1) , (1,−1,−1)

olacaktır.

7. Cozum I. ikinci denklemden z ≥ |x + y| − 1 olacagından, bunu ilkdenklemde yerine koyarsak

2xy − (1− |x+ y|)2 > 1

olacaktır. Buna gore,

2xy − (1− |x+ y|)2 = 2xy − |x+ y|2 + 2 |x+ y| − 1

= 2xy − x2 − y2 − 2xy + 2 (±x± y)− 1

= −x2 − y2 + (±x± y)− 1

eger isaret secimimizi uygularsak, yeni denklem asagıdaki gibi olacaktır.Buna gore,

0 > x2 + y2 − 2 (±x± y) + 1 + 1 = (1± x)2 + (1± y)2

Page 38: MATEMATİK OLİMPİYATI ÇALIŞMA KİTAPÇIĞI · C»˜oz um.˜ Bu soruda »c˜oz ume daha kolay i»slemlerle ula»smak i»cin baz‡ de‚gi»s-˜ ken e‚gi»stirmeler yapmak yerinde

2.4. TAMKARELER POZITIFTIR 37

olacagından her iki karede sıfır olmalıdır. Yani x = y = ±1 degerlerinialacaklardır. Birinci denklemden xy > 0 olacagına gore x ve y aynıisaretli olmalıdır. Yani x = y = 1 veya x = y = −1 olmalıdır. Egerbu degerleri yerine koyarsak x = y = z = 1 ve x = y = 1, z = 1 cozumlerolacaktır.

Cozum II. Eger ikinci denklemi z + 1 ≥ |x+ y| olarak yazıp karesinialırsak

(x+ y)2 ≥ (x+ y)2

olacaktır. Birinci esitsizligi 2 ile carpıp son yazdıgımız esitsizligi toplarsak

0 ≥ (x− y)2 + (z − 1)2

olacaktır. Demek ki x = y ve z = 1 olmalıdır. Eger bu esitlikleri yerinekoyarsak 2x2 ≥ 2 ve 2 ≥ 2|x| ise |x| = 1 olmalıdır.

8.

x4+ax3+2x2+ bx+1 =(x2 +

a

2x)2

+

(1 +

b

2x

)2

+1

4

(8− a2 − b2

)x2

esitliginden bu ifadenin pozitif oldugunu soyleyebiliriz. Ancak, 8− a2 −b2 ≤ 0 ise yani a2 + b2 ≥ 8 olursa durum bozulur.

9.

x4 + ax3 + bx2 + cx+ 1 =(x2 +

a

2x)2

+

(b− a2 + c2

4

)x2 +

( c2x+ 1

)2

olduguna ve b ≥ (a2 + c2)/4 ise ifade daima sıfırdan buyuk esittir.

10. Eger esitsizligi duzenlersek

1

2

[(x− y)2 + (y − z)2 + (z − x)2

]> 3

4(x− y)2

olacagından

2[(y − z)2 + (z − x)2

]> (x− y)2

Page 39: MATEMATİK OLİMPİYATI ÇALIŞMA KİTAPÇIĞI · C»˜oz um.˜ Bu soruda »c˜oz ume daha kolay i»slemlerle ula»smak i»cin baz‡ de‚gi»s-˜ ken e‚gi»stirmeler yapmak yerinde

38 BOLUM 2. KONULAR

olacaktır. Eger a = y − z ve b = z − x alırsak

2(a2 + b2

)> (a+ b)2

olur ki, dogrudur, ispat tamamlanır.

11. Yine bu sorudada amacımız, verilen denklemi kareler toplamı bicimindeyazmak olacaktır.Buna gore denklemi 2 ile genisletip, hepsini sol tarafatoplarsak

(x1 + x2 + · · ·+ xn)− 2√x1 − 1− 4

√x2 − 22 − · · · − 2n

√xn − n2

ifadesini elde ederiz. Buna gore,

(x1 − 1− 2

√x1 − 1 + 1

)+(x2 − 22 − 4

√x2 − 22 + 22

)+· · ·+

(xn − n2 − 2n

√xn − n2 + n2

)

ifadesi elde edilir. Eger bu ifade de kareler toplamı olarak yazılırsa

(√x1 − 1− 1

)2+

(√x2 − 22 − 2

)2+ · · ·+

(√xn − n2 − n

)2

ifadesi elde edilir. Eger dikkat edilirse tum toplamın sıfır olabilmesi icintamkare ifadelerin de sıfır olması gerektigini gormek zor degildir. Bunagore,

x1 = 2, x2 = 8, · · · , xn = 2n2

olacaktır.

12. a. Esitsizligin iki tarafınında karesini alırsak

a2b2 + b2c2 + c2a2 + 2abc (a+ b+ c) > 3abc (a+ b+ c)

olduguna gore,

1

2

[(ab− bc)2 + (bc− ca)2 + (ca− ab)2

]> 0

olacaktır. Ispat tamamlanır.

Page 40: MATEMATİK OLİMPİYATI ÇALIŞMA KİTAPÇIĞI · C»˜oz um.˜ Bu soruda »c˜oz ume daha kolay i»slemlerle ula»smak i»cin baz‡ de‚gi»s-˜ ken e‚gi»stirmeler yapmak yerinde

2.4. TAMKARELER POZITIFTIR 39

b. a+ b+ c = 1 ise soruda verilen esitsizligi,

√12abc (a+ b+ c) 6 2 (a+ b+ c)2 − (

a2 + b2 + c2)

olarak yazabiliriz. Buna gore esitsizligimiz

√12abc (a+ b+ c) 6 2 (ab+ bc+ ac)

olacaktır. Zaten benzer esitsizligi (a.) sıkkında cozmustuk. Aynıyontemle siz tamamlayabilirsiniz.

13. Bu bolumun konu anlatım bolumunde cozdugumgz ornekte oldugu gibiburadada m,n ve k icin bazı degerleri kontrol edecegiz. Eger m = n =k = 0 alırsak

2f (0)− f2 (0) > 1

olacagından 0 > (f(0)− 1)2 olacaktırki, buradan f(0) = 1 olarak bulu-nur. Eger m = n = k = 1 alırsak, bu seferde f(1) = 1 olacaktır. Egerm = n = 0 alınırsa 2f(k) ≥ 1 olacagından f(k) ≤ 1 olarak bulunur.Benzer bicimde k = 1 ve m = 0 alırsak 1 + f(n) − 1 ≥ 1 ise f(n) ≥ 1olacaktır. Demek ki fonksiyonumuz f(x) = 1 fonksiyonu olmalıdır.

14. Elimizde dik bir paralelyuz olduguna gore

d =√

a2 + b2 + c2

olacagından esitsizlik

(a2b2 + b2c2 + c2a2

)2 > 3a2b2c2(a2 + b2 + c2

)

olacaktır. Eger parantezleri acar ve gruplama yaparsak,

c2

2

(a2 − b2

)2+

a4

2

(b2 − c2

)2+

b4

2

(c2 − a2

)2 > 0

olacagından soruda verilen esitsizlik kanıtlanmıs olur.

Page 41: MATEMATİK OLİMPİYATI ÇALIŞMA KİTAPÇIĞI · C»˜oz um.˜ Bu soruda »c˜oz ume daha kolay i»slemlerle ula»smak i»cin baz‡ de‚gi»s-˜ ken e‚gi»stirmeler yapmak yerinde

40 BOLUM 2. KONULAR

15. Kosinus icin fark formulunu kullanırsak,

n∑

i=1

n∑

j=1

ij cos (ai − aj) =n∑

i=1

n∑

j=1

ij (cos ai cos aj + ij sin ai sin aj)

=n∑

i=1

i cos ai

n∑

j=1

j cos aj+n∑

i=1

i sin ai

n∑

j=1

j sin aj

=

(n∑

i=1

i cos ai

)2

+

(n∑

i=1

j sin aj

)2

> 0

oldugundan ispat tamamlanmıs olur.

2.5 Esıtsızlıkler I

Esitsizlikleri cozerken sıklıkla sayıları ve matematiksel ifadeleri karsılastı-rırız. Yada bize verilen bir matematiksel ifadenin en buyuk yada en kucukdegerini bulmaya calısırız. Bu ders notumuzda esitsizlik sorularında sıklıklakarsımıza cıkan temel tip esitsizlikleri ve uygulamalarını gorecegiz.

Aritmetik Orta-Geometrik Orta-Harmonik Orta Esitsizligi. a1, a2, · · · , an >0 ve ai ∈ R icin,

AO =a1 + · · ·+ an

n≥ GO = n

√a1 · a2 · · · an ≥ HO =

n1a1

+ 1a2

+ · · ·+ 1an

olacaktır. Esitlik olması durumunda ise a1 = a2 = · · · = an olacaktır.

Ornek.(1)A0−GO esitsiligi ile, x > 0 olmak uzere,

x+1

x≥ 2

√x · 1

x= 2

olacaktır. Esitlik ise x = 1 durumunda saglanacaktır.

(2) AO −HO esitsizliginden eger a1, a2, · · · , an > 0 ise

(a1 + a2 + · · ·+ an)(1

a1+

1

a2+ · · ·+ 1

an) ≥ n2

Page 42: MATEMATİK OLİMPİYATI ÇALIŞMA KİTAPÇIĞI · C»˜oz um.˜ Bu soruda »c˜oz ume daha kolay i»slemlerle ula»smak i»cin baz‡ de‚gi»s-˜ ken e‚gi»stirmeler yapmak yerinde

2.5. ESITSIZLIKLER I 41

olacaktır.

(3) a, b, c > 0 ve abc = 1 ise (a + b + c)(ab + cb + ac) carpımının en kucukdegerini bulunuz.

Cozum.AO −GO esitsizliginden,

a+ b+ c

3≥ 3

√abc ve

ab+ bc+ ac

3≥ 3

√ab · bc · ac

ise

(a+ b+ c)(ab+ ac+ bc) ≥ 9

olacaktır. Buna gore istenen en kucuk deger 9 olacaktır.

(4) n ∈ Z+ ise

(1 +1

n)n ≤ (1 +

1

n+ 1)n+1

esitsizligini kanıtlayınız.

Cozum. Varsayalım a1 = a2 = · · · = an = 1 + 1n ve an+1 = 1 olarak alalım.

Buna gore, AO −GO esitsizliginden

AO =n(1 + 1

n) + 1

n+ 1= 1 +

1

n+ 1≥ GO =

n+1

√(1 +

1

n)n · 1

esitsizliginden soruda istenen durum elde edilir.

(5)(1964, IMO) a, b, c bir ucgenin kenar uzunluklarıdır. Buna gore,

a2(b+ c− a) + b2(c+ a− b) + c2(a+ b− c) ≤ 3abc

esitizligini kanıtlayınız.

Cozum. Varsayalım,

x =a+ b− c

3, y =

b+ c− a

2, z =

c+ a− b

2

olarak alalım. Buradan a = x+z, b = x+y, c = y+z olacagından esitsizligimiz

(x+ z)22y + (x+ y)22z + (y + z)22x ≤ 3(z + x)(x+ y)(y + z)

Page 43: MATEMATİK OLİMPİYATI ÇALIŞMA KİTAPÇIĞI · C»˜oz um.˜ Bu soruda »c˜oz ume daha kolay i»slemlerle ula»smak i»cin baz‡ de‚gi»s-˜ ken e‚gi»stirmeler yapmak yerinde

42 BOLUM 2. KONULAR

olacaktır. Eger son buldugumuz son esitsizligi duzenlersek,

x2y + y2z + z2x+ xy2 + yz2 + zx2 ≥ 6xyz

esitsizligini elde ederiz. Buradan esitsizligin sol tarafına AO − GO esitsizligiuygulanırsa soruda istenen esitsizlik kanıtlanmıs olur.

(6)(2008, AUMO3) n dogal sayısının kac tane degeri icin,

x1 + x2 + · · ·+ xn = 91

x1+

1

x2+ · · ·+ 1

xn= 1

denklem siteminin pozitif reel sayılarda cozumu vardır?

Cozum. AO −HO esitsizligini kullanırsak,

n1x1

+ 1x2

+ · · ·+ 1xn

≤ x1 + x2 + · · ·+ xnn

⇒ n

1≤ 9

n⇒ n2 ≤ 9

olduguna gore, n = 1, 2, 3 degerlerini alabilir. Ancak, n = 1 degeri icin dogru-lanmadıgı acıktır. Oyleyse sadece 2 ve 3 icin cozumludur.

Cauchy-Schwartz Esitsizligi. a1, a2, · · · , an ve b1, b2, · · · , bn reel sayılarıicin

(a1b1 + a2b2 + · · ·+ anbn)2 ≤ (a21 + a22 + · · ·+ a2n)(b

21 + b22 + · · ·+ b2n)

esitsizligi vardır. Esitlik durumu aibj = aibj , i, j = 1, · · · , n icin vardır.

(7) 0 ≤ θ < 2π icin

a · cos θ + b · sin θifadesinin alabilecegi en buyuk ve en kucuk degerleri bulunuz.

Cozum. C.S. esitsizliginden

(a cos θ + b sin θ)2 ≤ (a2 + b2)(cos2 θ + sin2 θ)

3Antalya Universitesi Matematik Olimpiyatları, 2008

Page 44: MATEMATİK OLİMPİYATI ÇALIŞMA KİTAPÇIĞI · C»˜oz um.˜ Bu soruda »c˜oz ume daha kolay i»slemlerle ula»smak i»cin baz‡ de‚gi»s-˜ ken e‚gi»stirmeler yapmak yerinde

2.5. ESITSIZLIKLER I 43

esitsizliginden,

−√a2 + b2 ≤ a cos θ + b sin θ ≤

√a2 + b2

aralıgı bulunur.

(8)(1978, USAMO4) a, b, c, d, e reel sayıları icin a + b + c + d + e = 8 vea2 + b2 + c2 + d2 + e2 = 16 ise, e’nin alabilecegi en buyuk degeri bulunuz.

Cozum. C.S. esitsizliginden,

(a+ b+ c+ d)2 ≤ (1 + 1 + 1 + 1)(a2 + b2 + c2 + d2)

ise(8− e)2 ≤ 4(16− e2)

olacaktır. Buradan da, e(5e−16) ≤ 0 ise 0 ≤ e ≤ 16/5 olacaktır. C.S. esitsizli-ginin, esitlik duruu kullanılırsa a = b = c = d = 6/5 ve emax = 16/5 olacaktır.

(9) a, b, c > 0 ve abc = 1 ise

1

a3(b+ c)+

1

b3(a+ c)+

1

c3(a+ b)≥ 3

2

esitizligini kanıtlayınız.

Cozum. x = 1a = bc, y = 1

b = ac, z = 1c = ab alırsak, esitsizligin son hali

x2

z + y+

y2

z + x+

z2

x+ y≥ 3

2

olacaktır. Burada,

x+ y + z =x√z + y

√z + y +

y√x+ z

√x+ z +

z√x+ y

√x+ y

esitligine C.S.E. esitsizligini uygularsak,

(x+ y + z)2 ≤ (x2

z + y+

y2

x+ z+

z2

x+ y)((z + y) + (x+ z) + (y + x))

4United States Of America Math Olympiads, 1978

Page 45: MATEMATİK OLİMPİYATI ÇALIŞMA KİTAPÇIĞI · C»˜oz um.˜ Bu soruda »c˜oz ume daha kolay i»slemlerle ula»smak i»cin baz‡ de‚gi»s-˜ ken e‚gi»stirmeler yapmak yerinde

44 BOLUM 2. KONULAR

olacaktır. Son esitsizligi ve A.O.−G.O. esitsizliklerini kullanırsak,

x2

z + y+

y2

z + x+

z2

x+ y≥ x+ y + z

3≥ 3

23√xyz =

3

2

bulunur.

Yeniden Duzenleme (Permutasyon) Esitsizligi. a1 ≥ a2 ≥ · · · ≥ an veb1 ≥ b2 ≥ · · · ≥ bn ise,

a1b1+a2b2+ · · ·+anbn ≥ a1br1+a2br2+ · · ·+anbrn ≥ a1bn+a2bn−1+ · · ·+anb1

esitsizligi vardır. Burada (br1 , br2 , · · · , brn) dizilimi (b1, b2, · · · , bn) dizilimininbir permutasyonudur.

(10)(1978, IMO5) c1, c2, · · · , cn farklı pozitif tamsayılardır. Buna gore,

c1 +c222

+ · · ·+ cnn2

≥ 1 +1

2+ · · ·+ 1

n

esitsizligini kanıtlayınız.

Cozum. Varsayalım (a1, a2, · · · , an) dizilimi ci’lerin artan sırayla dizilimi ol-sun. ai’ler farklı pozitif tamsayılar olduguna gore, a1 ≥ 1, a2 ≥ 2,· · · , an ≥ ndiyebiliriz. Burada,

a1 < a2 < · · · < an

ve

1 >1

22>

1

32> · · · > 1

n2

ise Y.D.E.’ne gore

c1 +c222

+ · · ·+ cnn2

≥ a1 +a222

+ · · ·+ ann2

≥ 1 +2

22+ · · ·+ n

n2

olacaktır.

(11) Ornek (9)’u Y.D.E. kullanarak yapınız.

5International Math Olympiads, 1978

Page 46: MATEMATİK OLİMPİYATI ÇALIŞMA KİTAPÇIĞI · C»˜oz um.˜ Bu soruda »c˜oz ume daha kolay i»slemlerle ula»smak i»cin baz‡ de‚gi»s-˜ ken e‚gi»stirmeler yapmak yerinde

2.5. ESITSIZLIKLER I 45

Cozum. x, y, z icin tanımlamalarımız (9)’daki gibi olsun. Genelligi kaybet-meden x ≥ y ≥ z alalım. xyz = 1 ve x2 ≥ y2 ≥ z2 olduguna gore 1/(z + y) ≥1/(x+ z) ≥ 1/(y+x) uclusude ikinci dizilimimiz olsun. Bu noktada Y.D.E.’yiiki defa uygularsak,

x2

z + y+

y2

z + x+

z2

x+ y≥ x2

x+ y+

y2

z + y+

z2

x+ z

x2

z + y+

y2

z + x+

z2

x+ y≥ x2

z + x+

y2

x+ y+

z2

z + y

esitsizliklerini taraf tarafa toplarsak,

x2

z + y+

y2

z + x+

z2

x+ y≥ 1

2(y2 + x2

y + x+

z2 + y2

y + z+

z2 + x2

z + x)

esitsizligini elde ederiz.

a2 + b2 ≥ a2 + b2

2

esitsizligini, sag tarafın payları icin kullandıktan sonra A.O.−G.O. esitsizliginikullanırsak,

x2

z + y+

y2

z + x+

z2

x+ y≥ 1

2(y + x

2+z + y

2+x+ z

2) =

x+ y + z

2≥ 3

23√xyz =

3

2

esitsizligi elde edilir.

Chebyshev Esitsizligi. Eger a1 ≥ a2 ≥ · · · ≥ an ve b1 ≥ b2 ≥ · · · ≥ bn ise

a1b1 + a2b2 + · · ·+ anbn ≥ (a1 + · · ·+ an)(b1 + · · ·+ bn)

n

esitsizligi vardır.

(12)(1974, USAMO6) a, b, c > 0 ise

aabbcc ≥ (abc)(a+b+c)/3

6United States Of America Math Olympiads, 1974

Page 47: MATEMATİK OLİMPİYATI ÇALIŞMA KİTAPÇIĞI · C»˜oz um.˜ Bu soruda »c˜oz ume daha kolay i»slemlerle ula»smak i»cin baz‡ de‚gi»s-˜ ken e‚gi»stirmeler yapmak yerinde

46 BOLUM 2. KONULAR

oldugunu kanıtlayınız.

Cozum. Uclulerimizi (a, b, c) ve (log a, log b, log c) olarak secersek,

a log a+ b log b+ c log c ≥ (a+ b+ c)(log a+ log b+ log c)1

3

log aabbcc ≥ (a+ b+ c)

3log(abc)

log aabbcc ≥ log(abc)(a+b+c)/3

aabbcc ≥ (abc)(a+b+c)/3

olacaktır.

(13) 0 ≤ ak < 1, k = 1, 2, 3, · · · , n ve S = a1 + a2 + · · ·+ an ise

n∑

k=1

ak1− ak

≥ nS

n− S

esitsizligini kanıtlayınız.

Cozum. Genelligi kaybetmeden a1 ≥ a2 ≥ · · · an ≥ 0 alabiliriz. Buna gore,

0 < 1− a1 ≤ 1− a2 ≤ · · · ≤ 1− an vea1

1− a1≥ a2

1− a2≥ · · · ≥ an

1− an

olacaktır. Chebysev esitsizliginden,

S =a1

1− a1(1− a1) +

a21− a2

(1− a2) + · · ·+ an1− an

(1− an)

≤ 1

n

n∑

k=1

ak1− ak

n∑

k=1

(1− ak) =n− S

n

n∑

k=1

ak1− ak

olacaktır.

Matematikte ve tabii ki istatistikte sıklıla ortalamalrı kullanmaya ihtiyacduyarız. AO, GO, HO dısında kullandıgımız Kuvvet Ortalaması ve SimetrikOrtalarda vardır. Aslında, bu iki ortalama AO ve GO’yıda ozel birer durumolarak icerir.

Page 48: MATEMATİK OLİMPİYATI ÇALIŞMA KİTAPÇIĞI · C»˜oz um.˜ Bu soruda »c˜oz ume daha kolay i»slemlerle ula»smak i»cin baz‡ de‚gi»s-˜ ken e‚gi»stirmeler yapmak yerinde

2.5. ESITSIZLIKLER I 47

Kuvvet Ortalaması Esitsizligi. a1, a2, a3, · · · , an > 0 ve s < t icin

Ms = (as1 + as2 + · · ·+ asn

n)1/s ≤ Mt = (

at1 + at2 + · · ·+ atnn

)1/t

esitsizlikleri vardır. Esitlik a1 = a2 = · · · = an durumunda vardır.

Not. Bu esitsizlikte M1 = A.O., M−1 = H.O. ve M2 ise Karesel Ortalamadır.Karesel Orta istatistik ve fizikte kullanılır. Ayrıca, limitlerini alırsak, M+∞ifadesiMAX = max{a1, a2, · · · , an} veM0 ifadesi Geometrik Orta ve deM−∞ifadesi MIN = min{a1, a2, · · · , an} olacaktır. Dolayısıyla elimizde,

MAX ≥ KO ≥ AO ≥ GO ≥ HO ≥ MIN

esitsizligi olusacaktır.

Maclaurin Simetrik Orta Esitsizligi.a1, a2, · · · , an > 0 icin

AO ≥ S1 ≥ S1/22 ≥ · · · ≥ S1/n

n = GO

olacaktır. Mesela, Sj ifadesine n = 4 icin bakalım

S1 =a1 + a2 + a3 + a4

4

S2 =a1a2 + a1a3 + a1a4 + a2a3 + a2a4 + a3a4

6

S3 =a1a2a3 + a1a2a4 + a1a3a4 + a2a3a4

4S4 = a1a2a3a4

olacaktır.

(14) x, y, z pozitif sayılar olduguna gore,

x5 + y5 + z5 ≤ x5

√x2

yz+ y5

√y2

zx+ z5

√z2

xy

esitsizligini kanıtlayınız.

Page 49: MATEMATİK OLİMPİYATI ÇALIŞMA KİTAPÇIĞI · C»˜oz um.˜ Bu soruda »c˜oz ume daha kolay i»slemlerle ula»smak i»cin baz‡ de‚gi»s-˜ ken e‚gi»stirmeler yapmak yerinde

48 BOLUM 2. KONULAR

Cozum. Varsayalım a =√x, b =

√y, c =

√z olsun. Buna gore esitsizligimiz,

a10 + b10 + c10 ≤ a13 + b13 + c13

abc

olacaktır. Buna gore, KOE’yi kullanırsak

a13 + b13 + c13 = 3M1313 = 3M10

13M313 ≥ 3M10

10M30 = (a10 + b10 + c10)abc

olacaktır.

(15) a, b, c > 0 ise,1

a+

1

b+

1

c≤ a8 + b8 + c8

a3b3c3

esitsizligini kanıtlayınız.

Cozum. Esitsizligi duzenlersek,

a8 + b8 + c8 ≥ a3b3c3(1

a+

1

b+

1

c) = (abc)2(bc+ ac+ ab)

olacaktır. KOE ve SOE’yi kullanırsak,

a8 + b8 + c8 = 3M88 ≥ 3M8

1 = 3S81 = 3S6

1S21

≥ (S1/33 )63(S

1/22 )2 = (abc)2(bc+ ac+ ab)

olacaktır.

(16) a1, a2, · · · , an ≥ 0 ve (1 + a1)(1 + a2) · · · (1 + an) = 2n ise,

a1 · a2 · · · · · an ≤ 1

oldugunu gosteriniz.

Cozum. SO esitsizliginden,

2n = (1 + a1)(1 + a2) · · · (1 + an)

= 1 + nS1 +

(n

2

)S2 + · · ·+

(n

n− 1

)Sn−1 + Sn

≥ 1 + nS1/nn +

(n

2

)S2/nn + · · ·+

(n

n− 1

)S

n−1n

n + Sn = (1 + S1/nn )n

ise 2 ≥ 1 + S1/nn ve 1 ≥ Sn = a1 · a2 · · · · · an olur.

Page 50: MATEMATİK OLİMPİYATI ÇALIŞMA KİTAPÇIĞI · C»˜oz um.˜ Bu soruda »c˜oz ume daha kolay i»slemlerle ula»smak i»cin baz‡ de‚gi»s-˜ ken e‚gi»stirmeler yapmak yerinde

2.5. ESITSIZLIKLER I 49

2.5.1 Calısma Soruları

1. Konu anlatımında verilen tum ornekleri cozumlerine bakmadan yapınız.

2. x1, x2, · · · , xn > 0 icin

x21x2

+x22x3

+ · · ·+ x2nx1

≥ x1 + x2 + · · ·+ xn

esitsizligini kanıtlayınız.

3. 0 < a, b, c < 1 ve a+ b+ c = 2 ise

8(1− a)(1− b)(1− c) ≤ abc

esitsizligini kanıtlayınız.

4. Eger a, b, c, d > 0 ve c2 + d2 = (a2 + b2)3 ise

a3

c+

b3

d≥ 1

esitsizligini kanıtlayınız.

5. a1, a2, · · · , an > 0 ve a1 + a2 + · · ·+ an = 1 ise

(a1 +1

a1)2 + (a2 +

1

a2)2 + · · ·+ (an +

1

an)2

ifadesinin en kucuk degerini bulunuz.

6. Eger a, b, c, d > 0 ve S = a2 + b2 + c2 + d2 ise

a3 + b3 + c3

a+ b+ c+

a3 + b3 + d3

a+ b+ d+

a3 + c3 + d3

a+ c+ d+

b3 + c3 + d3

b+ c+ d≥ S

esitsizligini kanıtlayınız.

7. Eger x1, x2, · · · , xn > 0 ve x1 + x2 + · · ·+ xn = 1 ise,

n∑

k=1

xk√1− xk

≥ 1√n− 1

n∑

k=1

√xk

esitsizligini kanıtlayınız.

Page 51: MATEMATİK OLİMPİYATI ÇALIŞMA KİTAPÇIĞI · C»˜oz um.˜ Bu soruda »c˜oz ume daha kolay i»slemlerle ula»smak i»cin baz‡ de‚gi»s-˜ ken e‚gi»stirmeler yapmak yerinde

50 BOLUM 2. KONULAR

8. a, b, c bir ucgenin kenar uzunlukları olduguna gore,

a2b(a− b) + b2c(b− c) + c2a(c− a) ≥ 0

esitsizligini kanıtlayınız.

2.6 Esıtsızlıkler II

Bazı fonksiyon esitsizliklerini kanıtını yaparken, fonksiyonunun belli ara-lıklardaki seklide onemlidir. Bu ders notumuzda ele aldıgımız esitsizliklerincozumleride bu temel prensiplere uyularak yapılmıstır.

Isınma

Tanım [Konveks vs. Konkav]. I aralıgı uzerinde sureli bir f fonksiyonu,

f(x1 + x2

2) ≤ f(x1) + f(x2)

2x1, x2 ∈ I

esitsizligini saglıyorsa f fonksiyonuna konveks denir. Eger I aralıgı uzerindef fonksiyonu konveks ve esitlik hali x1 = x2 oluyorsa, f tam konveks olur. Iaralıgında −f konveks ise f fonksiyonu konkav olur. Bu durumda,

f(x1 + x2

2) ≥ f(x1) + f(x2)

2x1, x2 ∈ I

olacaktır. Benzer bicimde eger −f tam konveks ise f fonksiyonuda konveksolur.

Tanım [Ikinci Turev Testi]. Eger I = (a, b) aralıgında f ′′(x) ≥ 0 oluyorsa,f fonksiyonu konvekstir. Eger f ′′(x) > 0 oluyorsa, f fonksiyonu tam konveksolur. Konkav ve tam konkav icinde tanım benzer bicimdedir. Sadece esitsizlikyon degistirir. Bir fonksiyonun konveksiligini gostermek icin sınır noktalarınıiceren bir aralıkta ve bu aralıkta surekli olması ile ikinci turev testinin negatifolmaması yeterlidir. Ikinci turev testini kullanarak, asagıda verilen fonksiyo-ların tam konvex oldugunu soyleyebiliriz.

xp ∈ [0,∞) , p > 1 yada xp ∈ (0,∞), p < 0

ax ∈ (−∞,∞), a > 1 yada tanx ∈ [0, π/2)

Page 52: MATEMATİK OLİMPİYATI ÇALIŞMA KİTAPÇIĞI · C»˜oz um.˜ Bu soruda »c˜oz ume daha kolay i»slemlerle ula»smak i»cin baz‡ de‚gi»s-˜ ken e‚gi»stirmeler yapmak yerinde

2.6. ESITSIZLIKLER II 51

Benzer bicimde asagıda verilen fonksiyonlarda tam konkavdır.

xp ∈ [0,∞) , 0 < p < 1 yada loga x ∈ (0,∞), a > 1

cosx ∈ [−π/2, π/2), yada sinx ∈ [0, π/2π)

Bu noktada, konveksligi ve konkavlıgı esitsizlik sorularında kullanabilecegimizen guzel yer sanırız Jensen Esitsizligi ’dir.

Mevzu

Tanım [Jensen Esitsizligi]. f fonksiyonu I uzerinde konveks ve x1, x2, · · · , xn ∈I ise

f

(x1 + x2 + x3

n

)≤ f(x1) + f(x2) + · · ·+ f(xn)

n

olacaktır. Burada esitlik durumu ancak ve ancak x1 = x2 = · · · = xn esitligindeolur.

Tanım [Genellestirilmis Jensen Esitsizligi]. f konveks ve I aralıgındasurekli olmak uzere, x1, x2, · · · , xn ∈ I ve 0 < t1, t2, · · · , tn < 1, t1 + t2 + · · ·+tn = 1 ise,

f(t1x1 + t2x2 + · · ·+ tnxn) ≤ t1f(x1) + t2f(x2) + · · ·+ tnf(xn)

Sekil 2.1: Aralıkta Bir Konveks Fonksiyon

Page 53: MATEMATİK OLİMPİYATI ÇALIŞMA KİTAPÇIĞI · C»˜oz um.˜ Bu soruda »c˜oz ume daha kolay i»slemlerle ula»smak i»cin baz‡ de‚gi»s-˜ ken e‚gi»stirmeler yapmak yerinde

52 BOLUM 2. KONULAR

olacaktır. Konkav fonksiyonlarda ise esitsizlik yon degistirir.

Egzersiz

[1.] Bir ABC ucgeninde,

sinA+ sinB + sinC ≤ 3√3

2

esitsizligini gosteriniz. Esitlik durumu hangi durumda ortaya cıkar, acıklayınız.

Cozum. f(x) = sinx fonksiyonu [0, π] arasında konkavdır. Oyleyse,

sinA+sinB+sinC = f(A)+f(B)+f(C) ≤ 3f

(A+B + C

3

)= 3 sin

(A+B + C

3

)=

3√3

2

olacaktır. Esitlik durumu ancak ve ancak A = B = C = π/3 yani ABC bireskenar ucgense gerceklesir.

[2.] a, b, c > 0 ve a+ b+ c = 1 ise,

(a+1

a)10 + (b+

1

b)10 + (c+

1

c)10

ifadesinin en kucuk degerini bulunuz.

Cozum. 0 < a, b, c < 1 olarak zaten verilmis. f(x) = (x+ 1x)

10 ise f fonksiyonuI = (0, 1) aralıgında konveksdir. Cunku,

f ′′(x) = 90(a+1

x)8(x− 1

x2)2 + 10(x+

1

x)9(

2

x3) > 0 olacaktır.

Oyleyse JE’den

f(a)+f(b)+f(c) = (a+1

a)10+(b+

1

b)10+(c+

1

c)10 ≥ 3f(

a+ b+ c

3) = 3f(

1

3) =

1010

39

olarak bulunur.

Cozum.(Alternatif Yontem) Soruda verilen esitsizligi cozmenin bir digeryontemide Chebychev Esitsizligini kullanmak olabilirdi. Bu yontemi size bıra-kıyoruz.7

7Chebychev Esitsizligi ile alakalı olarak www.sbelian wordpress.com adresinden Ye-niden Duzenleme Esitsizligi Ders Notları’nı indirebilirsiniz.

Page 54: MATEMATİK OLİMPİYATI ÇALIŞMA KİTAPÇIĞI · C»˜oz um.˜ Bu soruda »c˜oz ume daha kolay i»slemlerle ula»smak i»cin baz‡ de‚gi»s-˜ ken e‚gi»stirmeler yapmak yerinde

2.6. ESITSIZLIKLER II 53

[3.] Aritmetik Orta - Geometrik Orta esitsizligi JE kullanarak gostermeye ca-lısalım. Buna gore, eger a1, a2, a3, · · · , an ise f(x) = log x’de (0,∞) aralıgındakonkav olduguna gore,

log(a1 + a2 + · · ·+ an

n) ≥ log a1 + log a2 + · · ·+ log an

n= log( n

√a1a2 · · · an)

ise istenen esitsizlik kanıtlanmıs olur.

[4.](Holder) p, q > 1, 1p +

1q = 1 ve a1, a2, · · · , an b1, b2, · · · , bn reel sayılarsa,

∣∣∣∣∣n∑

i=1

aibi

∣∣∣∣∣ ≤(

n∑

i=1

|ai|p)1/p( n∑

i=1

|bi|q)1/q

esitsizligini kanıtlayınız.

Cozum. Varsayalım

A =

n∑

i=1

|ai|p ve B =

n∑

i=1

|bi|q

olsun. Eger A veya B sıfır ise, ya tum ai’ler yada tum bi’ler sıfırdır. Bu da zatenesitsizligin iki tarafınıda sıfır yapar. Buna gore biz A 6= 0 ve B 6= 0 durumunuinceleyelim. Varsayalım t1 = 1

p ve t2 = 1q olsun. Oyleyse, 0 < t1, t2 < 1 ve

t1 + t2 = 1 olacaktır. Eger

xi =|ai|pA

ve yi =|bi|qB

isen∑

i=1

xi = 1 ve

n∑

i=1

yi = 1

olur. f(x) = ex fonksiyonu (−∞,+∞) aralıgında konveks oldugundan Genel-lestirilmis Jensen Esitsizligini kullanabiliriz. Buna gore,

x1/pi · y1/qi = f(t1 lnxi + t2 ln yi) ≤ t1f(lnx1) + t2f(ln yi) =

xip

+yiq

Page 55: MATEMATİK OLİMPİYATI ÇALIŞMA KİTAPÇIĞI · C»˜oz um.˜ Bu soruda »c˜oz ume daha kolay i»slemlerle ula»smak i»cin baz‡ de‚gi»s-˜ ken e‚gi»stirmeler yapmak yerinde

54 BOLUM 2. KONULAR

olacaktır. Buna gore,

n∑

i=1

|ai||bi|A1/pB1/q

≤ 1

p

n∑

i=1

xi +1

q

n∑

i=1

yi = 1

olacaktır. Bundan dolayıda,

∣∣∣∣∣n∑

i=1

aibi

∣∣∣∣∣ ≤n∑

i=1

|ai||bi| ≤ A1/pB1/q =(∑

|ai|p)1/p (∑

|bi|q)1/q

bulunur

Simdi de, Jensen esitsiliginin bir baska uygulamasına gecelim.

Tanım (Majorization8). Eger x1, · · · , xn ve y1, · · · , yn asagıdaki sartlarısaglıyorsa, yani

x1 ≥ x2 ≥ · · · ≥ xn, y1 ≥ y2 ≥ · · · ≥ yn

x1 ≥ y1, x1 + x2 ≥ y1 + y2, · · · , x1 + x2 + · · ·+ xn−1 ≥ y1 + y2 + · · ·+ yn−1

ve

x1 + x2 + · · ·+ xn = y1 + y2 + · · ·+ yn

ise (x1, x2, · · · , xn) majorize (y1, y2, · · · , yn) denir ve

(x1, x2, · · · , xn) Â y1, y2, · · · , yn)

ile gosterilir.

Tanım (Majorization Esitsizligi). I = [a, b] aralıgında f fonksiyonu kon-veks ve

(x1, x2, · · · , xn) Â y1, y2, · · · , yn), xi, yi ∈ I

ise

f(x1) + f(x2) + · · ·+ f(xn) ≥ f(y1) + f(y2) + · · ·+ f(yn)

olur. Yanlız ve yanlız xi = yi durumu icin esitlik vardır. Konkav fonksiyonlaricinse esitsizlik yon degistirir.

8http://en.wikipedia.org/wiki/Majorization

Page 56: MATEMATİK OLİMPİYATI ÇALIŞMA KİTAPÇIĞI · C»˜oz um.˜ Bu soruda »c˜oz ume daha kolay i»slemlerle ula»smak i»cin baz‡ de‚gi»s-˜ ken e‚gi»stirmeler yapmak yerinde

2.6. ESITSIZLIKLER II 55

[5.] Dar acılı bir ABC ucgeni icin,

1 ≤ cosA+ cosB + cosC ≤ 3

2

esitsizligini kanıtlayınız.

Cozum. Genelligi kaybetmeden, varsayalım A ≥ B ≥ C olsun. Buna goreA ≥ π/3 ve C ≤ π/3 olacaktır. π/2 ≥ A ≥ π/3, π ≥ A+B = (−π − C) ≥ 2π

3olacagından

2,π

2, 0) Â (A,B,C) Â (

π

3,π

3,π

3)

alabiliriz. f(x) = cosx fonksiyonu [0, π/2] aralıgında konkav olduguna gore,majorization teoreminden,

1 = f(π

2) + f(

π

2) + f(0) ≤ f(A) + f(B) + f(C) ≤ f(

π

3) + f(

π

3) + f(

π

3)

ise soruda gostermemiz istenen,

1 ≤ cosA+ cosB + cosC ≤ 3

2

elde edilir.

[6.] Eger x1 ≥ x2 ≥ · · · ≥ xn ise

(x1, x2, · · · , xn) Â (x, x, x, · · · , x)

durumu vardır. Burada x degeri, x1, x2, · · · , xn degerlerinin aritmetik ortası-dır.(Bunu Majorization uzerine uygularsak Jensen Esitsizligini elde ederiz.)Buna gore, k = 1, 2, · · · , n− 1 icin x1 + x2 + · · ·+ xk ≥ kx durumunu goster-memiz yeterli olacaktır. Buna gore,

(n− k)(x1 + x2 + · · ·+ xk) ≥ (n− k)kxk ≥ k(n− k)xk+1 ≥ k(xk+1 + · · ·+ xn)

olacagından(n− k)(x1 + · · ·+ xk) ≥ k(xk+1 + · · ·+ xn)

bulunur. Bu esitsizligin iki tarafınada k(x1 + · · ·+ xk) eklersek

n(x1 + · · ·+ xn) ≥ k(x1 + · · ·+ xn) = knx

Page 57: MATEMATİK OLİMPİYATI ÇALIŞMA KİTAPÇIĞI · C»˜oz um.˜ Bu soruda »c˜oz ume daha kolay i»slemlerle ula»smak i»cin baz‡ de‚gi»s-˜ ken e‚gi»stirmeler yapmak yerinde

56 BOLUM 2. KONULAR

olacagındanx1 + x2 + · · ·+ xk ≥ kx

olacaktır.

[7.] −1 ≤ a, b, c ≤ 1 ve a+ b+ c = 1/2 ise a12 + b12 + c12 en fazla kac olur?

Cozum. [−1, 1] aralıgında f(x) = x12 fonksiyonu konvekstir. Oyleki, f ′′(x) =132x10 ≥ 0 olacaktır. Eger, 1 ≥ a ≥ b ≥ c ≥ −1 ve a+ b+ c = −1/2 ise majoruclulerimizi

(1,−1

2,−1) Â (a, b, c)

olarak secebiliriz. Cunku, 1 ≥ a ve 12 = 1− 1

2 ≥ −c− 12 = a+ b olacaktır. Buna

gore, majorization esitsizliginden

a12 + b12 + c12 = f(a) + f(b) + f(c) ≤ f(1) + f(2) + f(−1) = 2 +1

212

olacaktır. Zaten, a = 1, b = −12 ve c = −1

2 icinde en buyuk deger dogrulanır.

[8.](1999,IMO9) n ≥ 2 bir tamsayıdır. Buna gore,

(a.)

1≤i<j≤n

xixj(x2i + x2j ) ≤ C

1≤i≤n

xi

4

esitsizligini x1, x2, x3, · · · , xn ≥ 0 reel sayıları icin saglayan en kucuk Csabitini bulunuz.

(b.) Bu C degeri icin, esitlik durumunu arastırınız.

Cozum. Ilk olarak n = 2 durumuna bakalım. Varsayalım x1 = m + h vex2 = m− h yani

m =x1 + x2

2ve h =

x1 − x22

olsun. Buna gore,

x1x2(x21 + x22) = 2(m4 − h4) ≤ 2m4 =

1

8(x1 + x2)

4

9International Math Olympiads, 1999

Page 58: MATEMATİK OLİMPİYATI ÇALIŞMA KİTAPÇIĞI · C»˜oz um.˜ Bu soruda »c˜oz ume daha kolay i»slemlerle ula»smak i»cin baz‡ de‚gi»s-˜ ken e‚gi»stirmeler yapmak yerinde

2.6. ESITSIZLIKLER II 57

olacaktır.n > 2 durumu icin varsayalım

ai =xi

x1 + x2 + · · ·+ xnve a1 + a2 + · · ·+ an = 1 olsun.

Buna gore, ai = [0, 1] olacaktır. Eger, ai cinsinden yazarsak, ispatlanacakesitsizlik ∑

1≤i<j≤n

aiaj(a2i + a2j ) ≤ C

olacaktır. Esitsizligin sol tarafı acılıp duzenlenirse,n∑

i=1

a3i (a1 + · · ·+ ai−1 + ai+1 + · · ·+ an) =n∑

i=1

a3i (1− ai)

olacaktır. f(x) = x3(1−x) = x3−x4 fonksiyonunun [0, 1/2] aralıgında konveksoldugu acıktır. Oyle ki,

f ′′(x) = 6x− 12x2 = 6x(1− 2x) > 0

olacaktır. Esitsizligimiz ai’lere gore simetrik oldugundan, a1 ≥ a2 ≥ · · · ≥ analabiliriz.Buan gore, eger a1 ≤ 1/2 ise,

(1

2,1

2, 0, 0, · · · , 0) Â (a1, a2, · · · , an)

olacagından majorization esitsizliginden

f(a1) + f(a2) + · · ·+ f(an) ≤ f(1

2) + f(

1

2) + f(0) + · · ·+ f(0) =

1

8

olacaktır. Eger, a1 > 1/2 ise 1− a1, a2, · · · , an ∈ [0, 1/2) olacaktır.

(1− a1, 0, 0, · · · , 0) Â (a2, a3, · · · , an)oldugundan majorization esitsizligi ve n = 2 durumunu goz onunde bulundu-rursak,

f(a1) + f(a2) + · · ·+ f(an) ≤ f(a1) + f(1− a1) + f(0) + f(0) + · · ·+ f(0)

= f(a1) + f(1− a1) ≤ 1

8olacaktır.

Buna gore, esitlik durumu ancak ve ancak iki degisken esit ve geri kalan (n−2)degisken 0 ise vardır.

Page 59: MATEMATİK OLİMPİYATI ÇALIŞMA KİTAPÇIĞI · C»˜oz um.˜ Bu soruda »c˜oz ume daha kolay i»slemlerle ula»smak i»cin baz‡ de‚gi»s-˜ ken e‚gi»stirmeler yapmak yerinde

58 BOLUM 2. KONULAR

2.6.1 Calısma Soruları

1. Jensen Esitisizligini kullanarak aabbcc ≥ (abc)(a+b+c)/3 esitsizligini kanıtlayınız.(a, b, cpozitif reel sayılar.)

2. x1, · · · , xn ∈ [0, 1] ve a1, · · · , an > 0 olmak uzere, a1+ · · ·+an = 1 olarakveriliyor. Buna gore,

n∑

i=1

ai1 + xi

≤ 1

1 + xa11 · · ·xa11esitsizligini kanıttlayınız. Esitlik durumu hangi sartlar altında gercekle-sir?

3. Eger a, b, c, d > 0 ve c2 + d2 = (a2 + b2)3 ise

a3

c+

b3

d≥ 1

esitsizligini Holder Esitsizligi kullanarak kanıtlayınız.

4. P noktası ABC ucgeni icersinde

m(PAB) = m(PBC) = m(PCA) = α

esitligini saglayan bir nokta ise α acısının π/6 oldugunu kanıtlayınız.

5. Dar acılı bir ABC ucgeninin iki acısı π/6’dan kucuk veya esittir. Bunagore,

sinA

2sin

B

2sin

C

2≥ sin

π

4sin

π

6sin

π

12

esitsizligini kanıtlayınız.

6. x, y, z > 1, xyz = 4096 ve max(x, y, z) < 32 olarak veriliyor. Buna gore,x+ y + z toplamının en buyuk ve en kucuk degerlerini bulunuz.

7. Eger a, b ≥ 0 ise,

3

√a+ 3

√a+

3

√b+

3√b ≤ 3

√a+

3√b+

3

√b+ 3

√a

esitsizligini kanıtlayınız.

Page 60: MATEMATİK OLİMPİYATI ÇALIŞMA KİTAPÇIĞI · C»˜oz um.˜ Bu soruda »c˜oz ume daha kolay i»slemlerle ula»smak i»cin baz‡ de‚gi»s-˜ ken e‚gi»stirmeler yapmak yerinde

2.7. INDIRGEMELI DIZILER 59

2.7 Indırgemelı Dızıler

2.7.1 Birinci Dereceden Indirgemeler

Herhalde matematik olimpiyatları sınavlarına hazırlanıpta Fibonacci Sa-yıları’nı bilmeyen yoktur. Zaten Fibonacci sayılarıda f0 = 0, f1 = 1 olmakuzere

fn+1 = fn + fn−1, n ≥ 1

yinelemesi ile tanımlanır. Bir yinelemenin derecesi ise en buyuk ve en kucukalt terimlerin farkına esittir. Mesela,

un+2 − un+1 = 2

birinci derecedendir, veun+4 + 9u2n = n5

yinelemesi ise dorduncu derecedendir. Eger bir yinelemede tum ifadelerin ust-leri bir ise bu yinelemeye Dogrusal Yineleme denir. Mesela,

un+2 − un+1 = 2

bir dogrusal yinelemedir. Ancak

x2n + nxn−1 = 1 ve xn + 2xn−1 = 3

yinelemeleri lineer degillerdir. Eger bir yinelemenin tum terimlerinin kuvvetleriaynı kuvvettense, bu yinelemeye Homojen’dir denir. Mesela,

xm+3 + 8xm+2 − 9xm = 0

yinelemesi homojendir. Ancak,

xm+3 + 8xm+2 − 9xm = m2 − 3

yinelemesi homojen degildir. Bir yinelemin sadece indis degiskenine gore ta-nımlanan denkleme ise o yinelemenin Kapalı Form’u denir. Kapalı form sa-yesinde rahatlıkla yinelemenin her hangi bir terimini bulabiliriz. Biz genelmanada bu ders notunda ilk olarak,

xn = axn−1 + f(n), a 6= 1

Page 61: MATEMATİK OLİMPİYATI ÇALIŞMA KİTAPÇIĞI · C»˜oz um.˜ Bu soruda »c˜oz ume daha kolay i»slemlerle ula»smak i»cin baz‡ de‚gi»s-˜ ken e‚gi»stirmeler yapmak yerinde

60 BOLUM 2. KONULAR

formundaki (f burada bir polinomdur) yinelemelerin cozumleriyle ilgilenece-giz. Ilgilenirken de asagıda verdigimiz basamakları takip edecegiz.

1. Once xn = axn−1 formunda verilen yinelemenin indislerini uste alarakyani, karakteristik denklem olusturarak, xn = axn−1 denklemini elde ede-riz. Sadelestirmeler yaptıgımızda x = a olacaktır. Buna gore, xn = axn−1

homojen formundaki yinelememiz bize xn = Aan denklemini verecektir.Burada A bir sabit sayıdır.

2. Daha sonra bulunan xn = Aan + g(n) formu test edilir. Burada g, f ileaynı dereceden bir polinomdur.

Ornek 1. x0 = 7 ve xn = 2xn−1, n ≥ 1 ise xn kapalı formunu bulunuz.

Cozum. Alt indisleri us olarak yazarsak karakteristik denklemimizi xn =2xn−1 olacaktır. Sadelestirme yaparsak, x = 2 olacaktır. Buna gore bizim,xn = A2n formunda cozum yapmamız gerekmektedir. Burada, 7 = x0 = A20

ise A = 7 olacagından, kapalı form xn = 7(2)n olarak bulunur.

Ornek 2. x0 = 7 ve xn = 2xn−1 + 1, n ≥ 1 formunda verilen yinelemeninkapalı formunu bulunuz.

Cozum. Karakteristik denklemi yazıldıgında xn = 2xn−1 veya x = 2 eldeedilir. Buradan cozumlerden birinin xn = A(2)n oldugu acıktır. Ancak yinele-menin bir parcasıda f(n) = 1 polinomu olduguna gore, kapalı denklemin xn =A2n+B formunda olması gerekmektedir. Buradan, 7 = x0 = A20+B = A+Bve 15 = x1 = 2A + B olduguna gore bu iki denklemin cozumunden A = 8 veB = −1 olacaktır. Oyleyse, soruda istenen kapalı form,

xn = 8(2n)− 1 = 2n+3 − 1

olacaktır.

Not. Ornek 2.’nin cozumunde dikkat edilirse olusturula karakteristik denklemiki parcadan olusuyor. Bunlardan ilki zaten alt indislerin us olarak yazılmasıylaelde edilirken, ikincisi bir polinom ve bu polinomun derecesi yineleme icersindeki polinomun derecesi ile aynı. Bundan sonraki cozumlerde de polinom secimibenzer sekilde olacaktır.

Ornek 3. x0 = 2 ve xn = 9xn−1 − 56n+ 63 ise xn kapalı formunu bulunuz.

Page 62: MATEMATİK OLİMPİYATI ÇALIŞMA KİTAPÇIĞI · C»˜oz um.˜ Bu soruda »c˜oz ume daha kolay i»slemlerle ula»smak i»cin baz‡ de‚gi»s-˜ ken e‚gi»stirmeler yapmak yerinde

2.7. INDIRGEMELI DIZILER 61

Cozum. Karakterisitk denklem yazıldıgında karakterisitik denklemimiz xn =9xn−1 veya x = 9 olacaktır. Buna gore, kapalı formun bir kısmı xn = A(9)n

formunda olacaktır. Ancak, soruda verilen yinelemenin ikinci kısmını f(n) =−56n+ 63 polinomu olusturdugundan, cozum olarak kullanacagımız polinomg(n) = Bn+ C olacaktır. Buna gore, yinelememizin kapalı formu

xn = A9n +Bn+ C

olacaktır. x0, x1, x2 icin cozumlere bakıldıgında,

2 = A+ C

25 = 9A+B + C

176 = 81A+ 2B + C

esitlikleri icin A = 2, B = 7 ve C = 0 olacaktır. Buna gore soruda istenenkapalı form, yada genel cozum

xn = 2(9n) + 7n

olarak bulunur.

Ornek 4. x0 = 1 ve xn = 3xn−1−2n2+6n−3 ise xn kapalı formunu bulunuz.

Cozum.Yinelemenin karakterisitk denklemini yazdıktan sonra kolaylıkla xn =A(3)n indirgemesini elde edebiliriz. Ancak yinelememizin bir parcasıda f(n) =−2n2 + 6n − 3 seklinde ki ikinci dereceen bir polinom oldugundan ozel cozu-mumuzde kullanacagımız g(n) polinomu Bn2 +Cn+D olmalıdır. Buna gore,yinelememizin indirgenmis hali,

xn = A3n +Bn2 + Cn+D

sekline olacaktır. Eger bilinen xi, i = 0, 1, 2, 3 icin katsayaları bulmaya calısır-sak,

1 = A+D,

4 = 3A+B + C +D,

13 = 9A+ 4B + 2C +D,

36 = 27A+ 9B + 3C +D

Page 63: MATEMATİK OLİMPİYATI ÇALIŞMA KİTAPÇIĞI · C»˜oz um.˜ Bu soruda »c˜oz ume daha kolay i»slemlerle ula»smak i»cin baz‡ de‚gi»s-˜ ken e‚gi»stirmeler yapmak yerinde

62 BOLUM 2. KONULAR

denklemlerini elde ederiz. Buradan da, A = B = 1, C = D = 0 olacagından,istenen kapalı form

xn = 3n + n2

olacaktır.

Ornek 5. x0 = 2 ve xn = 2xn−1 + 3n−1 ise kapalı formu bulunuz.

Cozum. Eger gerekli islemleri yaparsak, genel formun

xn = A2n +B3n

denklemi elde edilir. Burada, x0 = 2 ve x1 = 2(2) + 30 = 5 denklemlerinden,

2 = A+B

7 = 2A+ 3B

esitliklerini elde edilir. Buna gore, A = 1 ve B = 1 olacagından istenen kapalıform

xn = 2n + 3n

olacaktır.

Ornek 6. x0 = 7 ve xn = xn−1 + n, n ≥ 1 ise xn icin kapalı formu bulunuz.

Cozum. Simdi bu cozumu siz yapmaya calısın. Kapalı formu,

xn = 7 +n(n+ 1)

2

olarak bulmanız gerekmektedir.

Simdiye kadar cozduklerimizde genel olarak, lineer yinelemeler hakimdi. Sim-diki ornegimizde de lineer olmayan ancak lineerlestirilebilir, birinci drecen yi-nelemeler den birini cozecegiz.

Ornek 6. u0 = 3 ve u2n+1 = un, n ≥ 1 ise yinelemin kapalı formunu bulunuz.

Cozum. Varsayalım, vn = log un olsun. Buna gore,

vn = log un = log u1/2n−1 =

1

2log un−1 =

vn − 1

2

Page 64: MATEMATİK OLİMPİYATI ÇALIŞMA KİTAPÇIĞI · C»˜oz um.˜ Bu soruda »c˜oz ume daha kolay i»slemlerle ula»smak i»cin baz‡ de‚gi»s-˜ ken e‚gi»stirmeler yapmak yerinde

2.7. INDIRGEMELI DIZILER 63

olacaktır. Burada,

vn =vn−1

2

oldugundan,

vn =v02n

olacaktır. Buradan,

log un =log u02n

oldugundan istenilen kapalı form,

un = 31/2n

olarak bulunur.

2.7.2 Ikinci Dereceden Indirgemeler

Bir evvelki konumuzda birici dereceden yinelemeleri ele almıstık. Oyleki,her bir terim kendisinden bir onceki terime bagımlı olarak veriliyordu. Simdiverecegimiz formda ise durum artık biraz daha farklı. Oyleki artık karsılaca-gımız yinelemeler

xn = axn−1 + bxn−2

seklinde olacaktır. Bu tur yinelemelerin cozumleri icinde takip etmemiz gereknbazı cozum basamakları vardır. Buna gore,

1. Once alt indisleri us olarak alıp karakteristik denklemi xn = axn−1 +bxn−2 oldugundan kokleri r1 ve r2 olan x2 − ax− b = 0 olarak bulunur.

2. Eger kokler birbirinden farklı ise genel form

xn = A(r1)n +B(r2)

n

seklinde olacaktır.

3. Eger kokler aynı ise genel form

xn = A(r1)n +Bn(r1)

n

seklinde olacaktır.

Page 65: MATEMATİK OLİMPİYATI ÇALIŞMA KİTAPÇIĞI · C»˜oz um.˜ Bu soruda »c˜oz ume daha kolay i»slemlerle ula»smak i»cin baz‡ de‚gi»s-˜ ken e‚gi»stirmeler yapmak yerinde

64 BOLUM 2. KONULAR

Ornek 7. x0 = 1, x1 = −1 ve xn+2 + 5xn+1 + 6xn = 0 yinelemesinin kapalıformunu bulunuz.

Cozum. Soruda verilen yinelemenin karakteristik denklemi

x2 + 5x+ 6 = (x+ 3)(x+ 2) = 0

olarak elde edilir.Buna gore kapalı formumuz

xn = A(−2)n +B(−3)n

olacaktır. Buradan da, A = 2 ve B = −1 olacagından soruda istenilen kapalıform,

xn = 2(−2)n − (−3)n

olacaktır.

Ornek 8. Fibonacci yinelemesi icin kapalı formu, f0 = 0, f1 = 1 ve fn =fn−1 + fn−2 bilgilerini kullanarak bulunuz.

Cozum. Karakterisitk denklemimiz f2 − f − 1 = 0 olacagından kapalı formu-muz

fn = A

(1 +

√5

2

)n

+B

(1−√

5

2

)n

seklinde olacaktır. Baslangıc degerleri kullanıldıgında,

0 = A+B

1 = A

(1 +

√5

2

)n

+B

(1−√

5

2

)n

=1

2(A+B) +

√5

2(A−B) =

√5

2(A−B)

olacagından

A =1√5, B = − 1√

5

olacaktır. Sonuc olarakta Cauchy-Binet Formulu olarakta bilinen

fn =1√5

(1 +

√5

2

)n

+1√5

(1−√

5

2

)n

kapalı form bulunacaktır.

Page 66: MATEMATİK OLİMPİYATI ÇALIŞMA KİTAPÇIĞI · C»˜oz um.˜ Bu soruda »c˜oz ume daha kolay i»slemlerle ula»smak i»cin baz‡ de‚gi»s-˜ ken e‚gi»stirmeler yapmak yerinde

2.7. INDIRGEMELI DIZILER 65

Ornek 8.(Tubitak Deformesi10) x0 = 1, x1 = 4, xn = 4xn−1 − 4xn−2 isekapalı formu bulunuz.

Cozum. Karakteristik denklemimiz x2 − 4x + 4 = (x − 2)2 = 0 olacaktır.Burada koklerin birbirine esit oldugu acıktır. Buna gore kapalı formumuz,

xn = A2n +Bn2n

formunda olacaktır. Eger baslangıc degerlerini kullanırsak,

1 = A

4 = 2A+ 2B

A = 1 ve B = 1 olarak bulunur. Buna gore, istenilen kapalı form

xn = 2n + n2n

olacaktır.

2.7.3 Alıstırmalar

Asagıdaki sorulardan (1− 5) icin kapalı formları bulunuz.

1. x0 = 3, xn = xn−1+43

2. x0 = 1, xn = 5xn−1 − 20n+ 25

3. x0 = 1, xn = xn−1 + 12n

4. x0 = 5, xn = 2xn−1 + 9(5n−1)

5. a0 = 5, aj+1 = a2j + 2aj , j ≥ 0

6. (AIME, 1984) x19 = 94 ve

xn + xn−1 = n2, n ≥ 1

ise x94’un 1000 ile bolumunden kalan kactır?

Asagıdaki (6 − 10) icin ikinci dereceden yinelemelerin kapalı formlarınıbulunuz.

10Benzer bir soru TUBITAK matematik olimpiyatlarında da sorulmustur.

Page 67: MATEMATİK OLİMPİYATI ÇALIŞMA KİTAPÇIĞI · C»˜oz um.˜ Bu soruda »c˜oz ume daha kolay i»slemlerle ula»smak i»cin baz‡ de‚gi»s-˜ ken e‚gi»stirmeler yapmak yerinde

66 BOLUM 2. KONULAR

7. x0 = 0, x1 = 1, xn = 10xn−1 − 21xn−2

8. x0 = 0, x1 = 1, xn = 10xn−1 − 25xn−2

9. x0 = 0, x1 = 1, xn = 10xn−1 − 21xn−2 + n

10. x0 = 0, x1 = 1, xn = 10xn−1 − 21xn−2 + 2n

11. Bir duzlem uzerine cizilen n cember duzlemi parcalara ayırmaktadır.Buna gore, duzlem uzerindeki n cemberin ayırdıgı parcaların sayısınıveren denklemi bulunuz.

12. Bir duzlem uzerine cizilen n dogrunun duzlem uzerinde ayırdıgı parcala-rın sayısını veren denklemi bulunuz.

2.8 Yenıden Duzenleme [Rearrangement] Esıtsızlıgı

Varsayalım elimizde iki reel uclu (a1, a2, a3) ve (b1, b2, b3) olsun. Eger ikinciuclunun tum permutasyonlarını dusunursek elimizde 3! = 6 tane uclunun ola-cagı acıktır. Biz bu uclulerin kumesine P ve (x1, x2, x3) ∈ P olsun diyelim.Buna gore varsayalım,

S = a1x1 + a2x2 + a3x3

toplamıda elimizde bulunsun. Burada bizi ilgilendiren asıl soru S toplamınınne zaman en buyuk, ne zaman en kucuk oldugudur. Bu soruya cevap aramayabaslamadan evvel ileriki basamaklarda kullanacagımız bazı terimleri acıklaya-lım.Tanım. Varsayalım elimizde (a1, a2, a3) ve (b1, b2, b3) reel ucluleri olsun. Bunagore,

• Eger iki uclunun elemanlarıda artan veya iki uclunun elemanlarıda aza-lan bir sırada yazılmıssa bu ikiliye Benzer Duzenli diyelim. Yani a1 ≤a2 ≤ a3 ve b1 ≤ b2 ≤ b3 veya a1 ≥ a2 ≥ a3 ve b1 ≥ b2 ≥ b3 durumlarısaglansın.

• Eger iki ucluden biri artan digeri azalan sırada yazılmıssa bu ucluyedeAykırı Duzenli diyelim.

Page 68: MATEMATİK OLİMPİYATI ÇALIŞMA KİTAPÇIĞI · C»˜oz um.˜ Bu soruda »c˜oz ume daha kolay i»slemlerle ula»smak i»cin baz‡ de‚gi»s-˜ ken e‚gi»stirmeler yapmak yerinde

2.8. YENIDEN DUZENLEME [REARRANGEMENT] ESITSIZLIGI 67

Ornek.

a. (−1, 1, 3) ve (2, 5, 7) benzer duzenlidirler.

b. Eger 0 < a ≤ b ≤ c ise (a, b, c) ve ( 1a ,1b ,

1c ) aykırı duzenlidir. Ama, (a, b, c)

ve ( 1b+c ,

1a+c ,

1a+b) benzer duzenlidirler.

c. Eger 0 < a ≤ b ≤ c ve m ∈ R+ ise (a, b, c) ve (am, bm, cm) benzerduzenliyken, (a, b, c) ve ( 1

am , 1bm , 1

cm ) aykırı duzenlidir.

d. Eger a ≤ b ≤ c ve n bir tek tamsayı ise (a, b, c) ve (an, bn, cn) benzerduzenlidir.

Artık esitsizligimizi daha yakından tanıma zamanı geldi. Teorem [Re-arragement Inequality]. (a1, a2, a3) ve (b1, b2, b3) iki reel uclu olmak uzere(x1, x2, x3) uclusu (b1, b2, b3) uclusunun bir permitasyonu olsun. Buna gore,

• Eger (a1, a2, a3) ve (b1, b2, b3) benzer duzenli ise

a1b1 + a2b2 + a3b3 ≥ a1x1 + a2x2 + a3x3

olacaktır.

• Eger (a1, a2, a3) ve (b1, b2, b3) aykırı duzenli ise

a1b1 + a2b2 + a3b3 ≤ a1x1 + a2x2 + a3x3

olacaktır.

Kanıt. Varsayalım elimizde (a1, a2, a3) ve (b1, b2, b3) artan sırayla dizilmis ikiuclu olsun. (x1, x2, x3) uclusude (b1, b2, b3) uclusunun bir permutasyonu olsunve x1 ≥ x2 olsun. S ve S′ toplamlarınıda

S = a1x1 + a2x2 + a3x3

veS′ = a1x2 + a2x1 + a3x3

olarak alalım. Burada S′ toplamı, S toplamındaki x1 ve x2’nin yer degistirme-siyle elde edildigi acıktır. Eger bu iki toplamı farkını alırsak,

S′ − S = (x1 − x2︸ ︷︷ ︸+

)(a2 − a1︸ ︷︷ ︸+

) ≥ 0

Page 69: MATEMATİK OLİMPİYATI ÇALIŞMA KİTAPÇIĞI · C»˜oz um.˜ Bu soruda »c˜oz ume daha kolay i»slemlerle ula»smak i»cin baz‡ de‚gi»s-˜ ken e‚gi»stirmeler yapmak yerinde

68 BOLUM 2. KONULAR

olacaktır. Demekki S′ ≥ S’dir.

Buna gore, x1 ve x2’nin yer degistirmesi sadece S toplamının degerini ar-tırmaktadır. Oyleyse, eger tum (xi, xj) ikililerinin (xi ≥ xj , i < j) yerleridegistirilirse toplam ancak en buyuk olabilir. En buyuk olacak toplamda za-ten a1b1 + a2b2 + a3b3 olacaktır. Benzer bicimde eger (a1, a2, a3) ve (b1, b2, b3)uclulerinin ikiside azalan birer uclu olarak secilseydi, ispatın ikinci kısmıdaburadan yapılabilirdi.

Simdi, bu yeni esitsizligimizi birkac ornek uzerinde uygulayalım.

Ornek. a, b, c ∈ R olmak uzere

i. a2 + b2 + c2 ≥ ab+ bc+ ca

ii an + bn + cn ≥ an−1b+ bn−1c+ cn−1a

esitsizliklerini gosteriniz.

Cozum. Sorunun birinci sıkkı zaten ikinci sıkkın ozel bir durumu oldugun-dan sadece ikinci sıkkı cozmemiz yeterli olacaktır. Buna gore eger uclulerimizi(a, b, c) ve (an−1, bn−1, cn−1) benzer duzenlileri olarak belirlersek, istenen esit-sizlik yeniden duzenleme esitsizligi ile,

aan−1 + bbn−1 + ccn−1 ≥ abn−1 + bcn−1 + can−1

olarak bulunur.

Ornek. a, b, c > 0 oduguna gore, asagıdaki esitsizlikleri kanıtlayınız.

i. a+b+cabc ≤ 1

a2+ 1

b2+ 1

c2

ii. a2

b2+ b2

c2+ c2

a2≥ b

a + cb +

ac

iii. a2

b + b2

c + c2

a ≥ a+ b+ c

Cozum.

• a ≤ b ≤ c olarak kabul edelim ve ( 1a ,1b ,

1c ) ve (

1a ,

1b ,

1c ) benzer duzenlilerini

secelim. Buna gore,

1

a

1

a+

1

b

1

b+

1

c

1

c≥ 1

a

1

b+

1

b

1

c+

1

c

1

a1

a2+

1

b2+

1

c2≥ 1

ab+

1

bc+

1

ac

Page 70: MATEMATİK OLİMPİYATI ÇALIŞMA KİTAPÇIĞI · C»˜oz um.˜ Bu soruda »c˜oz ume daha kolay i»slemlerle ula»smak i»cin baz‡ de‚gi»s-˜ ken e‚gi»stirmeler yapmak yerinde

2.8. YENIDEN DUZENLEME [REARRANGEMENT] ESITSIZLIGI 69

olacaktır.

• Eger (ab ,bc ,

ca) ve (ab ,

bc ,

ca) benzer sıralılarını alırsak,

a

b

a

b+

b

c

b

c+

c

a

c

a≥ a

b

b

c+

b

c

c

a+

c

a

a

b

elde edilecektir.

• Bu seferde sıralı ikililerimizi aykırı duzenliler arasından secelim. Yani,(a2, b2, c2) ve ( 1a ,

1b ,

1c ) kullanacagımız uclulerimiz olsun. Buna gore,

a21

a+ b2

1

b+ c2

1

c≤ a2

1

b+ b2

1

c+ c2

1

a

elde edilecektir.

Ornek. [1963, Moskova Matematik Olimpiyatı] a, b, c > 0 oldugunagore,

a

b+ c+

b

a+ c+

c

a+ b≥ 3

2

esitsizligini kanıtlayınız.

Cozum. Varsayalım a ≤ b ≤ c olsun ve benzer sıralılarımız da (a, b, c) ve

(1

a+ b,

1

a+ c,

1

a+ b) (2.1)

olsun. Buna gore,

a1

b+ c+ b

1

c+ a+ c

1

a+ b≥ a

c+ a+

b

a+ b+

c

b+ c(2.2)

a1

b+ c+ b

1

a+ c+ c

1

a+ b≥ a

a+ b+

b

b+ c+

c

a+ c(2.3)

esitsizliklerini elde edebiliriz. Eger [1] ve [2.3] esitsizlerini altalta toplarsak,

2(a

b+ c+

b

a+ c+

c

a+ b) ≥ a

a+ c+

b

a+ b+

c

b+ c+

a

a+ b+

b

b+ c+

c

a+ c= 3

esitsizligindende soruda istenen [2.1] esitsizligini elde ederiz.

Page 71: MATEMATİK OLİMPİYATI ÇALIŞMA KİTAPÇIĞI · C»˜oz um.˜ Bu soruda »c˜oz ume daha kolay i»slemlerle ula»smak i»cin baz‡ de‚gi»s-˜ ken e‚gi»stirmeler yapmak yerinde

70 BOLUM 2. KONULAR

Ornek. [Chebyshev Esitsizligi] Eger (a1, a2, a3) ve (b1, b2, b3) benzerduzenliler ise

a1b1 + a2b2 + a3b33

≥ (a1 + a2 + a3

3)(b1 + b2 + b3

3) (2.4)

esitsizligini kanıtlayınız.

Cozum. Soruda zaten benzer duzenli ucluler verildigine gore, yeniden du-zenleme metodunu kullanalım

a1b1 + a2b2 + a3b3 = a1b1 + a2b2 + a3b3 (2.5)

a1b1 + a2b2 + a3b3 ≥ a1b2 + a2b3 + a3b1 (2.6)

a1b1 + a2b2 + a3b3 ≥ a1b3 + a2b1 + a3b2 (2.7)

esitsizliklerini elde ederiz. Eger [2.5], [2.6] ve [2.7] esitsizliklerini altalta toplar-sak,

3(a1b1 + a2b2 + a3b3) ≥ a1(b1 + b2 + b3) + a2(b1 + b2 + b3) + a3(b1 + b2 + b3))

esitsizligini ve sonuc olarakta soruda verilen [2.4] esitsizligini elde ederiz.

Not. Benzer bicimde, (a1, a2, a3) ve (b1, b2, b3) aykırı duzenlileri icinde,

a1b1 + a2b2 + a3b33

≤ (a1 + a2 + a3

3)(b1 + b2 + b3

3) (2.8)

esitsizligi elde edilebilir.

Ornek. [Aritmetik Orta - Karesel Orta Esitsizligi] a1, a2, a3 reelsayılar olmak uzere verilen

a1 + a2 + a33

≤√

a21 + a22 + a233

(2.9)

esitsizligini kanıtlayınız.

Cozum. Sorunun cozumu icin Ornek 0.7’de kanıtladıgımız esitsizligi kul-lanmamız kafidir. Buna gore, (a1, a2, a3) ve (a1, a2, a3) benzer duzenlileri icin,

a21 + a22 + a233

≤ (a1 + a2 + a3

3)(a1 + a2 + a3

3) = (

a1 + a2 + a33

)2 (2.10)

Page 72: MATEMATİK OLİMPİYATI ÇALIŞMA KİTAPÇIĞI · C»˜oz um.˜ Bu soruda »c˜oz ume daha kolay i»slemlerle ula»smak i»cin baz‡ de‚gi»s-˜ ken e‚gi»stirmeler yapmak yerinde

2.8. YENIDEN DUZENLEME [REARRANGEMENT] ESITSIZLIGI 71

olduguna gore, soruda istenen [2.9] esitsizligi kanıtlanmıs olur.

Ornek. [Aritmetik Orta-Geometrik Orta Esitsizligi]a1, a2, a3 pozitif sa-yılar olmak uzere verilen

a1 + a2 + a33

≥ 3√a1a2a3 (2.11)

esitsizligini kanıtlayınız.

Cozum. P = 3√a1a2a3 olmak uzere, sorua kullanacagımız aykırı duzenli-

lerimizi,

x1 =a1P, x2 =

a1a2P 2

, x3 =a1a2a3P 3

= 1 ve (2.12)

y1 =1

x1, y2 =

1

x2, y3 =

1

x3= 1 (2.13)

olarak secelim. Burada (x1, x2, x3) artan sıralı ise (y1, y2, y3) azalan sıralı ola-caktır. Buna gore,

x1y1 + x2y2 + x3y3 ≤ x1y3 + x2y1 + x3y2

1 + 1 + 1 ≤ a1P

+a2P

+a3P

=a1 + a2 + a3

P

olacaktır. Buradan da,

3P ≤ a1 + a2 + a3

P ≤ a1 + a2 + a33

olacagından, istenen kanıt tamamlanmıs olur.

Ornek. Chebyshev esitsizligini iki reel sayı icin gosteriniz.

Cozum. Varsayalım benzer duzenli ikililerimiz (a1, a2) ve (b1, b2) olsun.Buna gore,

a1b1 + a2b2 = a1b1 + a2b2

a1b1 + a2b2 ≥ a1b2 + a2b1

Page 73: MATEMATİK OLİMPİYATI ÇALIŞMA KİTAPÇIĞI · C»˜oz um.˜ Bu soruda »c˜oz ume daha kolay i»slemlerle ula»smak i»cin baz‡ de‚gi»s-˜ ken e‚gi»stirmeler yapmak yerinde

72 BOLUM 2. KONULAR

esitsizlikleri taraf tarafa toplanırsa,

2(a1b1 + a2b2) ≥ (a1 + a2)(b1 + b2)

a1b1 + a2b22

≥ (a1 + a2

2)(b1 + b2

2)

olacaktır.

Ornek.an + bn

a+ b≥ 1

2(an−1 + bn−1)

esitsizligini kanıtlayınız.

Cozum. Ornek 0.11’de kanıtladıgımız esitsizligi kullanalım. Buna goreikililerimiz (a, b) ve (an−1, bn−1) ikilileri olsun. Buna gore,

aan−1 + bbn−1

2≥ (

a+ b

2)(an−1 + bn−1

2)

an + bn

a+ b≥ 1

2(an−1 + bn−1)

Ornek. a, b ≥ 0 olmak uzere, asagıdaki esitsizlikleri kanıtlayınız.

a. 2(a5 + b5) ≥ (a3 + b3)(a2 + b2)

b. a9 + b9 ≥ a2b2(a5 + b5)

c. (a+ b)n ≤ 2n−1(an + bn)

Cozum.

a. (a2, b2), (a3, b3) benzer duzenlileri olsun. Buna gore,

a2a3 + b2b3

2≥ (

a2 + b2

2)(a3 + b3

2) ⇒ 2(a5 + b5) ≥ (a2 + b2)(a3 + b3)

olarak bulunur.

Page 74: MATEMATİK OLİMPİYATI ÇALIŞMA KİTAPÇIĞI · C»˜oz um.˜ Bu soruda »c˜oz ume daha kolay i»slemlerle ula»smak i»cin baz‡ de‚gi»s-˜ ken e‚gi»stirmeler yapmak yerinde

2.8. YENIDEN DUZENLEME [REARRANGEMENT] ESITSIZLIGI 73

b. Bu sıktada benzer duzenlilerimizi (a4, b4) ve (a5, b5) olarak secelim. Bunagore,

a4a5 + b4b5

2≥ (

a4 + b4

2)(a5 + b5

2) ≥

√a4b4(

a5 + b5

2) = a2b2(

a5 + b5

2)

olacaktır. Bu esitsizliktende,a9 + b9 ≥ a2b2(a5 + b5) esitsizligi elde edilir.

c. Soruyu Chebyshev Esitsizligi’ni kullanarak cozmeye calısalım. Ikilileri-miz (an−1, bn−1) ve (a, b) olsun. Buna gore,

an + bn ≥ 1

2(an−1 + bn−1)(a+ b) =

1

2(an−2a+ bn−2b)(a+ b)

an + bn ≥ 1

22(an−2 + bn−2)(a+ b)(a+ b)

an + bn ≥ 1

2n−1(a+ b)(a+ b) · · · (a+ b) =

1

2n−1(a+ b)n olacaktır.

Buna gore istenen esitsizlik,

2n−1(an + bn) ≥ (a+ b)n

elde edilmis olur.

Ornek. a, b, c > 0 ve n ∈ Z+ ise

an

b+ c+

bn

a+ c+

cn

a+ b≥ an−1 + bn−1 + cn−1

2

esitsizligini kanıtlayınız.

Cozum. a ≤ b ≤ c olmak uzere benzer duzenlilerimiz (an, bn, cn) ve( 1b+c ,

1a+c ,

1a+b) olarak secelim. Buna gore,

an

b+ c+

bn

a+ c+

cn

a+ b≥ an

a+ c+

bn

a+ b+

cn

b+ c(2.14)

an

b+ c+

bn

a+ c+

cn

a+ b≥ an

b+ a+

bn

b+ c+

cn

a+ c(2.15)

esitsizliklerini elde ederiz. Eger bu iki esitsizligi taraftarafa toplarsak,

2(an

b+ c+

bn

a+ c+

cn

a+ b) ≥ an + bn

a+ b+

bn + cn

b+ c+

an + cn

a+ c

Page 75: MATEMATİK OLİMPİYATI ÇALIŞMA KİTAPÇIĞI · C»˜oz um.˜ Bu soruda »c˜oz ume daha kolay i»slemlerle ula»smak i»cin baz‡ de‚gi»s-˜ ken e‚gi»stirmeler yapmak yerinde

74 BOLUM 2. KONULAR

esitsizligini elde ederiz. Bundan sonraki basamaktada Ornek 0.12 ’de kanıt-ladıgımız esitsizligi kullanırsak,

2(an

b+ c+

bn

a+ c+

cn

a+ b) ≥ an + bn

a+ b+

bn + cn

b+ c+

an + cn

a+ c

≥ 1

2(an−1 + bn−1) +

1

2(bn−1 + cn−1) +

1

2(an−1 + cn−1)

= an−1 + bn−1 + cn−1

esitsizliginden, soruda bizden istenen esitsizlige ulasılır.

Ornek. a, b, c > 0 olmak uzere,

aabbcc ≥ (abc)a+b+c

3

esitsizligini kanıtlayınız.

Cozum. a ≤ b ≤ c olarak alalım. Uclulerimizide, biraz sıradısı bir se-cim yaparak, (a, b, c) ve (log a, log b, log c) olarak alalım. Buna gore,Chebyshevesitsizliginden

a log a+ b log b+ c log c

3≥ (

a+ b+ c

3)(log a+ log b+ log c

3)

esitsizligi elde edilir. Buna gore,

log aabbcc

3≥ (

a+ b+ c

3)(log abc

3)

olacagından istenen esitsizlik,

log aabbcc ≥ log(abc)a+b+c

3 ⇒ aabbcc ≥ (abc)a+b+c

3

olarak bulunur.

Ornek. A,B,C bir ucgenin acıları olmak uzere (radyan cinsinden), a, b, cuzunukları da bu ucgenin kenar uzunluklarıdır. p = 1

2(a+b+c) olduguna gore,

A

p−A+

B

p− b+

C

p− c≥ 3π

p(2.16)

Page 76: MATEMATİK OLİMPİYATI ÇALIŞMA KİTAPÇIĞI · C»˜oz um.˜ Bu soruda »c˜oz ume daha kolay i»slemlerle ula»smak i»cin baz‡ de‚gi»s-˜ ken e‚gi»stirmeler yapmak yerinde

2.9. TRIGONOMETRIK DEGISKEN DEGISTIRME 75

esitsizligini kanıtlayınız.

Cozum. Varsayalım A ≤ B ≤ C olsun. Buna gore benzer duzenli uclule-rimizi (A,B,C) ve ( 1

p−a ,1

p−b ,1

p−c) secebiliriz. Chebyshev esitsizliginden,

1

3(

A

p−A+

B

p− b+

C

p− c) ≥ (

A+B + C

3)(

1

p− a+

1

p− b+

1

p− c)1

3

3(

1

p− a+

1

p− b+

1

p− c)1

3

≥ π

9· 9p

esitsizligindende [2.16] esitsizligi kolaylıkla cıkarılır.

Ornek. [1995,IM0] a, b, c ∈ R+ ve a · b · c = 1 olduguna gore,

1

a3(b+ c)+

1

b3(a+ c)+

1

c3(a+ b)≥ 3

2(2.17)

esitsiligini kanıtlayınız.

Cozum. Varsayalım x = 1a , y = 1

b , z = 1c olsun. a · b · c = 1 oldugundan,

x · y · z = 1 olacagı acıktır. Buna gore, yeni degiskenlere gore esitizligimizduzenlersek,

x2

y + z+

y2

x+ z+

z2

y + x≥ 3

2

esitsizligini elde ederiz. Cozumun bundan sonraki basamakları sadece Ornek0.14’deki esitsizligin kanıtının kullanılmasıyla olacaktır. Buna gore,

x2

y + z+

y2

x+ z+

z2

y + x≥ x+ y + z

2≥ 3

23√xyz =

3

2

olacaktır.

2.9 Trıgonometrık Degısken Degıstırme

Trigonometrik ozdesliklerin coklugu, bu denklemleri kullanarak cebirseldenklemlere oldukca kolay cozumler gelistirmemize yardımcı olur. Degisken de-gistirmede, hangi ozdesligi kullanacagımız aslında cogu zaman verilen sorunun

Page 77: MATEMATİK OLİMPİYATI ÇALIŞMA KİTAPÇIĞI · C»˜oz um.˜ Bu soruda »c˜oz ume daha kolay i»slemlerle ula»smak i»cin baz‡ de‚gi»s-˜ ken e‚gi»stirmeler yapmak yerinde

76 BOLUM 2. KONULAR

icine gomuludur. Bu sebebten sorulara cozmeye baslamadan once denkleminasıl yeniden duzenleyecegimizin kararına varmamız yerinde olur.

Ornek. Asagıdaki denklem sisteminin tum reel cozumlerini bulunuz.

x3 − 3x = y

y3 − 3y = z

z3 − 3z = x

Cozum. Soruda verilen x3 − 3x formu kosinus uc kata acı ozdesliginianımsatıyor. Ancak tabiki x3 degiskeninin onundeki katsayı yok. Bu yuzdendegisken yerine cift kat acı kosinusu yerine, sadece cosx degiskeni almamızyerinde olacaktır. Buna gore x = 2 cosu, y = 2 cos v, z = 2 cosw u, v, w ∈ [0, π]ise

2 cos 3u = 2 cos v

2 cos 3v = 2 cosw

2 cos 3w = 2 cosu

sistemini elde ederiz. Kosinus uc kat acı formulunu cos 3u ve cos v icin kul-lanırsak cos 9u = cos 3v olacaktır. Bu esitligi ikinci denklem de kullanırsakcos 9u = cosw olur. Aynen daha onceki gibi cos 27u = cos 3w ise ucuncu denk-lem cos 27u = cosu olacaktır.Demek ki, u = kπ/14, k = 0, 1, 2, · · · , 14 veu = kπ/13, k = 1, 2, · · · , 12 olacagından

x = 2 cos kπ/14, y = 2 cos 3kπ/14, z = 2 cos 9kπ/14, k = 0, 1, · · · , 14ve

x = 2 cos kπ/13, y = 2 cos 3kπ/13, z = 2 cos 9kπ/13, k = 1, 2, · · · , 12olacagından 3× 3× 3 = 27 tane cozum elde edilir.Simdide tanjant fonksiyonunu kullanarak sıradaki ornegi cozmeye calısalım.

Ornek. {xn}∞n=1 olmak uzere,

xn+1 =

√3xn − 1

xn +√3, n > 1

Page 78: MATEMATİK OLİMPİYATI ÇALIŞMA KİTAPÇIĞI · C»˜oz um.˜ Bu soruda »c˜oz ume daha kolay i»slemlerle ula»smak i»cin baz‡ de‚gi»s-˜ ken e‚gi»stirmeler yapmak yerinde

2.9. TRIGONOMETRIK DEGISKEN DEGISTIRME 77

Ise , bu dizinin periyodik oldugunu kanıtlayınız.

Cozum. Tanjant fark formulunu hatırlarsak

tan (a− b) =tan a− tan b

1− tan a · tan bolacaktır. tan (π/6) = 1/

√3 oldugunuda biliyoruz. Buna gore,

xn+1 =xn − 1/

√3

1 + xn1/√3

olarak yazılabilir. Eger x1 = tan t alırsak x2 = tan (t− π/6) olacagından,

xn = tan (t− (n− 1)π/6) , n > 1

olur. Tanjant fonksiyonuda periyodik olduguna gore, demek ki, xn = xn+6

olacagından soruda verilen dizimiz periyodiktir ve periyodu 6 dır.

Ornek. a, b, c, x, y, z > 0 ise(a2 + x2

) (b2 + y2

) (c2 + z2

)> (ayz + bzx+ cxy − xyz)2

esitsizligini kanıtlayınız.

Cozum. a = x tanα, b = y tanβ, c = z tan γ α, β, γ ∈ [0, π/2] olarakalalım. Buna gore,

1 ≥ cos2(x+ y + z) = (sinx · sin (y + z)− cosx · cos (y + z))2

= (sinx · sin y · sin z + sinx · cos y · sin z + cosx · sin y · sin z − cosx · cos y · cos z)2

esitsizliginin iki tarafınıda

cos2 α cos2 β cos2 γ

ile bolersek,

sec2 α sec2 β sec2 γ > (tanα+ tanβ + tan γ − 1)2

olacagından,(x2 tan2 α+ x2

) (y2 tan2 β + z2

) (z2 tan2 γ + z2

)> x2y2z2 (tanα+ tanβ + tan γ − 1)2

olacaktır. Buda zaten soruda istenen esitsizligi dogrular.

Page 79: MATEMATİK OLİMPİYATI ÇALIŞMA KİTAPÇIĞI · C»˜oz um.˜ Bu soruda »c˜oz ume daha kolay i»slemlerle ula»smak i»cin baz‡ de‚gi»s-˜ ken e‚gi»stirmeler yapmak yerinde

78 BOLUM 2. KONULAR

2.9.1 Calısma Soruları

1. a reel parametresinin hangi degerleri icin√

1− x2 > a− x

esitsizligini saglayan bir x reel degeri vardır?

2. (0, 1) aralıgından dort farklı sayı alınıyor. Buna gore, bu sayılardan ikitanesi olan xve y’nin

0 < x√1− y2 − y

√1− x2 <

1

2

esitligini saglayacagını kanıtlayınız.

3. Secilen 4 farklı reel sayı arasından

1 + ab√1 + a2

√1 + b2

>1

2

esitsizligini saglayan bir (a, b) ikilisinin bulunabilecegini kanıtlayınız.

4. x ∈ R olmak uzere verilen x2 +(4x3 − 3x

)2= 1 denkleminin tum reel

cozumlerini bulunuz.

5.

I =

∫ √2 +

√2 + · · ·+ 2 + xdx

integralini hesaplayınız.(Not: Ifade icinde n ≥ 1 tane kok vardır.)

6. {xn}∞n>1 dizisi√xn+2 + 2 6 xn 6 2 esitsizligini saglandıgına gore, x1986

’nın alabilecegi tum degerleri bulunuz.

7.

2x+ x2y = y

2y + y2z = z

2z + z2x = x

denklem sisteminin tum reel cozumlerini bulunuz.

Page 80: MATEMATİK OLİMPİYATI ÇALIŞMA KİTAPÇIĞI · C»˜oz um.˜ Bu soruda »c˜oz ume daha kolay i»slemlerle ula»smak i»cin baz‡ de‚gi»s-˜ ken e‚gi»stirmeler yapmak yerinde

2.9. TRIGONOMETRIK DEGISKEN DEGISTIRME 79

8. Asagıdaki denklem sisteminin tum reel cozumlerini bulunuz.

x1 − 1

x1= 2x2

x2 − 1

x2= 2x3

x3 − 1

x3= 2x4

x4 − 1

x4= 2x1

9. x, y ∈ R olmak uzere, asagıdaki esitsizligi kanıtlayınız,

−1

26 (x+ y) (1− xy)

(1 + x2) (1 + y2)6 1

2

10. x ∈ R , {xn}n ve x1 = x olmak uzere

xn+1 =1

xn− 1

1 + xn

esitligi veriliyor. Eger xn = ±1 ise dizimiz kilitleniyor, cunku xn+1 ta-nımlanamıyor. Buna gore, 8. terimden sonra kilitlenen bu sekilde bir dizivar mıdır?

11. Reel tanımlı {ak}∞k=1 dizisi, k ∈ Z+ olmak uzere

ak+1 =k · ak + 1

k − ak

olarak veriliyor. Buna gore bu dizinin sonsuz sayıda pozitif ve sonsuzsayıda negatif terimi oldugunu kanıtlayınız.

12. −1 6 a1 6 a2 6 · · · 6 an 6 1 ise

n−1∑

i=1

√1− aiai+1 −

√(1− a2i

) (1− a2i+1

)<

π√2

2

esitsizligini kanıtlayınız.

Page 81: MATEMATİK OLİMPİYATI ÇALIŞMA KİTAPÇIĞI · C»˜oz um.˜ Bu soruda »c˜oz ume daha kolay i»slemlerle ula»smak i»cin baz‡ de‚gi»s-˜ ken e‚gi»stirmeler yapmak yerinde

80 BOLUM 2. KONULAR

13. x0 = 0, x1, x2, · · · , xn > 0 ven∑

k=1

xk = 1 ise

n∑

k=1

xk√1 + x0 + x1 + · · ·+ xk−1

√xk + xk+1 + · · ·+ xn

2

esitsizligini kanıtlayınız.

14. x, y, z ∈ (0, 1) olmak uzere

x2 + y2 + z2 + 2xyz = 1

esitligini saglayan tum (x, y, z) tamsayı uclulerini bulunuz.

15. a, b, c pozitif tamsayılar olmak uzere verilen

x+ y + z = a+ b+ c

4xyz − (a2x+ b2y + c2z) = abc

esitliklerini saglayan x, y, z reel sayılarını bulunuz.

16. cos 2π/5 ifadesinin degerini bulunuz.

2.9.2 Cozumler

1. t ∈ [0, π] olmak uzere cos t = x olsun. Cunku |x| ≤ 1 olmak zorundadır.Buradan

√1− x2 =

√1− cos2 x = sin t olacaktır. Cunku t ∈ [0, π] icin

sinus fonksiyonu pozitiftir. Buna gore esitsizligimiz,

sin t+ cos t > a

olacaktır.

f (t) = sin t+ cos t = 2 sinπ

4cos

(t− π

4

)=

√2 cos

(t− π

4

)

olacagından, f (t)’nin [0, π] aralıgındaki en buyuk degeri√2 olacaktır.

Buna gore, a’nın bulundugu aralık√2’den buyuk olmayan reel sayılar

kumesi olacaktır.

Page 82: MATEMATİK OLİMPİYATI ÇALIŞMA KİTAPÇIĞI · C»˜oz um.˜ Bu soruda »c˜oz ume daha kolay i»slemlerle ula»smak i»cin baz‡ de‚gi»s-˜ ken e‚gi»stirmeler yapmak yerinde

2.9. TRIGONOMETRIK DEGISKEN DEGISTIRME 81

2. Varsayalım sayılarımız a1, a2, a3, a4 olsun. Burada, ak = sin tk, tk ∈(0, π/2) alalım. Buna gore aslında problemde sorulan soru i ve j gibiiki indisin

0 < sin ti cos tj − sin tj cos ti <1

2

esitsizligini saglayacak bicimde varolup olmadıklarıdır. Eger esitsizligi-mizi duzenlersek

sin ti cos tj − sin tj cos ti = sin (ti − tj)

olacaktır. Buna gore, ispatlamamız gereken durum artık ti > tj veti − tj < π/6 olacak sekilde i ve j’nin bulundugunu gostermek olacak-tır. Burada devreye guvercin yuvası ilkesi girer oyleki, sececegimiz dortsayıdan iki tanesi kesinlikle

(0, π/6] , (0, π/3] , (π/3, π/2)

aralıgında olacaktır. Buna gore esitsizligi saglayan ti ve tj vardır, dola-yısıyla x ve y’de vardır.

3. x ∈ (0, π) olmak uzere, tum reel sayılar tanx cinsinden ifade edilebilir.Buna gore a = tanx ve b = tan y olarak alırsak

1 + tanx · tan y√1 + tan2 x ·

√1 + tan2 y

= cosx · cos y + sinx · sin y = cos (x− y)

olacaktır. Buna gore simdi problemimizi yeniden kuralım. Yeni durumu-muz,

cos(x− y) > 1/2

oldugundan (x− y) < π/3 olmalıdır. Gercektende (0, π) aralıgında sece-cegimiz dort sayıdan ikisinin farkı π/3’ten kucuk11 olacaktır.

4. Dikkat edilirse |x| ≤ 1 ve |4x3 − 3x| ≤ 1 olarak verilmistir. Buna gore,x = cos a, 0 ≤ a ≤ π alabiliriz. Eger cos 3a ve cos a ifadelerini kullanırsaksoruda verilen denklemimiz

cos2 a+ cos2 3a = 1

11Pigeon Hole Principle yada Guvercin Yuvası Ilkesi bu kucuklugu garantiler.

Page 83: MATEMATİK OLİMPİYATI ÇALIŞMA KİTAPÇIĞI · C»˜oz um.˜ Bu soruda »c˜oz ume daha kolay i»slemlerle ula»smak i»cin baz‡ de‚gi»s-˜ ken e‚gi»stirmeler yapmak yerinde

82 BOLUM 2. KONULAR

olacaktır. Buradan da,

1 + cos 2a

2+

1 + cos 6a

2= 1

olacagından 2a ve 4a degerleri

2a =π

2,3π

2ve 4a =

π

2,3π

2,5π

2,7π

2

olarak bulunur. Buna gore istenilen cozumler, ±π2 ve ±

√2±√

22 degerle-

rini alacaktır.

5. Varsayalım x = 2 cos t olsun, buna gore

√2 +

√2 + · · ·+√

2 + x =

√2 +

√2 + · · ·+√

2 + 2 cos t

=

√2 +

√2 + · · ·+ 2 cos

t

2

= 2 cost

2n

olacaktır. Buradan integralimiz

I =

∫2 cos

t

2ndx =

∫−2 cos

t

2n2 sin t dt

= −4

∫sin t · cos t

2ndt

= −2

∫ (sin

2n + 1

2nt− sin

2n − 1

2nt

)dt

esitliginden istenilen cevap,

2n+1

2n + 1cos

(2n+1

2n + 1arccos

x

2

)− 2n+1

2n − 1cos

(2n − 1

2narccos

x

2

)+ c

olarak bulunur.

Page 84: MATEMATİK OLİMPİYATI ÇALIŞMA KİTAPÇIĞI · C»˜oz um.˜ Bu soruda »c˜oz ume daha kolay i»slemlerle ula»smak i»cin baz‡ de‚gi»s-˜ ken e‚gi»stirmeler yapmak yerinde

2.9. TRIGONOMETRIK DEGISKEN DEGISTIRME 83

6. 0 ≤ xn ≤ 2 olduguna gore, xn = 2 cos yn, yn ∈ [0, π/2] degisken degistir-mesini kullanabiliriz. Buna gore

√xn+2 + 2 ≤ xn

esitsizligi ve

cos 2α+ 1 = 2 cos2 α

esitliginden

cos

(yn + 2

2

)≤ cos yn

esitsizligi bulunacaktır. Secilen aralıkta kosinus fonksiyonu azalan birfonksiyon olduguna gore,

yn+2

2≥ yn

olacaktır. Buradan da

yn ≤ yn+2k

2k, k ∈ Z+

elde edilir. Burada k degeri sonsuza giderken yn ≤ 0 yani yn = 0 olacak-tır. Dolayısıyla

xn = 2 cos yn = 2 · 1 = 2

olacagından xn = 2 ve x1986 = 2 bulunur.

7. Eger degiskenlerden herhangi biri, varsayalım x = ±1 olursa

2x+ x2y = y ⇒ 2 · ±1 + y = y ⇒ 2 = 0

olacaktır ki, bu durum imkansızdır. Buna gore denklem sistemimizi ye-niden yazarsak,

2x

1− x2= y

2y

1− y2= z

2z

1− z2= x

Page 85: MATEMATİK OLİMPİYATI ÇALIŞMA KİTAPÇIĞI · C»˜oz um.˜ Bu soruda »c˜oz ume daha kolay i»slemlerle ula»smak i»cin baz‡ de‚gi»s-˜ ken e‚gi»stirmeler yapmak yerinde

84 BOLUM 2. KONULAR

esitliklerini elde ederiz. Burada da x = tan a, a ∈ (−π/2, π/2) olarakalalım. Buna gore ilk iki denklemden y = tan 2a ve z = tan 4a olacaktır.Son denklemde ise tan 8a = tan a esitligi elde edilir. Buna gore 8a −a = kπ ise a = kπ/7 olacaktır. a ∈ (−π/2, π/2) olduguna gore k =−3,−2,−1, 0, 1, 2, 3 degerlerini alabiliriz. Buradan da istenilen degerlerbulunabilir.

8. Sorunun cozumune yardımcı olacak trigonometrik ozdeslik aslında soru-nun icine gomulmustur. Eger cift kat acı icin kotanjant toplamını kontroledersek

2 cot 2α = cotα− 1

cotα

esitligini elde ederiz. Eger x1 = cotα, α ∈ (0, π) olarak alırsak x2 =cot 2α olacagından, x3 = cot 4α ve x4 = cot 8α olacaktır. Benzer se-kilde dongu basa geleceginden x1 = cot 16α olacaktır. Demek ki cotα =cot 16α yani 16α − α = kπ, k ∈ Z esitligi bulunacaktır. Buradandacozumlerimiz

α =kπ

15, k = 1, 2, 3, · · · , 14

olacagından

x1 = cotkπ

15, x2 = cot

2kπ

15, x3 = cot

4kπ

15, x4 = cot

8kπ

15

olacaktır.

9. Varsayalım x = tan a ve y = tan b olsun. Buradan,

x+ y = tan a+ tan b =sin(a+ b)

cos a cos b

1− xy = 1− tan a tan b =cos(a+ b)

cos a cos b

esitlikleri elde edilir. Ayrıca,

1

1 + x2= cos2 a

1

1 + y2= cos2 b

Page 86: MATEMATİK OLİMPİYATI ÇALIŞMA KİTAPÇIĞI · C»˜oz um.˜ Bu soruda »c˜oz ume daha kolay i»slemlerle ula»smak i»cin baz‡ de‚gi»s-˜ ken e‚gi»stirmeler yapmak yerinde

2.9. TRIGONOMETRIK DEGISKEN DEGISTIRME 85

esitlikleri elde edilir. Eger yerine koyarsak,

−1

26

sin(a+b)cos a cos b · cos(a+b)

cos a cos b1

cos2 a1

cos2 b

6 1

2

ise −1 ≤ 2 sin(a + b)cos(a + b) ≤ 1 esitsizligini kanıtlamamız gerekir.Zaten bu esitsizlik −1 ≤ sin(2a+2b) ≤ 1 oldugundan kanıt tamamlanır.

10.1

1− xn− 1

1 + xn=

2xn1− x2n

esitligini yazabiliriz. Eger x1 = tanβ, β ∈ (−π/2, π/2) alırsak

x2 =2 tanβ

1− tan2 β= tan 2β

olacaktır. Bu sekilde devam edersek, xn = tan 2n−1β olacagından x8 =tan 27β = tan 128β olacaktır. 8 terim uzunlugundaki bir dizi icin, tan 128β =±1 olmalıdır. Buna gore

128β =(2k + 1)π

4

olacaktır. Dolayısıyla,

x = tan

(±(2k + 1)π

512

), k = −128, · · · , 128

olacaktır.

11. Eger b1 = tan−1 a1 ve bk+1 = bk + tan−1(1/k), k = 1, 2, 3, · · · olacakbicimde bir dizi tanımlarsak tanjant toplam formulunden

tan(x+ y) =tanx+ tan y

1− tanx tan y

esitliginin ısıgında ak = tan bk oldugunu gorebiliriz.

limx→0

tanx

x= 1

Page 87: MATEMATİK OLİMPİYATI ÇALIŞMA KİTAPÇIĞI · C»˜oz um.˜ Bu soruda »c˜oz ume daha kolay i»slemlerle ula»smak i»cin baz‡ de‚gi»s-˜ ken e‚gi»stirmeler yapmak yerinde

86 BOLUM 2. KONULAR

olduguna gore,

limk→∞

tan−1(1/k)

1/k= 1

olacaktır. Buradan da,

b0 +

∞∑

k=1

tan−1 1

k

ıraksak olacaktır. Ote yandan serinin elemanları k → ∞ iken, sıfıra ya-kınsayacaktır. (2πn, 2πn + π/2) aralıgında sonsuz coklukta parcalı top-lam olacaktır. Benzer bicimde (2πn+ π/2, (2n+1)π) aralıgında da son-suz coklukta toplam olacaktır. Fakat, parcalı toplamlardam birisi bm veam = tan bm oldugundan, demek ki sonsuz coklukta pozitif ve negatifam degeri bulunmaktadır.

13. Her bir ai degerimiz [−1, 1] aralıgında olduguna gore, ai = cosxi, xi ∈[0, π] seklinde secim yapabiliriz. Buna gore sorudaki esitsizligin sol tarafı

n−1∑

i=1

√1− cosxi cosxi+1 − sinxi sinxi+1 =

n−1∑

i=1

√1− cos (xi+1 − xi)

=√2n−1∑

i=1

sinxi+1 − xi

2olacaktır.

Sinus fonksiyonu [0, π] aralıgında asagı konkav olduguna gore JensenEsitsizliginden

1

n− 1

n−1∑

i=1

sin

(xi+1 − xi

2

)6 sin

(1

n− 1

n−1∑

i=1

xi+1 − xi2

)

esitsizligi elde edilir. Buna gore,

√2n−1∑

i=1

sin

(xi+1 − xi

2

)6 (n− 1)

√2 sin

xn − xn−1

2 (n− 1)6

√2 (n− 1) sin

π

2 (n− 1)

olacaktır. Burada xn − xi ∈ (0, π) dir. Aynı aralıkta, sinx < x, x > 0esitsizligi kullanılırsa

√2 (n− 1) sin

2 (n− 1)

)6

√2π

2

Page 88: MATEMATİK OLİMPİYATI ÇALIŞMA KİTAPÇIĞI · C»˜oz um.˜ Bu soruda »c˜oz ume daha kolay i»slemlerle ula»smak i»cin baz‡ de‚gi»s-˜ ken e‚gi»stirmeler yapmak yerinde

2.9. TRIGONOMETRIK DEGISKEN DEGISTIRME 87

olacaktır.

13. Tum xi degerleri pozitif ve toplamlarıda 1 olduguna gore,

x1 + x2 + · · ·+ xk = sin ak

vea0 = 0 < a1 < a2 < · · · < an =

π

2, k = 0, 1, · · · , n

secimlerini yapabiliriz. Buna gore,

n∑

k=1

sin ak − sin ak−1√1 + sin ak−1

√1− sin ak−1

2

olacaktır. Bu esitsizligi yeniden duzenlersek

n∑

k=1

2 sin(ak−ak−1

2

)cos

(ak+ak−1

2

)

cos (ak − 1)

olacaktır. 0 < x < π/2 aralıgında kosinus fonksiyonu azalandır ve sinx <x esitsizligi vardır. Buna gore esitsizligin sol tarafı

n∑

k=1

2ak−ak−1

2 cos ak−1

cos ak−1

isen∑

k=1

(ak − ak−1) =π

2

olacaktır ki bu da zaten ispatı tamamlar.

14. Denklemi saglayan uclulerden herbiri dar acılı bir ucgenin acılarının ko-sinus degerleridir. Once, eger A, B, C acıları bir ucgenin acı degerleriise

cos2A+ cos2B + cos2C + 2 cosA cosB cosC = 1

oldugunu gosterelim. Aslında,

cosA = − cos (B + C) = sinB sinC − cosB cosC

Page 89: MATEMATİK OLİMPİYATI ÇALIŞMA KİTAPÇIĞI · C»˜oz um.˜ Bu soruda »c˜oz ume daha kolay i»slemlerle ula»smak i»cin baz‡ de‚gi»s-˜ ken e‚gi»stirmeler yapmak yerinde

88 BOLUM 2. KONULAR

olacagına gore

= cos2A+ cos2B + cos2C + 2 cosA cosB cosC

= (cosA+ cosB + cosC)2 + 1− (1− cos2A

) (1− cos2C

)

= (sinB sinC)2 + 1− sin2B sin2C = 1

olarak bulunur. Demek ki, A + B + C = π esitligini saglayan A, B, Cdegerleri birer cozumdur.

15. Ikinci denklemi duzenlersek

a2

yz+

b2

zx+

c2

xy+

abc

xyz= 4

olacaktır. Buna gore,

x1 =a√yz

, y1 =b√zx

, z1 =c√xy

olarak alırsak, esitligimiz

x21 + y21 + z21 + x1y1z1 = 4

olacagından 0 < x1 < 2, 0 < y1 < 2, 0 < z1 < 2 olacaktır. Buna gore,

x1 = 2 cosA, y1 = 2 cosB, z1 = 2 cosC

olarak alabiliriz. Bu degerleri kullanırsak,

2√yz cosA = a, 2

√zx cosB = b, 2

√xy cosC

olacaktır. Eger x+ y + z = a+ b+ c ise

x+ y + z − 2√yx cosA−

√2zx cosB − 2

√xy cosC = 0

olacaktır. Bu esitligin sol tarafını iki kare toplamı seklinde yazmaya ca-lısırsak

= x+ y + z − 2√yz cosA− 2

√zx cosB − 2

√xy cosC

= x+ y + z − 2√yz cosA− 2

√zx cosB + 2

√xy (cosA cosB − sinA sinB)

= x(sin2B + cosB2

)+ y

(sin2A+ cos2A

)+ z − 2

√yz cosA

− 2√zx cosB + 2

√xy cosA cosB − 2

√xy sinA sinB

=(√

x sinB −√y sinA

)2+(√

x cosB +√y cosA−√

z)

Page 90: MATEMATİK OLİMPİYATI ÇALIŞMA KİTAPÇIĞI · C»˜oz um.˜ Bu soruda »c˜oz ume daha kolay i»slemlerle ula»smak i»cin baz‡ de‚gi»s-˜ ken e‚gi»stirmeler yapmak yerinde

2.9. TRIGONOMETRIK DEGISKEN DEGISTIRME 89

olacaktır. Burada iki kare toplamı sıfır olduguna gore

√x sinB −√

y sinA = 0

ve √x cosB +

√y cosA =

√z

olacaktır. Buradan,

√z =

√x

b

2√zx

+√y

a

2√yz

=b+ a

2√z

olacaktır. Demek ki

z =a+ b

2

’dir. Simetriden dolayı

y =a+ c

2ve x =

b+ c

2

olarak bulunur.

16. Once bildigimiz bazı trigonometrik soruda kullanabilecegimiz bazı ge-ometrik esitlikleri yazalım. Buna gore,

cos 2θ = 2 cos2 θ − 1

vecos 3θ = 4 cos3 θ − 3 cos θ

esitliklerini kullanmamız yerinde olacaktır. Eger x = 2π/5 alırsak, cos 6π/5 =cos 4π/5 olacaktır. Simdi denklemi x degiskenine bagımlı olarak yazalım.Burada

4x3 − 2x2 − 3x+ 1 = 0

ise(x− 1)

(4x2 + 2x− 1

)= 0

olacaktır. Buradan

x = 1 veya x =

√5− 1

2

Page 91: MATEMATİK OLİMPİYATI ÇALIŞMA KİTAPÇIĞI · C»˜oz um.˜ Bu soruda »c˜oz ume daha kolay i»slemlerle ula»smak i»cin baz‡ de‚gi»s-˜ ken e‚gi»stirmeler yapmak yerinde

90 BOLUM 2. KONULAR

olacagından cos(2π/5) 6= 1 ise istenilen cevap

cos

(2π

5

)=

√5− 1

2

olacaktır.

2.10 Cebırde Teleskopık Toplamlar ve Carpımlar

Bu konuda da cebirde sonsuz toplamlar ve farklarla alakalı soruları cozece-giz. Aslında cozumlerde kullanacagımız genel yontem oldukca basittir. Sorudaverilen toplamı yada carpımı, farkların toplamı biciminde yazarak sonuca ulas-maya calısacagız. Buna gore elde edecegiz farkların toplamı

n∑

k=2

[F (k)− F (k − 1)]

formunda olacagından ve 2 ≤ k ≤ n− 1 indislerine sahip degerler sadelesecegiicin, toplamımız

F (n)− F (1)

olacaktır.

Ornek.n∑

k=1

k!k

toplamını hesaplayınız.

Cozum. Eger k! ·k = k!(k+1−1) = (k+1)!−k! degisken degistirmesiniyaparsak,

n∑

k=1

(k + 1)!− k! = (n+ 1)!− 1

olacaktır.

Page 92: MATEMATİK OLİMPİYATI ÇALIŞMA KİTAPÇIĞI · C»˜oz um.˜ Bu soruda »c˜oz ume daha kolay i»slemlerle ula»smak i»cin baz‡ de‚gi»s-˜ ken e‚gi»stirmeler yapmak yerinde

2.10. CEBIRDE TELESKOPIK TOPLAMLAR VE CARPIMLAR 91

Ornek.n∑

k=1

1

(k + 1)√k + k

√k + 1

toplamını hesaplayınız.

Cozum. Temel dusunce paydayı rasyonel yapmak olmalıdır. Yani,

((k + 1)

√k − k

√k + 1

)((k + 1)

√k + k

√k + 1

)= k (k + 1)2 − (k + 1) k2

= k(k + 1)

olduguna gore,

n∑

k=1

(k + 1)√k − k

√k + 1

k (k + 1)=

n∑

k=1

(1√k− 1√

k + 1

)= 1− 1√

n+ 1

olacaktır.

Ornek. n ∈ Z+ olmak uzere asagıda verilen esitsizligi kanıtlayınız.

n− 1 <1√

1 +√2+

1√2 +

√5+ · · ·+ 2n− 1√

(n− 1)2 + 1 +√n2 + 1

< n

Cozum. Soruda verilen esitsizligi iki yonunude ispatlamadan evvel, bazıgenellemeler yapalım. Buna gore,

2k − 1√(k − 1)2 + 1 +

√k2 + 1

=

(2k − 1)

(√k2 + 1−

√(k − 1)2 + 1

)

k2 + 1− (k − 1)2 − 1

=√k2 + 1−

√(k − 1)2 + 1

ise

Page 93: MATEMATİK OLİMPİYATI ÇALIŞMA KİTAPÇIĞI · C»˜oz um.˜ Bu soruda »c˜oz ume daha kolay i»slemlerle ula»smak i»cin baz‡ de‚gi»s-˜ ken e‚gi»stirmeler yapmak yerinde

92 BOLUM 2. KONULAR

=1√

1 +√2+

3√2 +

√5+ · · ·+ 2n− 1√

(n− 1)2 + 1 +√n2 + 1

=√2− 1 +

√5−

√2 + · · ·+

√n2 + 1−

√(n− 1)2 + 1

=√

n2 + 1− 1

olacaktır.

Buna gore, yeni esitsizligimiz

n− 1 <√

n2 + 1 < n

olacaktır ki, bu esitsizligi gostermek oldukca kolaydır.

Ornek. x, Q(x) polinomunun bir koku degildir. Buna gore,

P (x)

Q (x)− P (x+ 1)

Q (x+ 1)=

1

x (x+ 2)

esitligini saglayan tum P ve Q polinomlarını bulunuz.

Cozum. Once soruda verilen esitligi duzenleyelim, buna gore

P (x)

Q (x)− P (x+ 1)

Q (x+ 1)=

2x+ 1

2x (x+ 1)− 2 (x+ 1) + 1

2 (x+ 1) (x+ 2)

esitligi elde edilir. Buna gore, n ∈ Z+ ve yeterince buyuk bir x degeri icin,

P (x+ k)

Q (x+ k)−P (x+ k + 1)

Q (x+ k + 1)=

2 (x+ k) + 1

2 (x+ k + 1) (x+ k + 2)− 2 (x+ k + 1) + 1

2 (x+ k + 1) (x+ k + 2)

olacaktır. Eger esitlikleri taraf tarafa toplarsak,

P (x)

Q (x)− P (x+ n)

Q (x+ n)=

2x+ 1

2x (x+ 1)− 2 (x+ n) + 1

2 (x+ n) (x+ n+ 1)

esitligini elde ederiz. Burada n ⇒ ∞ iken, P (x+n)/Q(x+n) ifadeside sonsuzagidecektir. Buna gore,

P (x)

Q (x)=

2x+ 1

2x (x+ 1)+ c, c ∈ R

Page 94: MATEMATİK OLİMPİYATI ÇALIŞMA KİTAPÇIĞI · C»˜oz um.˜ Bu soruda »c˜oz ume daha kolay i»slemlerle ula»smak i»cin baz‡ de‚gi»s-˜ ken e‚gi»stirmeler yapmak yerinde

2.10. CEBIRDE TELESKOPIK TOPLAMLAR VE CARPIMLAR 93

olacaktır. Eger islemi devam ettirirsek,

2x (x+ 1)P (x) = (2x+ 1 + 2cx (x+ 1))Q (x)

esitligini elde ederiz. Buradan istenen polinomlar

P (x) = (2x+ 1 + 2cx (x+ 1))R (x) ve Q (x) = 2x (x+ 1)R (x)

olarak bulunur.

Ornek. ∞∏

n=2

(1− 1

n2

)=

1

2

esitligini kanıtlayınız.

Cozum.

limN→∞

N∏

n=2

(1− 1

n2

)= lim

N→∞

N∏

n=2

(1− 1

n

)(1 +

1

n

)=

N∏

n=2

n− 1

n

N∏

n=2

n+ 1

n

=1

N· N + 1

2=

N + 1

2N

olduguna gore,

limN→∞

N + 1

2N=

1

2bulunur.

2.10.1 Calısma Soruları

1.n∑

k=1

k!(k2 + k + 1

)

toplamını hesaplayınız.

2. a1, a2, · · · , an bir aritmetik dizinin elemanları ve ortak farkları d ise

n∑

k=1

1

ak · ak+1

toplamını hesaplayınız.

Page 95: MATEMATİK OLİMPİYATI ÇALIŞMA KİTAPÇIĞI · C»˜oz um.˜ Bu soruda »c˜oz ume daha kolay i»slemlerle ula»smak i»cin baz‡ de‚gi»s-˜ ken e‚gi»stirmeler yapmak yerinde

94 BOLUM 2. KONULAR

3. ∞∑

k=1

6k

(3k − 2k) (3k+1 − 2k+1)

toplamını hesaplayınız.

4. {xn}∞n=1 dizisinde x1 = 1/2 , xk+1 = x2k + xk ise

1

x1 + 1+

1

x2 + 1+ · · ·+ 1

x100 + 1

toplamından kucuk en buyuk tamsayı degerini bulunuz.

5. Fn bir Fibonacci dizisi olmak uzere F1 = 1, F2 = 1, Fn+1 = Fn + Fn−1

olarak veriliyor. Buna gore asagıdaki toplamları hesaplayınız.

a. ∞∑

n=2

Fn

Fn−1 · Fn+1

b. ∞∑

n=2

1

Fn−1 · Fn+1

6. √1 +

1

12+

1

22+

√1 +

1

22+

1

32+ · · ·+

√1 +

1

19992+

1

20002

toplamını hesaplayınız.

7.1√

1 +√3+

1√5 +

√7+ · · ·+ 1√

1997 +√9999

> 24

esitsizligini kanıtlayınız.

8. 1 6 m < n m,n ∈ Z olduguna gore, asagıdaki esitsizligi kanıtlayınız.

2(√

n+ 1−√m)<

1√m+

1√m+ 1

+· · ·+ 1√n− 1

+1√n< 2

(√n−√

m− 1)

Page 96: MATEMATİK OLİMPİYATI ÇALIŞMA KİTAPÇIĞI · C»˜oz um.˜ Bu soruda »c˜oz ume daha kolay i»slemlerle ula»smak i»cin baz‡ de‚gi»s-˜ ken e‚gi»stirmeler yapmak yerinde

2.10. CEBIRDE TELESKOPIK TOPLAMLAR VE CARPIMLAR 95

9.

ak =k

(k − 1)3 + k4/3 + (k + 1)4/3

ise999∑

k=1

ak < 50

esitsizligini kanıtlayınız.

10. ∞∑

n=1

1

(n+ 1)√n< 2

esitsizligini kanıtlayınız.

11. Fn, fibonacci serisi olmak uzere verilen,

∞∑

n=0

1

F2n

toplamının esitini bulunuz.

12. ∞∏

n=2

n3 − 1

n3 + 1=

2

3

esitligini kanıtlayınız.

13. ∞∑

n=0

(1 +

1

22n

)

carpımını hesaplayınız.

14. L1 = 2, L2 = 2 ve Ln+2 = Ln+1 + Ln, n > 1 olarak tanımlanan LucasDizisi veriliyor. Buna gore,

m∏

k=1

L2k+1 = F2m+1

esitligini kanıtlayınız.

Page 97: MATEMATİK OLİMPİYATI ÇALIŞMA KİTAPÇIĞI · C»˜oz um.˜ Bu soruda »c˜oz ume daha kolay i»slemlerle ula»smak i»cin baz‡ de‚gi»s-˜ ken e‚gi»stirmeler yapmak yerinde

96 BOLUM 2. KONULAR

Edouard Lucas12 (1842-1891)

Fransız matematikci ve bilimadamı olan Lucasdaha cok kendi adı ile anılan Lucas Dizileri ve Fi-bonacci Dizisinin n. terimini veren formulu ile anı-lır. Universite egitimini Ecole Normale Superieure’detamamlayan Lucas uzun bir sure Paris Gozleme-vinde’de calısmalarını surdurmus ve Profesorluk un-vanınıda yine aynı sehirde Paris’te almıstır. Ayrıca

N∑

n=1

n2 = M2

Diphantine denkleminin yegane cozumunun N = 24ve M = 70 ikilisi oldugunu savunmustur. Bu ikili-den baska ikililerin olmadıgı ise ancak 1918 yılındahiperbolik fonksiyonlar kullanılarak kanıtlanabilmis-tir. Ayrıca, sayıların asallıgı uzerinede calısmalar ya-pan Lucas 19 yıllık bir calısma sonucunda, 2127 − 1 sayısının asal oldugunukanıtlamıs ve bu kanıtı sonucunda ulastıgı sayı o zamana kadar ulasılabilenen buyuk Mersenne Asalı olarak kabul edilmistir. Matematigin eglenceli tara-fıylada alakadar olan Lucas, ozellikle Hanoi Kuleleri’ni kesfetmesi ve cozumyontemleri gelistirmesi bulmaca meraklılarının buyuk saygısını kazanmıstır.

2.10.2 Cozumler

1.

n∑

k=1

k!(k2 + k + 1

)=

n∑

k=1

[(k + 1)2 − k

]k!

=

n∑

k=1

[(k + 1)! (k + 1)− k!k]

= (n+ 1)! (n+ 1)− 1 olacaktır.

12Francois Edouard Anatole Lucas

Page 98: MATEMATİK OLİMPİYATI ÇALIŞMA KİTAPÇIĞI · C»˜oz um.˜ Bu soruda »c˜oz ume daha kolay i»slemlerle ula»smak i»cin baz‡ de‚gi»s-˜ ken e‚gi»stirmeler yapmak yerinde

2.10. CEBIRDE TELESKOPIK TOPLAMLAR VE CARPIMLAR 97

2.n∑

k=1

1

akak+1=

1

d

n∑

k=1

ak+1 − akakak+1

=1

d

n∑

k=1

(1

ak− 1

ak+1

)

olacagından, toplam1

d

an+1 − a1an+1a1

olacaktır. an+1 − a1 = nd olduguna gore

1

d

nd

(nd+ a1) a1=

n

(nd+ a1) a1

olacaktır.

3. Kesri parcalara bolerek cozume gitmek yerinde olacaktır. Buna gore,

6k

(3k − 2k) (3k+1 − 2k+1)=

A

3k − 2k− B

3k+1 − 2k+1

esitliginde (3k+1 − 2k+1

)A−

(3k − 2k

)B = 6k

olacaktır. Buradan A = 2k ve B = 2k + 1 olarak kolaylıkla bulunabilir.Buna gore,

∞∑

k=1

6k

(3k − 2k) (3k+1 − 2k+1)=

2

3− 2− lim

k→∞2k+1

3k+1 − 2k+1= 2

olarak bulunur.Not. Benzer bicimde A = 3k ve Bk +1 bulunabilirdi, ancak bu degerlersonucu degistirmezdi. Deneyip gorebilirsiniz.

4. Indirgemeli dizi cozum yontemini kullanalım. Buna gore, xk+1 = x2k+xkise

1

xk+1=

1

xk (xk + 1)=

1

xk− 1

xk + 1

olacagından

1

x1 + 1+

1

x2 + 1+ · · ·+ 1

x100 + 1=

1

x1− 1

x101

Page 99: MATEMATİK OLİMPİYATI ÇALIŞMA KİTAPÇIĞI · C»˜oz um.˜ Bu soruda »c˜oz ume daha kolay i»slemlerle ula»smak i»cin baz‡ de‚gi»s-˜ ken e‚gi»stirmeler yapmak yerinde

98 BOLUM 2. KONULAR

olacaktır. x1 = 1/2 ve 0 < 1/x101 < 1 olduguna gore istenen deger 1olacaktır.

5. a.

∞∑

n=2

Fn

Fn−1Fn+1=

∞∑

n=2

Fn+2 − Fn−1

Fn−1Fn+1=

∞∑

n=2

(1

Fn−1− 1

Fn+1

)

= limn→∞

(1

F1+

1

F2− 1

Fn− 1

Fn+1

)=

1

F1+

1

F2= 2 olacaktır.

b.

∞∑

n=2

1

Fn−1Fn+1=

∞∑

n=2

Fn

Fn−1FnFn+1=

∞∑

n=2

Fn+1 − Fn−1

Fn−1FnFn+1

=∞∑

n=2

(1

Fn−1Fn− 1

FnFn+1

)

= limn→∞

(1

F1F2− 1

FnFn+1

)=

1

F1F2= 1

olacaktır.

6. n ∈ Z+ olmak uzere,

1 +1

n2+

1

(n+ 1)2=

n2 (n+ 1)2 + (n+ 1)2 + n2

n2 (n+ 1)2=

(n2 + n+ 1

)

n2 (n+ 1)2

ise √1 +

1

n2+

1

(n+ 1)2=

n2 + n+ 1

n2 + n= 1 +

1

n (n+ 1)

olacaktır. Buna gore,

1997∑

n=1

(1 +

1

n (n+ 1)

)=

1999∑

n=1

(1 +

1

n− 1

n+ 1

)= 2000− 1

2000

olur.

Page 100: MATEMATİK OLİMPİYATI ÇALIŞMA KİTAPÇIĞI · C»˜oz um.˜ Bu soruda »c˜oz ume daha kolay i»slemlerle ula»smak i»cin baz‡ de‚gi»s-˜ ken e‚gi»stirmeler yapmak yerinde

2.10. CEBIRDE TELESKOPIK TOPLAMLAR VE CARPIMLAR 99

7. Teleskopik toplam yapmak mantıklı gorunsede, bazı elemanlar eksik ol-dugundan cozume gidemeyiz. Buna gore esitsizligin sol kısmı,

1√3 +

√5+

1√7 +

√9+ · · ·+ 1√

9999 +√10001

toplamından kucuktur. Eger sorudaki esitsizlikle son yazdıgımız esitsiz-ligi toplarsak

1√1 +

√3+

1√3 +

√5+

1√5 +

√7+ · · ·+ 1√

9999 +√10001

> 48

olacaktır. Simdi artık teleskopik toplamı kullanabilriz. Buna gore,

1

2

(√3−

√1 +

√5−

√3 +

√7−

√5 + · · ·+

√10001−

√9999

)=

1

2

(√10001− 1

)> 48

olacaktır.

8. k ∈ R icin(√

k + 1− k) (√

k + 1 + k)= 1 olduguna gore,

2(√

k + 1−√k)=

2√k + 1 +

√k<

1

k

ve1√k<

2√k +

√k − 1

= 2(√

k −√k − 1

)

olacaktır. Buna gore,

2(√

k + 1−√k)<

1√k< 2

(√k −

√k − 1

)

olacaktır. Eger bu esitsizlikleri toplarsak,

2(√

n+ 1−√m)<

1√m+

1√m+ 1

+· · ·+ 1√n− 1

+1√n< 2

(√n−√

m− 1)

olacagından soruda verilen esitsizlik kanıtlanmıs olur.

Page 101: MATEMATİK OLİMPİYATI ÇALIŞMA KİTAPÇIĞI · C»˜oz um.˜ Bu soruda »c˜oz ume daha kolay i»slemlerle ula»smak i»cin baz‡ de‚gi»s-˜ ken e‚gi»stirmeler yapmak yerinde

100 BOLUM 2. KONULAR

9. ilk once an ifadesinin paydasını kucultelim ve rasyonel hale getirelim.Buna gore, k3 yerine (k − 1)2/3(k + 1)2/3 yazılırsa,

ak <k

(k − 1)4/3 + (k − 1)2/3 (k + 1)2/3 + (k + 1)4/3

=k((k + 1)2/3 − (k − 1)2/3

)

(k + 1)2 − (k − 1)2=

1

4

((k + 1)2/3 − (k − 1)2/3

)olacagından

999∑

k=1

an =1

4

999∑

k=1

((k + 1)2/3 − (k − 1)2/3

)

=1

4

(10002/3 + 9992/3 − 12/3 − 02/3

)<

1

4(100 + 100− 1) < 50 bulunur.

10. Verilen toplamı duzenlersek

1√n (n+ 1)

=

√n

n (n+ 1)=

√n

n−

√n

n+ 1

olacagından yeni toplamımız

1 +

∞∑

n=2

√n−√

n− 1

n

olacaktır. Bu toplam teleskopik degildir, ancak ustten sınırlıdır. Oyleki

1 +∞∑

n=2

√n−√

n− 1√n√n− 1

= 1 +∞∑

k=2

(1√n− 1

− 1√n

)

olacakır. Ve bu toplam da 2’ye yakınsar. Ispat tamamlanır.

11. Sorunun cozumune tumevarım ile baslayalım. Buna gore, tumevarımyontemi kullanılarak

F2mFm−1 − F2m−1Fm = (−1)m Fm, m > 1

Page 102: MATEMATİK OLİMPİYATI ÇALIŞMA KİTAPÇIĞI · C»˜oz um.˜ Bu soruda »c˜oz ume daha kolay i»slemlerle ula»smak i»cin baz‡ de‚gi»s-˜ ken e‚gi»stirmeler yapmak yerinde

2.10. CEBIRDE TELESKOPIK TOPLAMLAR VE CARPIMLAR 101

oldugu bulunabilir. Eger m = 2n−1 alınırsa

F2nF2n−1−1 − F2n−1F2n−1 = F2n−1 , n > 2

veya1

F2n=

F2n−1−1

F2n−1

− F2n−1

F2n, n > 2

olacagından

∞∑

n=0

1

F2n=

1

F1+

1

F2+ lim

N→∞

(F1

F2− F2N−1

F2N

)= 3− 1

√5+12

=7−√

5

2

olacaktır.

12.

limN→∞

N∏

n=2

n3 − 1

n3 + 1= lim

N→∞

N∏

n=2

(n− 1)(n2 + n+ 1

)

(n+ 1) (n2 − n+ 1)

= limN→∞

N∏

n=2

n− 1

n+ 1

N∏

n=2

(n+ 1)2 − (n+ 1) + 1

n2 − n+ 1

= limN→∞

1 · 2 ·((N + 1)2 − (N + 1) 4 + 1

)

3N · (N + 1)=

2

3

olarak bulunur.

13. Eger soruda verilen ifadeyi acarsak

∞∏

n=0

(1 +

1

22n

)= 2 lim

N→∞

(1− 1

20

) ∞∏

n=0

(1 +

1

22n

)

olacaktır. Buradan,(1− 1

22n

)(1 +

1

22n

)=

(1− 1

22n+1

)

olacagından, carpımımızı teleskopik hale getirebiliriz. Buradan carpım

1− 1

22N+1

olacaktır.N → ∞ ve 1− 1

22N+1 → 1 ise carpımın sonucu 2 olarak bulunur.

Page 103: MATEMATİK OLİMPİYATI ÇALIŞMA KİTAPÇIĞI · C»˜oz um.˜ Bu soruda »c˜oz ume daha kolay i»slemlerle ula»smak i»cin baz‡ de‚gi»s-˜ ken e‚gi»stirmeler yapmak yerinde

102 BOLUM 2. KONULAR

14. Eger kontrol edilirse Fibonacci ve Lucas dizilerinin

Fn+1 + Fn−1 = Ln+1 ve F2n = F 2n+1 − F 2

n−1, n > 1

icin saglandıgı gorulebilir. Oyleyse,

F2n = (Fn+1 + Fn−1) (Fn+1 − Fn−1) = Ln+1Fn

olacaktır. Buna gore,

Ln+1 =F2n

Fn, n > 1

olacaktır. Oyleyse,

m∏

k=1

L2k+1 =m∏

k=1

F2k+1

F2k=

F2m+1

F2= F2m+1

olacaktır. Ispat tamamlanır.

2.11 Tamdeger Fonksıyon Problemlerı

x ∈ R olmak uzere, x degerini gecmeyen en buyuk tamsayı degeri [x] ilegosterilir. Bu foksiyona tam deger fonsiyon (floor function) denir. Buna gorex − 1 < [x] ≤ x oldugu acıktır. Bu ifade ayrıca [x] ≤ x < [x] + 1 olaraktayazılabilir. Ayrıca sayının ondalıklı kısmıda {x} ile temsil edilir. Buna gore;{x} = x − [x] dir. Buna gore x = [x] + {x}, 0 ≤ {x} < 1 dir. Tam degerfonksiyonun en temel teoremleri asagıda verilmistir.

Teorem. α, β ∈ R, a ∈ Z, n ∈ N ise;

1. [α+ a] = [α] + a

2. [αn ] = [ [α]n ]

3. [α] + [β] ≤ [α+ β] ≤ [α] + [β] + 1

Kanıt.

Page 104: MATEMATİK OLİMPİYATI ÇALIŞMA KİTAPÇIĞI · C»˜oz um.˜ Bu soruda »c˜oz ume daha kolay i»slemlerle ula»smak i»cin baz‡ de‚gi»s-˜ ken e‚gi»stirmeler yapmak yerinde

2.11. TAMDEGER FONKSIYON PROBLEMLERI 103

1. Varsayalım m = [α+ a] olsun. Buna gore m ≤ α+ a < m+1 dolayısıylada m − a ≤ α < m − a + 1 olacaktır. m − a = [α] olduguna gore ispattamamlanmıs olur.

2. αn ifadesini α

n = [αn +θ], 0 ≤ θ < 1 olarak yazalım. n · [αn ] ifadesi oldugnanartık elimizde 1 durumu olustu. Yani;

[α] = [n[α/n] + nθ] = n[α/n] + [nθ] olur.

Simdi, 0 ≤ [nθ] < nθ < n ve bundan dolayı 0 ≤ [nθ/n] < 1 olur.Eger

Θ = [nθ]/n oldugunu varsayarsak [α]n = [αn ]+Θ, 0 ≤ Θ < 1 olur ki, buda

zaten istenen sonuctur.

3. Varsayalım α = n+ r, β = m+ t ve 0 ≤ r < 1, 0 ≤ t < 1 olsun.

[α] + [β] = n+m ≤ [n+ r +m+ t] = [α+ β] olur.

= m+ n+ [r + t] ≤ n+m+ 1

= [α] + [β] + 1 olur.

Ornek. [x3 ] = x2 + 1 denklemini saglayan x tamsayılarının toplamını

bulunuz.

Cozum. Varsayalım [x3 ] = t olsun. Buna gore, t ≤ x3 < t + 1 ise 3t ≤

x < 3t + 3 olmalıdır. [x3 ] = t = x2 + 1 olduguna gore x = 2t − 2 olur. Bu

son esitligi esitsizligimizde yerine koyarsak 3t ≤ 2t − 2 < 3t + 3 ise −5 < t ≤−2 bulunur. t = −2,−3,−4 olabileceginden x degerleride −8,−10,−6 olarakbulunur. istenen cevap −24 olur.

Teorem.(Hermite Ozdesligi) [nx] = [x]+[x+ 1n ]+[x+ 2

n ]+ · . . . ·+[x+ n−1n ]

dir.Kanıt. Eger teoremde verilen esitligi duzenlersek;

n∑

k=0

[x+k

n] = [nx]

Page 105: MATEMATİK OLİMPİYATI ÇALIŞMA KİTAPÇIĞI · C»˜oz um.˜ Bu soruda »c˜oz ume daha kolay i»slemlerle ula»smak i»cin baz‡ de‚gi»s-˜ ken e‚gi»stirmeler yapmak yerinde

104 BOLUM 2. KONULAR

ifadesini elde ederiz. Oyleyse bu esitligi ispatlamamız yeterlidir. x = [x] + {x}oldugunu zaten biliyoruz. Buna gore; {1, 2, 3, . . . , n} kumesinin elemanı olankesin bir k′ degeri vardır ki;

[x] = [x+k′ − 1

n] ≤ x < [x+

k′

n] = [x] + 1

ise

0 = [{x}+ k′ − 1

n] ≤ {x} < [{x}+ k′

n] = 1

olacaktır. Buna gore;

1− k′

n≤ {x} < 1− k′ − 1

n⇒ n− k′ ≤ n{x} < n− k′ + 1 olur.

En basta yazdıgımız esitligi kullanırsak

n−1∑

k=0

[x+k

n] =

k′−1∑

k=0

[x] +n−1∑

k=k′([x] + 1) = n · [x] + n− k′

= n[x] + [n{x}]= [n[x] + n{x}] = [nx] olur.

Ornek. [x]+[2x]+[4x]+[8x]+[16x]+[32x] = 12345 denklemini saglayanx degerlerini bulunuz.

Cozum. Bu soruda amacımız once, denkligi saglayan x degerlerinin var-lıgını arastırmak ve bu degerleri bulmak olacaktır. Konunun en basında ver-digimiz teoremlerden biliyoruz ki, x− 1 < [x] ≤ x olacaktır. Buna gore;

x−1+2x−1+4x−1+. . .+32x−1 < [x]+[2x]+. . .+[32x] ≤ x+2x+4x+. . .+32x

ise 63x − 6 < 12345 ≤ 63x ve x degeride 195 < x < 196 arasında olacaktır.Eger x degerini 2 lik tabanda yazarsak x = 195+ a1

2 + a222

+ . . ., ak = 0 veya 1.Buna gore;[2x] = 2 · 195 + a1[4x] = 4 · 195 + 2a1 + 2a2[8x] = 8 · 195 + 4a1 + 2a2 + a3

Page 106: MATEMATİK OLİMPİYATI ÇALIŞMA KİTAPÇIĞI · C»˜oz um.˜ Bu soruda »c˜oz ume daha kolay i»slemlerle ula»smak i»cin baz‡ de‚gi»s-˜ ken e‚gi»stirmeler yapmak yerinde

2.11. TAMDEGER FONKSIYON PROBLEMLERI 105

[16x] = 16 · 195 + 8a1 + 4a2 + 2a3 + a4[32x] = 32 · 195 + 16a1 + 8a2 + 4a3 + 2a4 + a5

Eger bu degerleri alt alta toplarsak 63 ·195+31 ·a1+15 ·a2+7a3+3 ·a4+a5olacaktır yani; 31 · a1+15 · a2+7 · a3+3 · a4+ a5 = 60 olur. Ancak 0 ≤ ai ≤ 1oldugundan ifade en fazla 31 + 15 + 7 + 3 + 1 = 57 degerini alabilir. Celiskivardır. Cozumu saglayan bir x degeri yoktur.

Ornek. [x], x in tamdeger foksiyonu olmak uzere, {x} = x − [x] olaraktanımlansın. Her x reel sayısı icin x = f(x) − 2 · f({x}) esitligini saglayan ffonksiyonunun x = −13

5 noktasındaki degerini bulunuz.

Cozum. {−135 } = −13

5 − (−3) = 25 olugundan; x = 2

5 icin

2

5= f(

2

5)− 2 · f({2

5}) ⇒ f(

2

5) = −2

5

olacaktır. x = −135 icin

−13

5= f(−13

5)− 2 · f({−13

5}) = f(−13

5)− 2 · f(2

5)

olduguna gore

f(−13

5) = −17

5

olur.

Teorem. a ve b aralarında asal dogal sayılar olmak uzere;

[a

b] + [

2a

b] + [

3a

b] + . . .+ [

(a− 1) · bb

] =(a− 1) · (b− 1)

2

esitligi vardır.Kanıt. Aslında teoremde verilen ifadeyi duzenlersek;

a−1∑

k=1

[kb

a] =

b−1∑

k=1

[ka

b] =

(a− 1)(b− 1)

2

Page 107: MATEMATİK OLİMPİYATI ÇALIŞMA KİTAPÇIĞI · C»˜oz um.˜ Bu soruda »c˜oz ume daha kolay i»slemlerle ula»smak i»cin baz‡ de‚gi»s-˜ ken e‚gi»stirmeler yapmak yerinde

106 BOLUM 2. KONULAR

esitligini elde ederiz. Oyleyse bu esitlikleri ispatlamamız yeterli olacaktır.Varsayalımanalitik duzlem uzerinde, kose koordinatları (0, 0), (0, b), (a, 0), (a, b) olan birdikdortgenimiz olsun. Bu dikdortgen uzerinde (a−1)(b−1) tane latis (lattice)noktası, yani koordinat bilesnleri tamsayıar olan noktalar vardır. Bu dikdort-geni y = xb

a dogrusu ile iki parcaya ayıralım. Bu dogru uzerindeki baslangıc vebitis noktaları haric hicbir noktanın koordinatlarının tamsayı olmadıgı acıktır.Eger boyle bir nokta varsa, mesela (m,n), 0 < m < a, 0 < n < b, n

m = ba

olmalıdır. Demek ki nm kesri, b

a sadelesebilen kesrinin bir sadelesmi halidir.Ancak bu durum acık bir celiskidir. Cunku (a, b) = 1 kabul etmistik.Lk = (k, kba ), 1 ≤ k ≤ a − 1 oktalarının her biri dogrunu uzerindedir. Buna

gore [kba ] ifadesi (k, 0) dan (k, kba ) noktasına giden dogrunun uzerindeki latisnoktalarıdır. Yani

a−1∑

k=1

[kb

a]

ifadesi dikdortgenin alt yarısındaki latis noktalarının sayısıdır. Benzer bicimde,

b−1∑

k=1

[ka

b]

ifadeside ust yarıdaki noktalar olduguna gore ve bu noktalar alt ve ust yarıdaesit sayıda bulunabildigine gore toplam nokta sayısının yarısı

(a− 1)(b− 1)

2

sitenen cevap olur. ispat tamamlanır.

Ornek. n ∈ Z+ olmak uzere, [√n +

√n+ 1] = [

√4n+ 2] esitligini

kanıtlayınız.

Cozum. Ifadenin karesini alırsak√n +

√n+ 1 toplamının

√4n+ 1 ile√

4n+ 3 ifadelerinin arasında oldugunu gormek zor degildir. Tam kare ifadelermod4 altında 0 veya 1 olduklarından 4n + 2 ve 4n + 3 ifadelerinin birer tamkare olmadıkları acıktır. Buna gore [

√4n+ 2] = [

√4n+ 3] oldugu acıktır.

Teorem.(De - Polignac ve Legendre Formulu) n! ifadesini bolen p asal

Page 108: MATEMATİK OLİMPİYATI ÇALIŞMA KİTAPÇIĞI · C»˜oz um.˜ Bu soruda »c˜oz ume daha kolay i»slemlerle ula»smak i»cin baz‡ de‚gi»s-˜ ken e‚gi»stirmeler yapmak yerinde

2.11. TAMDEGER FONKSIYON PROBLEMLERI 107

sayısının en buyuk kuvveti α ise

α =∞∑

k=1

[n

pk]

esitligi vardır.

Ornek. 999! ifadesinin ondalık yazılımının sonunda kac tane 0 vardır?

Cozum. Sıfırların sayısı carpımdaki 2 ve 5 sayılarına baglıdır buna gore5α|999! olmasını saglayan en buyuk α degeri istenen cevap olacaktır. Bunagore;

[999

5] + [

999

52] + [

999

53][999

54] = 179 + 39 + 7 + 1 = 246 olur.

Ornek.

7|(1000

500

)

ifadesi dogrulugunu arastırınız.

Cozum. 7α|1000! olmasını saglayan en buyuk α degeri‘;

[1000

7] + [

1000

72] + [

1000

73] = 164

olacaktır. Benzer bicimde 7α|500! olmasını saglayan en buyuk α degeride 82bulunur. (

1000

500

)=

1000!

(500!)2

oldugundan, bu ifadeyi bolen 7 nin en buyuk kuvveti 164−2 ·82 = 0 oldugun-dan, soruda verilen ifade dogru degildir.

2.11.1 Calısma Soruları

1.[

3√1] + [

3√2] + . . .+ [

3√x3 − 1] = 400

denkleminin dogal sayılardaki cozum kumesinin elemanlarını bulunuz.

Page 109: MATEMATİK OLİMPİYATI ÇALIŞMA KİTAPÇIĞI · C»˜oz um.˜ Bu soruda »c˜oz ume daha kolay i»slemlerle ula»smak i»cin baz‡ de‚gi»s-˜ ken e‚gi»stirmeler yapmak yerinde

108 BOLUM 2. KONULAR

2. (6+√35)1980 acılımındaki virgulden sonraki ilk 1000 basamagın 9 oldu-

gunu gosteriniz.

3. [x2] = [x] denkleminin cozum kumesini bulunuz.

4.8

15<

n

n+ k<

7

13

esitsizligini saglayan en buyuk n degeri kactır? (k degeri tektir).

5. r ∈ R olmak uzere;91∑

k=19

[r +k

100] = 546

ise [100r] ifadesinin esitini bulunuz.

6. f(n) fonksiyonu n1/4 sayısına en yakın tamsayıyı temsil etmek uzere,

1995∑

n=1

1

f(n)

toplamının tamdegerini bulunuz.

7. [x] ifadesi x sayısının tamdegerini temsil etmek uzere

[12

1998], [

22

1998], . . . , [

19972

1998]

dizisindeki farklı tamsayıların sayısını bulunuz.

2.11.2 Cozumler

1. x ∈ N olduguna gore;

3√1, . . . ,

3√7 → 1 · 7 = 7

3√8, . . . ,

3√26 → 2 · 19 = 38

3√27, . . . ,

3√63 → 3 · 37 = 111

3√64, . . . ,

3√124 → 4 · 61 = 244

ise 7 + 38 + 111 + 244 = 400 ise tek cozum x = 5 olmalıdır.

Page 110: MATEMATİK OLİMPİYATI ÇALIŞMA KİTAPÇIĞI · C»˜oz um.˜ Bu soruda »c˜oz ume daha kolay i»slemlerle ula»smak i»cin baz‡ de‚gi»s-˜ ken e‚gi»stirmeler yapmak yerinde

2.11. TAMDEGER FONKSIYON PROBLEMLERI 109

2. (6 +√35)1980 + (6 − √

35)1980 = 2k, k ∈ Z olur. Burada 6 − √35 <

110 olmalıdır aksi durumda yani eger 1

10 < 6 − √35 olursa, iki tarafın

karesinden 3500 < 3481 olur.

0 < (6−√35)1980 < 10−1980 ⇒ 2k − 1

101980< (6 +

√35)1980 < 2k

⇒ 2k − 1 + 0, 999 . . . 9︸ ︷︷ ︸1979 tane 9

< (6 +√35)1980 < 2k

3. x2 ≥ 0 ise [x2 ≥] olur. x ∈ [0, 1] aralıgındaki tum sayıların sagladıgıacıktır. Biz (1, 2) aralıgına bakalım;x = 1 + {x}, [(1 + {x}2)] = [1 + {x}] ise [1 + 2{x}+ {x}2] = [1 + {x}][1 + 2{x} + {x}2] = 1 0 ≤ 2{x} + {x}2 < 1 ⇒ {x} > 0 veya {x}2 +2{x} − 1 < 0 durumları olusur. Buradan eger y = {x} olarak alırsaky ∈ (0,

√2−1) olmalıdır. Buradan da, x ∈ (0,

√2) arasında olur. 0 zaten

ahil oldugundan istenilen aralık x ∈ [0,√2] bulunur.

4. Verilen ifadeyi duzenlersek

6n

7< k <

7n

8

olacaktır. Fakat buradan 7n8 − 6n

7 = n56 bulunur. Buna gore eger n > 112

ise, bu aralıkta kesinlikle, yani (6n7 , 7n8 ) aralıgında, iki tamsayı olacaktır.n = 112 icin ise, 96 < k < 98 olacagından cevap 112 olmalıdır.

5. 546 = 7 · 73 + 35 = 38 · 7 + 35 · 8 olur. Demek ki

7 = [r + 0, 19] = [r + 0, 20] = [r + 0, 21] = . . . [r + 0, 56]

8 = [r + 0, 57] = [r + 0, 58] = . . . = [r + 0, 91]

olacagından 7, 43 ≤ x < 7, 44 ise 74, 3 ≤ 100r < 74, 4 ve [100r] = 74olur.

6. f(k) ifadesi 1, 2, 3, . . . , 7 degerlerinden birini alır. Sınır degerler

(n+1

2)4 = n4 + 2n3 +

1

2· (3n2 + n) +

1

16

Page 111: MATEMATİK OLİMPİYATI ÇALIŞMA KİTAPÇIĞI · C»˜oz um.˜ Bu soruda »c˜oz ume daha kolay i»slemlerle ula»smak i»cin baz‡ de‚gi»s-˜ ken e‚gi»stirmeler yapmak yerinde

110 BOLUM 2. KONULAR

olduguna gore

5 tanesi : 1

34 tanesi : 1/2

111 tanesi : 1/3

260 tanesi : 1/4

505 tanesi : 1/5

870 tanesi : 1/6

210 tanesi : 1/7

olacagına gore, toplam 400 olur.

7. [9982

1998 ] = 498 < 499 = [9992

1998 ] olduguna gore once k = 1, 2, 3, . . . , 998 vek = 999, 1000, . . . , 1997 durumlarını ayrı ayrı inceleyelim. Buna gore,k = 1, 2, 3, . . . , 997 icin

(k + 1)2

1998− k2

1998=

2k + 1

1998< 1

olacaktır. Buna gore

[12

1998] = 0, 1, 2, . . . , 498 = [

9982

1998]

dizisinde her bir sayı en az bir defa tekrarlanacaktır. Buna gore dizininbu bolumunde 499 terim vardır. k = 999, 1000, . . . , 1996 ise

(k + 1)2

1998− k2

1998=

2k + 1

1998> 1

ise burada da, 1997−999+1 = 999 farklı terim vardır. Buna gore dizidekifarklı tamsayıların sayısı 1498 olacaktır.

2.12 Bolunebılme ve Asal Sayılar

Tanım. a 6= 0, b ∈ Z olmak uzere, eger a · c = b esitligini saglayan birc ∈ Z bulunuyorsa bu duruma a boler b denir ve a|b olarak yazılır.

Page 112: MATEMATİK OLİMPİYATI ÇALIŞMA KİTAPÇIĞI · C»˜oz um.˜ Bu soruda »c˜oz ume daha kolay i»slemlerle ula»smak i»cin baz‡ de‚gi»s-˜ ken e‚gi»stirmeler yapmak yerinde

2.12. BOLUNEBILME VE ASAL SAYILAR 111

Teorem.

1. a, b, c,m, n ∈ Z olmak uzere c|a, c|b ise c|(a ·m+ b · n) dir.2. x, y, z ∈ Z olmsk uzere; x|y, y|z ise x|z dir.

Kanıt.

1. s, t ∈ Z olmak uzere varsayalım s · c = a, t · c = b olsun. Buna goream+ bn = c(sm+ tn) ise c|(am+ bn) olur.

2. Ve varsayalım u, v ∈ Z olmak uzere xu = y, yv = z olsun. Buna gorexuv = z ise x|z oldugu acıktır. Ayrıca eger a|b ve b 6= 0 ise ≤ |a| < |b|durumu vardır.

Ornek. (n + 1)|(n2 + 1) durumunu dogrulayan tum n ∈ Z+ degerlerinibulunuz.

Cozum. n2 + 1 = n2 − 1 + 2 = (n− 1) · (n+ 1) + 1 ise (n− 1) · (n+ 1)sayısı (n + 1) ise bolunur. Bu durumda (n + 1)|2 olmalıdır. |n + 1| < |2| isen = 1 olur.

Ornek. n herhangi bir dogal sayı olmak uzere; n3 + 2n sayısının 3 ilebolunebildigini gosteriniz.

Cozum. n degeri 3k + 1, 3k + 2 ve 3k formunda olabilir.n = 3k ise (3k)3 + 2 · (3k) = 3 · (32 · k3) + 3 · (2k) ise 3|n3 + 2nn = 3k+1 ise (3k+1)3+2·(3k+1) = (3k+1)3+2·3k+2 ≡ 1+0+2 ≡ 0(mod3)ise 3|n3 + 2nn = 3k+2 ise (3k+2)3+2·(3k+2) = (3k+2)3+2·3k+4 ≡ 2+0+1 ≡ 0(mod3)ise 3|n3 + 2n.

Ornek. 7|3x+ 2 ise 7|(15x2 − 11x− 14) oldugunu kanıtlayınız.

Cozum. 15x2 − 11x − 14 = (3x + 2) · (5x − 7) = 7s · (5x − 7) ise kanıttamamlanmıs olur.

Page 113: MATEMATİK OLİMPİYATI ÇALIŞMA KİTAPÇIĞI · C»˜oz um.˜ Bu soruda »c˜oz ume daha kolay i»slemlerle ula»smak i»cin baz‡ de‚gi»s-˜ ken e‚gi»stirmeler yapmak yerinde

112 BOLUM 2. KONULAR

Ornek. a, b, c dogal sayılar olmak uzere, a2 + b2 = c2 esitligi saglanmak-tadır. Buna gore, a, b ve c den en az birinin 3 ile bolunebildigini gosterelim.

Cozum. Eger a, b ve c degerleri icin bir genellme yaparsak, varsayalımhicbiri 3 ile tam bolunmesin. Oyleyse, a = 3k+1, b = 3t+1, c = 3r+1 olsun.Buna gore,

(3k + 1)2 + (3t+ 1)2 ≡ 1 + 1(mod3)

ise c2 ≡ (mod3) olur, ancak tamsayının karesi bu denkligi saglamaz.

Teorem. Ardısık n tamsayının carpımı n! ile tam bolunur.Kanıt. Varsayalım ardısık n tamsayımız m+ 1,m+ 2,m+ 3, . . . ,m+ n for-munda olsun ve bu dizinin her bir elemanı pozitif tamsayı olsun. Buna gore,gostermemiz gereken durum;

n!|m+ 1,m+ 2,m+ 3, . . . ,m+ n

dir. Burada binom katsayıların tamsayı olma ozelligini kullanırsak

(m+ n

n

)=

(m+ n)!

(m+ n−m)!(m+ n− n)!=

(m+ n)!

n!m!=

(m+ 1) · (m+ 2) · (m+ 3) · . . . · (m+ n)

n!∈ Z

ise ispat tamamlanır.

Ornek. 6|n3 − n oldugunu gosteriniz.

Cozum. n3 − n = n · (n2 − 1) = n · (n− 1) · (n+1) ise daha once verilenteoremden dolayı dogrudur.

Teorem. n ∈ Z+ olmak uzere, n bir bilesik sayı13 ise, n sayısının p ≤ √n

olacak sekilde bir asal carpanı vardır.Kanıt. Varsayalım n = a · b, 1 < a ≤ b olsun. Eger bu iki degerde

√n de

buyukse, n = a · b =√n · √n = n olur ki, bu bir celiskidir. Demek ki, n

sayısının 1 den farklı ve√n den kucuk esit olan bir carpanı vardır. Buna gore,

p bir asal carpan olmak uzere, bu degerde√n den kucuk olur.

132 den buyuk olan ve asal olmayan sayılar

Page 114: MATEMATİK OLİMPİYATI ÇALIŞMA KİTAPÇIĞI · C»˜oz um.˜ Bu soruda »c˜oz ume daha kolay i»slemlerle ula»smak i»cin baz‡ de‚gi»s-˜ ken e‚gi»stirmeler yapmak yerinde

2.12. BOLUNEBILME VE ASAL SAYILAR 113

Ornek. 101 sayısının asal olup olmadıgını anlamak icin bu sayıyı en azkac sayıya bolmeliyiz.

Cozum.√101 ≈ 10 ise p = 2, 3, 5, 7 olmalıdır. Toplamda 4 sayı yeterlidir.

Ornek. Elemanlarından hicbirisi asal sayı olmayan gelisiguzel uzunluktabir ardısık sayı dizisi bulunabilir mi?

Cozum. k ∈ Z, k ≥ 2 olmak uzere

k! + 2, k! + 3, . . . , k! + k

dizisinin elemanları ardısıktır ve asal degildir. Yani

A = (n+ 1)! + 2, (n+ 1)! + 3, . . . , (n+ 1)! + (n+ 1)

dizisini ele alalım. 2 ≤ k < n + 1 bagıntısını saglayan herbir k tamsayısı icink|(n+ 1)! dir. k|(n+ 1)! + k ise ispat tamamlanır. Mesela

23! + 1, 23! + 2, . . . , 23! + 28

dizisindeki sayılardan hicbirisi asal degildir.

Not. p > 3 olmak uzere butun asal sayılar, n ∈ Z olmak uzere p = 6n ± 1formundadır.

Ornek. p ve p2 +2 asal sayılar ise p3 +2 ninde asal oldugunu gosteriniz.

Cozum. p degeri tek olmalıdır. p = 3 icin p2 + 2 = 11 ve p3 + 2 = 29dur. Burada p > 3 ise p = 6n± 1 olarak alınırsa, p2+2 sayısı 3 ile bolunebilir.Demek ki, p = 3 dısında soruda verilen durumu dogrulayan bir sayı yoktur.Tek cozum, p = 3 olur.

Ornek. 19 dan 92 ye kadar olan sayılar yanyana yazılarak elde edilen,

19202122 . . . 9192

sayısını bolen 3 un en buyuk kuvvetini bulunuz.

Page 115: MATEMATİK OLİMPİYATI ÇALIŞMA KİTAPÇIĞI · C»˜oz um.˜ Bu soruda »c˜oz ume daha kolay i»slemlerle ula»smak i»cin baz‡ de‚gi»s-˜ ken e‚gi»stirmeler yapmak yerinde

114 BOLUM 2. KONULAR

Cozum. 9 ile bolubeilme kuralına bakalım 19+20+21+ . . .+92 = 372 ·3ise bu sayı 9 ile bolunemez, sadece 3 ile bolunebilir.

Ornek.

p3 + p2 + 11p+ 2

ifadesini asal yapan en buyuk p asal sayısını bulunuz.

Cozum. p = 3 icin, ifademiz 71 degerini alır. 3 ten buyuk degerler icinyani p = 3k+ 1 ve p = 3k+ 2 degerlerinde ifademiz sırasıyla 3 · (9k3 + 12k2 +16k+5) ve 9 · (3k3+7k2+9k+4) degerlerini alır. Ancak bu degerlerin ikisideasal degildir. Demek ki tek deger p = 3 tur.

Ornek. 2a+1 ve 3a+1 birer tamkare ise 5a+3 sayısının asal olmadıgınıgosteriniz.

Cozum. 2a+ 1 = x2 ve 3a+ 1 = y2 ise

5a+ 3 = 4 · (2a+ 1)− (3a+ 1) = 4x2 − y2 = (2x− y)(2x+ y)

durumu vardır. 5a+3 asal ise, 2x−y = 1 ve 2x+y = 5a+3 olmalı. 4x = 5a+3ise 5x2−8x+3 = 0 olur. Burada bulunan her bir kok tamsayı olmak zorundadır.Buradan istenen tamsayı x degeri −1 bulunur. Buna gore, sadece (1, 1, 3) icindurum saglanır diger durumlarda saglanmaz.

2.12.1 Calısma Soruları

1. n ∈ Z+ olmak uzere,

(n+ 10)|(n3 + 100)

olmasını saglayan en buyuk n tamsayı degerini bulunuz.

2. n5−5n3+4n iadesinin daima 120 ile kalansız bolunebilecegini gosteriniz.

3. p > 3 ve p asal bir sayı olmak uzere, 24|(p2 − 1) oldugunu gosteriniz.

4. Herhangi bir tamsayının karesinin 4k veya 4k + 1 formunda oldugunugosteriniz.

Page 116: MATEMATİK OLİMPİYATI ÇALIŞMA KİTAPÇIĞI · C»˜oz um.˜ Bu soruda »c˜oz ume daha kolay i»slemlerle ula»smak i»cin baz‡ de‚gi»s-˜ ken e‚gi»stirmeler yapmak yerinde

2.12. BOLUNEBILME VE ASAL SAYILAR 115

5. 11, 111, 1111, 11111, . . . dizisindeki sayılardan hicbirinin tamkare olmadı-gını kanıtlayınız.

6. Bir tamsayının 5. kuvvetine esit olan 6p + 1 formundaki tum p asalsayılarını bulunuz.

7. 21 · p+ 1 sayısını tamkare yapan kac p asal sayısı vardır?

8. 105 sayısı bir kac ardısık dogal sayının toplamı olarak kac farklı bicimdeyazılabilir?

9. Kac tane p asal sayısı icin p2 + 17 sayısının 4 tane farklı pozitif bolenivardır?

10. n > 3 olmak uzere, (n− 3)|n2 − n durumunu dogrulayan butun n tam-sayılarını bulunuz.

11. {1, 2, 3, 4, . . . , 23, 24, 25} kumesinden en az kac eleman atılmalı ki geriyekalan elemenların carpımı bir tam kareye esit olsun.

12.√13p+ 289 sayısının bir tamsayı olmasını saglayan en buyuk p asal sa-

yısını bulunuz.

13. a. N sayısı b tabanında 777 dir. N nin bir tamsayının dorduncu kuv-vetine esit olmasını saglayan en kucuk b tamsayısını bulunuz.

b. n bir tamsayı olsun. n2 nin onlar basamagındaki rakam 7 ise, n2

nin birler basamagındaki rakamı bulunuz.

14. n+1 ve 16n+1 ifadelerinin ikisinide tamkare yapan n ≥ 1 tamsayılarınınsayısını bulunuz.

15. (n + 1) · (n4 + 2n) + 3 · (n3 + 57) ifadesinin (n2 + 2) ile bolunmesinisaglayan en buyuk n tamsayısının degerini bulunuz. (ipucu: polinombolmesini kullanınız)

16.49, 4489, 444889, 44448889, . . . , 44 . . . 44︸ ︷︷ ︸

n tane 4

88 . . . 88︸ ︷︷ ︸n-1 tane 8

9, . . .

dizisindeki her bir terimin tamkare oldugunu kanıtlayınız.

Page 117: MATEMATİK OLİMPİYATI ÇALIŞMA KİTAPÇIĞI · C»˜oz um.˜ Bu soruda »c˜oz ume daha kolay i»slemlerle ula»smak i»cin baz‡ de‚gi»s-˜ ken e‚gi»stirmeler yapmak yerinde

116 BOLUM 2. KONULAR

17. 111 . . . 11︸ ︷︷ ︸221 tane 1

sayısının bilesik sayı oldugunu gosteriniz.(ipucu: 221 = 17·13)

18. a = 1111 . . . 1111︸ ︷︷ ︸m tane 1

, b = 1000 . . . 000︸ ︷︷ ︸m-1 tane 0

5 ise a · b + 1 sayısının bir tamkare

oldugunu gosteriniz.

19.

111 . . . 111︸ ︷︷ ︸2n tane 1

− 222 . . . 222︸ ︷︷ ︸n tane 2

farkının bir tamkare oldugunu gosteriniz.

20. 2x2−x−36 ifadesini bir asal sayının karesi yapan butun x tamsayılarınıbulunuz.

21. En az iki ardısık pozitif tamsayının toplamı seklinde yazılabilen pozitifbir tamsayının 2 nin kuvvetine esit olamayacagını gosteriniz.

22. 1 den 100 e kadar (100 dahil) sayılardan kac tanesi, iki veya daha fazlaardısık pozitif tamsayının toplamı olarak yazılabilir?

23. Ilk terimi n olan 98 ardısık sayının toplamı 192 ile bolunmektedir. Bunagore en kucuk n ∈ Z degerini bulunuz.

24. a · b = c · d esitligini saglayan a, b, c, d > 0 tamsayıları verildiginde a2 +b2 + c2 + d2 toplamının hic bir zaman asal olamayacagını gosteriniz.

25. 2903n−803n−464n+261n ifadesinin n in alabilecegi herhani bir tamsayıdegeri icin 1897 ile bolunebilecegini gosteriniz.

26. 1492n − 1770n − 1863n + 2141n ifadesinin n nin alabilecegi herhangi birtamsayı degeri icin 1946 ile bolunebilecegini gosteriniz.

2.12.2 Cozumler

1. Eger polinom bolmesi ozeligini kullanırsak elimizde

n3 + 100 = (n+ 10)(n2 − 10n+ 100)− 900

Page 118: MATEMATİK OLİMPİYATI ÇALIŞMA KİTAPÇIĞI · C»˜oz um.˜ Bu soruda »c˜oz ume daha kolay i»slemlerle ula»smak i»cin baz‡ de‚gi»s-˜ ken e‚gi»stirmeler yapmak yerinde

2.12. BOLUNEBILME VE ASAL SAYILAR 117

esitligi olusur. Ancak bolme isleminin kalansız olabilmesi icin bolme is-leminde kalanın sıfır olması gerekir. Demek ki,

n+ 10|900

olmalıdır. n+ 10 ≤ 900 ise n degerinin en buyuk degeri 890 olur.

2. n(n4−5n2+4) = (n2−1) ·(n2−1) ·n = (n−2) ·(n+2) ·(n−1) ·(n+1) ·nise ifade ardısık 5 sayının carpımıdır ve 5! ile kalansız bolunur.

3. p > 3 ise p = 6k ± 1 formundadır. Buna gore,

(6k+1)2−12 = (6k+1−1)·(6k+1+1) = 6k·(6k+2) = 6k·2(3k+1) = 12k·(3k+1)

olur. k sayısı tek ise 3k + 1 = 2A, A ∈ Z⇒ 12k · (2A) = 24 ·A ·K olur.

4.

2a, 2a+ 1 ⇒ (2a)2 = 4a2

(2a+ 1)2 = 4a2 + 4a+ 1 = 4(a2 + a) + 1 olur.

5. Bir onceki ornekten dolayı tamkare olmadıgı acıktır. Cunku tamkare birifade 4k veya 4k + 1 formundadır.

6. (x)5 = (6p + 1) ise x sayısı da kesinlikle asaldır. x = 3 ise x5 = 243olur. 240 = 6 · 40 + 3 ise x = 3 olamaz. 6p = x5 − 1 = (x − 1) ·( x4 + x3 + x2 + x+ 1︸ ︷︷ ︸Bu kısım x=2 icin 6 dan buyuk olur.

). Demek ki (x − 1) de 6 nın carpanı ise

x = 2, 3, 4 veya 7 olur. Bunlardan sadece 7 saglar. 75 = 6 · 2801+ 1 olur.

7.

21 · p+ 1 = x2

21 · p = x2 − 1

21 · p = (x− 1) · (x+ 1) ⇒ (x+ 1)− (x− 1) = 2 olmalıdır.

Yani, p = 23 veya p = 19 olabilir.3 · 7 · p = (x − 1) · (x + 1) ⇒ p = 3olabilir.

Page 119: MATEMATİK OLİMPİYATI ÇALIŞMA KİTAPÇIĞI · C»˜oz um.˜ Bu soruda »c˜oz ume daha kolay i»slemlerle ula»smak i»cin baz‡ de‚gi»s-˜ ken e‚gi»stirmeler yapmak yerinde

118 BOLUM 2. KONULAR

8. n = k + (k + 1) + (k + 2) + . . .+ (k +m) olarak secelim. Buna gore

n =(m+ 1)

m+ 2k

1

2

olur. Burada, (2k+m)− (m+1) = 2k− 1 ifadesi tek sayı olduguna gore

(m+ 1) · (m+ 2k) = 210

sayısını veren

(105, 2); (70, 3); (42, 5); (35, 6); (30, 7); (21, 10); (15, 14)

biciminde 7 sayı vardır.

9. p2+17 sayısında p asal ise p = 2 icin 4+17 = 21 sayısının 4 farklı bolenive p = 3 icin p2+17 = 26 = 2 ·13 ise 4 tane bolende burada vardır. p ≥ 3ise p2+17 = x ve p2−1 = (x−18) olduguna gore, (p−1)(p+1) = x−18olur. Burada (p− 1)(p+ 1), 3 ile tam bolunur. Buna gore;

(p− 1)(p+ 1) ≡ 0 ≡ x− 18 ≡ x(mod3) ise 3|x durumu vardır.

(p− 1)(p+ 1) ≡ 0 ≡ x− 18 ≡ x(mod2) ise 2|x durumu vardır.

x = a ·b veya x = c3 olabilir. x = 2 ·3 = 6 olmalıdır. x = 6 ise p2+17 = 6durumu olmaz. Demek ki sadece 2 ve 3 icin 4 asal boleni vardır.

10. Eger dogrulayan n degerlerini bulmak istiyorsak, ifademizi

n2 − n = (n− 3) · (n+ 2) + 6

olarak yazabiliriz. Tam bolunebilme olabilmesi icin (n − 3)|6 olmalıdır.Buna gore n degerleri, {4, 5, 6, 9} olmalıdır.

11. Varsayalım

A = 25! = 2a1 · 3a2 · 5a3 · 7a4 · 11a5 · 13a6 · 17a7 · 19a8 · 23a9

olsun. Burada, a1 = 22, a2 = 10, a3 = 6, a4 = 3, a5 = 2, a6 = 1, a7 =1, a8 = 1, a9 = 1 olacagından bu carpımdan cıkarılması gereken sayılar{23, 19, 17, 13, 7} olacaktır. Bu sayıların atılması yeterlidir.

Page 120: MATEMATİK OLİMPİYATI ÇALIŞMA KİTAPÇIĞI · C»˜oz um.˜ Bu soruda »c˜oz ume daha kolay i»slemlerle ula»smak i»cin baz‡ de‚gi»s-˜ ken e‚gi»stirmeler yapmak yerinde

2.12. BOLUNEBILME VE ASAL SAYILAR 119

12.√13p+ 289 = x, x ∈ Z+ olduguna gore, 13p + 289 = x2 ve 13p =

(x− 17) · (x+17) bulunur. p asal oldugundan boleni yoktur. x− 17 = 13ve x = 30 bulunur. Demek ki, 13 · p = 13 · 4 ise p = 47 bulunur.

13. a. N = (777)b = 7b0+7b1+7b2 = x4 ⇒ 7(b2+ b+1) = x4 esitligindenb2 + b + 1 = 73 · A olmalıdır. A = 1 olursa, b · (b + 1) = 73 − 1 iseburadan b = 18 bulunur.

b. Varsayalım n = 10x+y olsun. Buradan n2 = 100 · x2︸ ︷︷ ︸cift

+20 · xy︸ ︷︷ ︸cift

+ y2︸︷︷︸tek olmalı

.

Buna gore y2 = 36 yada 16 olabilir. Buna gore istenen cevap 6 ol-malıdır.

14. Varsayalım

x2 = n+ 1

y2 = 16n+ 1

olsun. Buna gore, y2 − x2 = (y − x) · (y+ x) = 15 · n ifadesinde n yerinex2−1 yazarsak yeni ifademiz (4x−y) · (4x+y) = 15 elde edilir. Buradanistenen tek deger n = 3 olur.

15. Bu sorunun cozumu okuyucuya bırakılmıstır.

16.

44 . . . 44︸ ︷︷ ︸n tane 4

88 . . . 88︸ ︷︷ ︸n-1 tane 8

9 = 44 . . . 4 · 10n + 88 . . . 8 · 10 + 9

=4

9(10n − 1) +

8

9(10n−1 − 1) + 9

=4

9· 102n +

4

9· 10n +

1

9

=1

9· (2 · 10n + 1)2 = (

2 · 10n + 1

3)2 olur.

17. Bu sorunun cozumu okuyucuya bırakılmıstır.

18. Bu sorunun cozumu okuyucuya bırakılmıstır.

Page 121: MATEMATİK OLİMPİYATI ÇALIŞMA KİTAPÇIĞI · C»˜oz um.˜ Bu soruda »c˜oz ume daha kolay i»slemlerle ula»smak i»cin baz‡ de‚gi»s-˜ ken e‚gi»stirmeler yapmak yerinde

120 BOLUM 2. KONULAR

19. Bu sorunun cozumu okuyucuya bırakılmıstır.

20. 2x2 − x− 36 = p2 ise (x+ 4) · (2x− 9) = p2 olacaktır. Buna gore;

i.

x+ 4 = p2

2x− 9 = 1 ⇒ x = 5, p = 3

ii.

x+ 4 = p

2x− 9 = p ⇒ x = 13, p = 17

iii.

x+ 4 = 1

2x− 9 = p2 ⇒ x = −3, p2 = −15 olamaz.

iv.

x+ 4 = −p2

2x− 9 = −1 ⇒ x = 4, p2 = −8 olamaz.

v.

x+ 4 = −p

2x− 9 = −p ⇒ x = 13, p = −17 olamaz.

vi.

x+ 4 = −1

2x− 9 = −p2 ⇒ x = −5, p = 19 olamaz.

21. Varsayalım ardısık sayılarımız,

m, (m+ 1), (m+ 2), . . . , (m+ k)

Page 122: MATEMATİK OLİMPİYATI ÇALIŞMA KİTAPÇIĞI · C»˜oz um.˜ Bu soruda »c˜oz ume daha kolay i»slemlerle ula»smak i»cin baz‡ de‚gi»s-˜ ken e‚gi»stirmeler yapmak yerinde

2.12. BOLUNEBILME VE ASAL SAYILAR 121

ise Bu sayıların toplamı,

(k + 1)(2m+ k)1

2

olacaktır. Eger bu toplam 2 nin bir kuvvetine esit ise, u ∈ Z icin,

2u = (k + 1)(2m+ k)1

2⇒ 2u+1 = (k + 1)(2m+ k)

esitligi ele edilir. Buradan da,k tek ise, m cift olacaktır. Buna gore, (T + 1)(C + T ) = C · T →Tek sayı 6= 2u+1

k cift ise, m tek olacaktır. Buna gore, (C + 1)(C + C) = T · C →Tek sayı 6= 2u+1 olacaktır.

22. Onceki cozulen orneklerin sonucundan faydalanırsak, 2 nini kuvvetleri-nin yazılamayacagını goruruz. Eger ilk yuz sayıdan ikinin kuvvetlerinicıkarırsak, istenen sonuca ulasabiliriz.

23.

n+ (n+ 1) + (n+ 2) + · · ·+ (n+ 97) = 192 ·K98 · n+

97 · 982

= 192 ·K98 · n+ 97 · 49 = 192 ·K49 · (2n+ 97) = 192 ·K

(2n+ 97) = 192 = 361 ise n = 32 olur.

24. Varsayalım, a = p · q, b = r · s, c = p · r ve d = q · s olsun. Buna goresoruda verilen toplam

p2q2 + r2s2 + p2r2 + q2s2 = (p2 + s2)(q2 + r2)

elde edilirki buda cozum icin yeterlidir.

25. (x − y)|(xn − yn) oldugunu zaten biliyoruz. Buna gore, 2903n − 803n

ifadesi 2903− 803 = 2100 = 7 · 300 ve 261n − 464n ifadeside 261− 464 =−203 = −29 · 7 ile bolunebilir. Demek ki, verilen ifade 7 ile bolunebilir.

Page 123: MATEMATİK OLİMPİYATI ÇALIŞMA KİTAPÇIĞI · C»˜oz um.˜ Bu soruda »c˜oz ume daha kolay i»slemlerle ula»smak i»cin baz‡ de‚gi»s-˜ ken e‚gi»stirmeler yapmak yerinde

122 BOLUM 2. KONULAR

Benzer bicimde, 2903n − 464n ifadeside 2903 − 464 = 2439 = 9 · 271 ve−803n + 261n ifadeside −803 + 261 = −542 = −2 · 271 ise deekki, ifade271 ilede tam bolunur. Buna gore, ifade 7 ·271 = 1897 ilede tam bolunur.

26. Bu sorunun cozumu okuyucuya bırakılmıstır.

2.13 Trıgonometrıde Sonsuz Toplam ve Farklar

Aslında baslık olarak hernekadar cebirdeki sonsuz toplam ve farklarıanım-satsa da aynısı degildir. Ancak genel mantıkta benzesmeler ve hatta ortusme-ler oldugu soruların cozumleri yapılırken goze carpar.Ilerleyen orneklerde vecalısma sorularında sizde farkına varacaksınız ki, genel amacımız verilen birsonsuz toplamı fark formunda, yani

n∑

k=2

[F (k)− F (k − 1)]

formunda yazarak F (n) − F (1) sonucuna ulasmak olacaktır. Once nispetendaha kolay bir ornekle baslayalım.

Size tavsiyemiz bu bolumdeki sorulara calısmaya baslamadan evvel trigo-nometrik ozdesliklere bir goz atmanız yonunde olacaktır. Boylelikle konuyadaha hakim olabilirsiniz.

Ornek.n∑

k=1

cos kx

toplamını hesaplayınız.

Cozum. Egern∑

k=1

cos kx = A esitliginde esitligin iki tarafınıda 2 sin x2

ile carparsak, carpımdan toplama gecebilmek icin ozdeslikleri kullanabiliriz.

Page 124: MATEMATİK OLİMPİYATI ÇALIŞMA KİTAPÇIĞI · C»˜oz um.˜ Bu soruda »c˜oz ume daha kolay i»slemlerle ula»smak i»cin baz‡ de‚gi»s-˜ ken e‚gi»stirmeler yapmak yerinde

2.13. TRIGONOMETRIDE SONSUZ TOPLAM VE FARKLAR 123

Oyleki

2 sinx

2·A =

n∑

k=1

sinx

2cos kx

=

n∑

k=1

(sin

(k +

1

2

)x− sin

(k − 1

2

)x

)

= sin

(n+

1

2

)x− sin

x

2olacaktır.

Buna gore, bu esitlikten

A =sin

(n+ 1

2

)x

2 sin(x2

) − 1

2

bulunur.

Ikinci ornegimiz de tanjant toplamını kullanarak rahatlıkla cozume ulasa-bilecegimiz bir ornek.

Ornek.n∑

k=0

tan−1

(1

k2 + k + 1

)

toplamını hesaplayınız.(tan−1 x = arctanx)

Ornek. Tanjant fark formulu

tan (a− b) =tan a− tan b

1 + tan a tan b

olduguna gore

tan−1 u− tan−1 v = tan−1 u− v

1 + uv

olacaktır. Daha da basite indirgemek icin ak = tan−1 k olarak alalım. Bunagore,

tan (ak+1 − ak) =tan ak+1 − tan ak1 + tan ak+1 tan ak

=k + 1− k

1 + k (k + 1)=

1

k2 + k + 1olacaktır.

Page 125: MATEMATİK OLİMPİYATI ÇALIŞMA KİTAPÇIĞI · C»˜oz um.˜ Bu soruda »c˜oz ume daha kolay i»slemlerle ula»smak i»cin baz‡ de‚gi»s-˜ ken e‚gi»stirmeler yapmak yerinde

124 BOLUM 2. KONULAR

Oyleyse toplamımız,

n∑

k=0

tan−1 (tan (ak+1 − ak)) =n∑

k=0

(ak+1 − ak) = an+1 − a0

= tan−1 (n+ 1) olacaktır.

Benzer cozum bicimleri verilen calısma soruları uzerinde uygulanabilir. Bunagore sonsuz bir toplamı cozerken, ifadeyi farkların sonsuz toplamı bicimindeyazarak sadelestirmeler yapmak yerinde olacaktır.

2.13.1 Calısma Soruları

1.sinx

cosx+

sin 2x

cos2 x+ · · ·+ sinnx

cosn x= cotx− cos (n+ 1)x

sinx · cosn xesitligini kanıtlayınız.

2.1

cos 0 · cos 1 +1

cos 1 · cos 2 + · · ·+ 1

cos 88 · cos 89 =cos 1

sin2 1

esitligini kanıtlayınız.

3. n ∈ Z+ ve a ∈ R olmak uzere a/π bir rasyonel sayı olarak veriliyor. Bunagore,

1

cos a− cos 3a+

1

cos a− cos 5a+ · · ·+ 1

cos a− cos (2n+ 1) a

toplamının esitini bulunuz.

4.n∑

k=1

tan−1 1

2k2= tan−1

(n

n+ 1

)

esitligini kanıtlayınız.

Page 126: MATEMATİK OLİMPİYATI ÇALIŞMA KİTAPÇIĞI · C»˜oz um.˜ Bu soruda »c˜oz ume daha kolay i»slemlerle ula»smak i»cin baz‡ de‚gi»s-˜ ken e‚gi»stirmeler yapmak yerinde

2.13. TRIGONOMETRIDE SONSUZ TOPLAM VE FARKLAR 125

5. a 6= kπ, k ∈ Z olmak uzere,

∞∑

n=1

1

2ntan

a

2n

toplamının esitini bulunuz.

6. ∞∑

n=1

3n−1 sin3a

3n=

1

4(a− sin a)

esitligini kanıtlayınız.

7. n = 2, 4, 6, · · · , 180 olmak uzere verilen n · sinn◦ sayılarının aritmetikortalamasının cot 1◦ oldugunu kanıtlayınız.

8. m,n ∈ Z+ ve x 6= kπm olduguna gore,

1

sin 2x+

1

sin 4x+ · · ·+ 1

sin 2nx= cotx− cot 2nx

esitligini kanıtlayınız.

9.tan 1

cos 2+

tan 2

cos 4+ · · ·+ tan 2n

cos 2n+1

toplamını hesaplayınız.

10. Sıfırdan farklı her x reel sayısı icin,

∞∏

n=1

cosx

2n=

sinx

x

esitligini kanıtlayınız.

11. n > 1 ve n ∈ Z icin

cos2π

2n − 1· cos 4π

2n − 1· · · · · cos 2nπ

2n − 1=

1

2n

esitligini kanıtlayınız.

Page 127: MATEMATİK OLİMPİYATI ÇALIŞMA KİTAPÇIĞI · C»˜oz um.˜ Bu soruda »c˜oz ume daha kolay i»slemlerle ula»smak i»cin baz‡ de‚gi»s-˜ ken e‚gi»stirmeler yapmak yerinde

126 BOLUM 2. KONULAR

12.n∏

k=1

(1− tan2

2kπ

2n + 1

)

carpımının esitini bulunuz.

13. (1− cot 1◦) (1− cot 2◦) (1− cot 3◦) · · · (1− cot 44◦) carpımını hesaplayı-nız.

14.

(1

2+ cos

π

20

)(1

2+ cos

20

)(1

2+ cos

20

)(1

2+ cos

27π

20

)=

1

16

esitligini kanıtlayınız.

15. n ∈ Z+, x 6= 2k+1(π3 + lπ

)ve k = 1, 2, · · · , n, l isen∏

k=1

(1− 2 cos

x

2k

)

carpımının sonucunu bulunuz.

16.n∑

k=1

(1 + 2 cos

2π · 3k3n + 1

)= 1

esitligini kanıtlayınız.

2.13.2 Cozumler

1. sin kx ·sinx = cos kx ·cosx−cos (k + 1)x ise bu esitsizligin iki tarafınıdasinx · cosk x ile bolelim. Buradan

sin kx

cosk x=

cos kx

sinx · cosk−1 x− cos (k + 1)x

sinx · cosk x

Page 128: MATEMATİK OLİMPİYATI ÇALIŞMA KİTAPÇIĞI · C»˜oz um.˜ Bu soruda »c˜oz ume daha kolay i»slemlerle ula»smak i»cin baz‡ de‚gi»s-˜ ken e‚gi»stirmeler yapmak yerinde

2.13. TRIGONOMETRIDE SONSUZ TOPLAM VE FARKLAR 127

olacaktır. Buna gore,

=sinx

cosx+

sin 2x

cos2 x+

sin 3x

cos3 x+ · · ·+ sinnx

cosn x

=cosx

sinx− cos 2x

sinx · cosx +cos 2x

sinx · cosx − cos 3x

sinx · cos2 x + · · ·+ cosnx

sinx · cosn−1 x− cos (n+ 1)x

sinx · cosn x= cotx− cos (n+ 1)x

sinx · cosn xolarak bulunacaktır.

2. Esitligin iki tarafınıda sin 1 ile carparsak,

sin 1

cos 0 cos 1+

sin 1

cos 1 cos 2+ · · ·+ sin 1

cos 88 cos 89=

cos 1

sin 1

olacaktır. Buradan

sin (1− 0)

cos 1 cos 0+

sin (2− 1)

cos 2 cos 1+ · · ·+ sin (89− 88)

cos 89 cos 88= cot 1

esitligi elde edilir. Burada

sin (a− b)

cos a cos b= tan a− tan b

ise son yazdıgımız esitligin sol tarafı

89∑

k=1

[tan k − tan (k − 1)] = tan 89− tan 0 = cot 1

olacaktır.

3. Soruda verilen esitligi duzenlersek,

n∑

k=1

1

cos a− cos (2k + 1) a=

1

2

n∑

k=1

1

sin ka · sin (k + 1) a

esitligini elde ederiz. Bir onceki problemin cozumunde oldugu gibi, egersin a ile carparsak, toplam

1

2

n∑

k=1

sin ((k + 1) a− ka)

sin ka · sin (k + 1) a=

1

2

n∑

k=1

(cot ka− cot (k + 1) a)

=1

2(cot a− cot (n+ 1) a)

Page 129: MATEMATİK OLİMPİYATI ÇALIŞMA KİTAPÇIĞI · C»˜oz um.˜ Bu soruda »c˜oz ume daha kolay i»slemlerle ula»smak i»cin baz‡ de‚gi»s-˜ ken e‚gi»stirmeler yapmak yerinde

128 BOLUM 2. KONULAR

olacagından sorudaki toplamın esiti,

(cot a− cot (n+ 1) a)

2 sin a

olarak bulunur.

4. Eger soruda verilen esitligin sol tarafını duzenlesek

n∑

k=1

tan−1 1

2k2=

n∑

k=1

tan−1

((2k + 1)− (2k − 1)

1− (2k − 1) (2k + 1)

)

esitligini elde ederiz. Eger arktanjant icin fark formulunu kullanırsak

n∑

k=1

(tan−1 (2k + 1)− tan−1 (2k − 1)

)

olacaktır. Bu toplam da

tan−1 (2n+ 1)− tan−1 (1) = tan−1

(2n+ 1− 1

1 + 2n+ 1

)= tan−1

(n

n+ 1

)

olacaktır.

5.

tanx =1

tanx− 1− tan2 x

tanx=

1

tanx− 2

tan2 x= cotx− 2 cot 2x

olduguna gore,

1

2ntan

a

2n=

1

2ncot

a

2n− 1

2n−1cot

a

2n−1

olacaktır. Bu toplam da

limx→0

x cot ax = limx→0

cos axx

sin ax=

1

a

olacagından istenilen deger

1

a− cot a

olarak bulunur.

Page 130: MATEMATİK OLİMPİYATI ÇALIŞMA KİTAPÇIĞI · C»˜oz um.˜ Bu soruda »c˜oz ume daha kolay i»slemlerle ula»smak i»cin baz‡ de‚gi»s-˜ ken e‚gi»stirmeler yapmak yerinde

2.13. TRIGONOMETRIDE SONSUZ TOPLAM VE FARKLAR 129

6. sin 3x = 3 sinx− 4 sin3 x olduguna gore,

3n−1 sin3a

3n=

1

4

(3n sin

a

3n− 3n−1 sin

a

3n−1

)

olacaktır. Teleskopik toplamdan

1

4limn→∞

sin a3n

13n

− 1

4sin a =

1

4(a− sin a)

olacaktır.

7. Oncelikle,90∑

k=1

2k sin (2k) sin 1

ifadesini hesaplayalım. Carpımdan, farkların toplamına gidersek

90∑

k=1

k cos (2k − 1)− k cos (2k + 1) =90∑

k=1

(k − (k − 1)) cos (2k − 1)− 90 cos 181

=

90∑

k=1

cos (2k − 1) + 90 cos 1

olacaktır. cos(180−x) = − cosx olduguna gore, son toplamdaki eleman-lar ikili ikili sadlesecektir. Sonuc olarak geriye sadece 90 cos 1 kalacaktır.Burayıda sin 1 ile bolersek

90 cos 1

sin 1= cot 1

olacaktır.

8.

1

sin 2x=

2 cos2 x− (2 cos2 x− 1

)

sin 2x

=2 cos2 x

2 sinx cosx− 2 cos2 x− 1

sin 2x

=cosx

sinx− cos 2x

sin 2x= cotx− cot 2x

Page 131: MATEMATİK OLİMPİYATI ÇALIŞMA KİTAPÇIĞI · C»˜oz um.˜ Bu soruda »c˜oz ume daha kolay i»slemlerle ula»smak i»cin baz‡ de‚gi»s-˜ ken e‚gi»stirmeler yapmak yerinde

130 BOLUM 2. KONULAR

olduguna gore, soruda verilen esitligin sol tarafındaki teleskopik toplam,yine bir teleskopik farklar toplamına gideceginden, esiti

cotx− cot 2nx

olacaktır.

9.

tan a

cos 2a=

tan a(1 + tan2 a

)

1− tan2 a=

2 tan a− tan a(1− tan2 a

)

1− tan2 a

=2 tan a

1− tan2 a− tan a = tan 2a− tan a

olduguna gore soruda verilen toplam,

tan 2− tan 1 + tan 4− tan 2 + · · ·+ tan 2n+1 − tan 2n = tan 2n+1 − tan 1

olacaktır.

10. Sinus fonksiyonunda yarımacı formulunden,

cosu =sin 2u

2 sinu

olacaktır. Bundan dolayı,

∞∏

n=1

cosx

2n= lim

k→∞

k∏

n=1

cosx

2n= lim

k→∞

k∏

n=1

1

2

sin x2n−1

sin x2n

esitligini elde ederiz. Bu esitlige gore,

limk→∞

1

2ksinx

sin x2k

=sinx

xlimk→∞

x2k

sin x2k

=sinx

x

olacaktır.

11. Eger esitligin sol kısmını

2n sin2π

2n − 1

Page 132: MATEMATİK OLİMPİYATI ÇALIŞMA KİTAPÇIĞI · C»˜oz um.˜ Bu soruda »c˜oz ume daha kolay i»slemlerle ula»smak i»cin baz‡ de‚gi»s-˜ ken e‚gi»stirmeler yapmak yerinde

2.13. TRIGONOMETRIDE SONSUZ TOPLAM VE FARKLAR 131

ile carpıp sinus formulunu uygularsak

= 2n sin2π

2n − 1· cos 2π

2n − 1· cos 4π

2n − 1· · · · · cos 2nπ

2n − 1

= 2n−1 sin4π

2n − 1· cos 4π

2n − 1· · · · · cos 2nπ

2n − 1

= sin2n+1π

2n − 1= sin

(2π +

2n − 1

)= sin

(2π

2n − 1

)

olacaktır. Eger sonucu

2n sin

(2π

2n − 1

)

ile bolersek, istenen esitligi elde etmis oluruz.

12.

1− tan2 x =2 tanx

tan 2x

ise

1− tan22kπ

2n + 1= 2

tan 2kπ2n+1

tan 2k+1π2n+1

olacaktır. Buradanda

2ntan 2π

2n+1

tan 2n+1π2n+1

= 2ntan 2π

2n+1

tan(2π − 2π

2n+1

) = −2n

olacaktır.

13.

1− cotx =√2(cos

π

4− cos

π

4cotx

)=

√2sin (x− π/4)

sinx

olacagından,

(1− cot 1) (1− cot 2) · · · (1− cot 44) = 222sin (−44)

sin 1· sin (−43)

sin 2· · · · · sin (−1)

sin 44= 222

olacaktır.

Page 133: MATEMATİK OLİMPİYATI ÇALIŞMA KİTAPÇIĞI · C»˜oz um.˜ Bu soruda »c˜oz ume daha kolay i»slemlerle ula»smak i»cin baz‡ de‚gi»s-˜ ken e‚gi»stirmeler yapmak yerinde

132 BOLUM 2. KONULAR

14.cos 3x

cosx= 4 cos2 x− 3 = 2 (1 + cos 2x)− 3 = 2 cos 2x− 1

olacaktır. Dolayısıyla,

1

2+ cosx = −1

2

(cos

(32x+ 3π

2

)

cos(x2 + π

2

))

=1

2

sin 32x

sin x2

olacaktır. Buna gore soruda verilen carpım

=1

16· sin

3π40

sin π40

· sin9π40

sin 3π40

· sin27π40

sin 9π40

· sin81π40

sin 27π40

=1

16· sin

81π40

sin π40

=1

16

olacaktır.

15.

1− 2 cos a =1− 4 cos2 a

1 + 2 cos a=

1− 2 (1 + cos 2a)

1 + cos a

= −1 + 2 cos 2a

1 + cos a

ise sorumuzda ki sonsuz carpım

(−1)n · 1 + cosx

1 + 2 cos x2n

olacaktır.

16.

1 + 2 cos ak = 1 + 2(1− sin2 ak

)= 3− 4 sin2 ak

=sin 3aksin ak

ve buradanda,

ak =3k · 2π3n + 1

Page 134: MATEMATİK OLİMPİYATI ÇALIŞMA KİTAPÇIĞI · C»˜oz um.˜ Bu soruda »c˜oz ume daha kolay i»slemlerle ula»smak i»cin baz‡ de‚gi»s-˜ ken e‚gi»stirmeler yapmak yerinde

2.14. KUS, GUVERCIN, YUVA... 133

olacaktır. Buradan sonsuz carpımımız,

sin 3ansin a1

=sin 3n+1π

3n+1

sin 3π3n+1

olacaktır. Son yazılan kesrin payı

sin

(3π − 3π

3n + 1

)= sin

3n + 1

ise esitligin sonucunun 1 olacagı acıktır.

EAT SLEEP DO MATH

2.14 Kus, Guvercın, Yuva...

Guvercin Yuvası Ilkesi her nekadar isminde guvercinlere yer versede aslındabu guzel canlılarla uzaktan yakından bir alakası yoktur. Ancak temel prensibinkavranması icin verilen orneklerde hep bu canlıyla alakalı orneklemeler yapıl-mıstır. Peki nedir bu ilke? Acıklayalım. Eger (n+1) tane nesneyi (Guvercini),n tane kutu (Guvercin yuvası) arasından secersek, bu nesnelerden en az ikitanesi aynı kutudan gelmis olacaktır. Veya Istanbul ili sınırları icerisinde ya-sayan ve sac tellerinin sayısı aynı olan en az iki kisi bulabiriz. Yazdıgımız buiki onermede guvercin yuvası ilkesinin kavranması icin yeterlidir. Yada benzerbicimde, herhangi bir ormandan secilecek en az iki agacın yaprak sayılarınınaynı olmasını saglayan yeterli ve gerekli sartlar nelerdir? Sorusunun cevabıdayine aynı ilkeyle bulunacaktır. Simdi bu ilkenin sorular uzerindeki uygulama-sını gormek icin ornekleri inceleyelim.

Ornek. 1, 2, 3, · · · , 99, 100 sayıları arasında 51 tane sayı seciliyor. Bunagore bu 51 sayıdan iki tanesinin aralarında asal oldugunu kanıtlayınız.

Page 135: MATEMATİK OLİMPİYATI ÇALIŞMA KİTAPÇIĞI · C»˜oz um.˜ Bu soruda »c˜oz ume daha kolay i»slemlerle ula»smak i»cin baz‡ de‚gi»s-˜ ken e‚gi»stirmeler yapmak yerinde

134 BOLUM 2. KONULAR

Cozum. Varsayalım bu 100 sayı arasından

(1, 2), (3, 4), · · · , (99, 100)

seklinde ikililer olusturalım. 51 tane sayı secilecegine gore, guvercin yuvasıilkesi uyarınca bu sayılar arasında (k, k + 1) ikilisi bulunacaktır. Buna gorek + 1 ve k sayılarını bolen p asalı varsa, bu asal (k + 1) − k = 1 sayısınıda bolecektir ki, bu durum celiski olur. Demek ki, (k, k + 1) sayılarının ortakboleni yoktur.

Ornek. 1, 2, 3, · · · , 99, 100 sayıları arasından 51 tane sayı seciliyor. Bunagore, bu 51 sayı arasında ki sayılardan birinin, bir diger sayıyı bolecegini gos-teriniz.

Cozum. 50 tek sayıyı, 1, 3, 5, · · · , 99 sayılarını alalım. Her biri icinbir kutu dusunelim. Bu kutular hem sayının kendisini hem de sayı ile 2’ninkuvvetinin carpımını icersin. Buna gore ilk kutu 1, 2, 4, 8, 16, · · · , ikinci kutu3, 6, 12, 24, 48, · · · olarak gidecektir. Eger 51 sayı secersek guvercin yuvası il-kesine gore bu iki sayı mecburen aynı kutudan gelecektir. Yani sayılardan biri2nk iken digeri 2mk olacak ve aynı k carpanına sahip olacaklardır. Dolayısıylasayılardan biri digerini bolecektir.

Cozulen iki ornektede farkedeceginiz uzere, her nekadar soru icerisindebahsi gecmesede kutu formuyla cozume gidilmistir. Demek ki cozumdeki anah-tar basamak kutu prensibini kavramak olacaktır. Simdi asagıda verilen ornek-leri inceleyerek bu formu daha iyi kavramaya calısınız.

Ornek. Secilen herhangi dokuz faklı reel sayı arasında

0 <(a− b)

1 + ab<

√2− 1

esitsizligini saglayan bir (a, b) ikilisinin bulunabilecegini kanıtlayınız.

Cozum. Verilen esitsizligin ortasındaki ifadeyi incelersek bu ifadenintan(x− y) formulune benzedigini gorebiliriz. Yani ifade aslında

tan (x− y) =tanx− tan y

1 + tanx tan y

Page 136: MATEMATİK OLİMPİYATI ÇALIŞMA KİTAPÇIĞI · C»˜oz um.˜ Bu soruda »c˜oz ume daha kolay i»slemlerle ula»smak i»cin baz‡ de‚gi»s-˜ ken e‚gi»stirmeler yapmak yerinde

2.14. KUS, GUVERCIN, YUVA... 135

bicimindedir. Simdi artık sorumuza biraz daha trigonometrik yaklasalım. Bunagore,(−π/2, π/2] aralıgını 8 esit uzunlukta parcaya ayıralım. Bu parcaların

(−π

2,−3π

8

],

(−3π

8,−π

4

], · · · ,

4,3π

8

],

(3π

8,π

2

]

olacagı acıktır. Burada xi = arctan ai, i = 1, 2, · · · , 9 olacak bicimde sayılarsecelim. Guvercin yuvası prensibine gore, xi’lerden iki tanesi, biz bunlara xj vexk diyelim, xj > xk olacak bicimde ayırdıgımız 8 aralıktan birinde olacaktır.Buna gore

0 < xj − xk <π

8ve dolayısıyla

0 < tan (xj − xk) =aj − ak1 + ajak

< tan(π8

)=

√2− 1

olacaktır.

Ornek. Bir kenarı 1 birim olan bir karenin icine yerlestirilen koseleri kareuzerinde bulunan bir ucgenin karenin merkezini icermemesi isteniyor. Bunagore ucgenin bir kenarının 1 birimden kucuk olması gerektigini kanıtlayınız.

Cozum. Karenin merkezini C noktası olarak alıp, kareyi 4 esit parcayabolelim. Eger cizilen ucgenin C noktasını icermemesi isteniyorsa, ucgenin ko-selerinin komsu iki parcanın icerisinde olması gerekir. Eger ucgen olusturmakistiyorsak alacagımız 3 noktanında komsu kareler icerisinde olması gerekecek-tir. Ancak 3 noktadan 2 tanesini aynı kareden alacagımıza gore bu noktalararası uzaklık kesinlikle 1 birimden kucuk olacaktır.

2.14.1 Calısma Soruları

1. 1, 2, 3, · · · , 99, 100 sayıları arasından 11 tane sayı seciliyor. Buna gore,bu 11 sayının altkumelerinden en az iki tanesinin elemanları toplamınınaynı olacagını kanıtlayınız.

2. Varsayalım koordinatları birer tamsayı olan 9 nokta uc boyutlu koordi-nat duzleminde secilsin. Buna gore, bu noktaların birlestirilmesiyle olus-turulan dogru parcalarından secilen bir tanesinin ucuncu bir tamsayıkoordinatlı noktayı icerdigini kanıtlayınız.

Page 137: MATEMATİK OLİMPİYATI ÇALIŞMA KİTAPÇIĞI · C»˜oz um.˜ Bu soruda »c˜oz ume daha kolay i»slemlerle ula»smak i»cin baz‡ de‚gi»s-˜ ken e‚gi»stirmeler yapmak yerinde

136 BOLUM 2. KONULAR

3. Altı kisilik bir grup icinde ya herhangi uc kisinin birbirini tanıdıgını yadaen az uc kisinin birbirini tanımadıgını kanıtlayınız.

4. Secilecek herhangi bir 16 basamaklı dogal sayının ardısık basamaklarınıncarpımının birer tamkare olmasını saglayan en bir zincirin bulundugunukanıtlayınız. Ornegin

12 343︸︷︷︸36

21524716 91︸︷︷︸9

8 gibi.

2.14.2 Cozumler

1. Onbir elemanlı bir kumenin 211 − 2 = 2046 tane eleman sayısı 11’den azolan bos kumeden farklı altkumesi vardır. Bu alt kumelerden herhangibirinin elemanları toplamı en fazla

91 + 92 + 93 + · · ·+ 99 + 100 = 955

olacaktır. Buna gore, guvercin yuvası ilkesi uyarınca, elemanları toplamıbirbirine esit olan iki, tane alt kume bulunacaktır. Eger bu kumelerin or-tak elemanları varsa, bu elemanı iki kumedende cıkararak kesisimleri bosume olan ve elemanları toplamları aynı olan iki alt kume elde edebiliriz.

2. Dokuz nokta icin her bir (x, y, z) koordinatları ya tek yada cift olacaktır.Bu durumda 23 = 8 farklı teklik ciftlik durumu ortaya cıkar. Buna gorebu dokuz noktadan iki tanesi mecburen koordinat pariteleri esit nokta-lar olacaklardır. Ve dolayısıyla orta noktalarının koordinatlarıda birertamsayı olacaktır.

3. Once elimizdeki 6 kisiyi duzgun bir altıgenin koselerine yerlestirdigimizdusunelim. Eger iki kisi biribirini tanıyorsa bu iki koseyi birlestiren kose-gen kırmızıya boyansın, tanımıyorsa da maviye boyansın. Aslında soru-nun cozumu ile bizim altıgenin koseleriyle tum kenarları aynı renk olanbir ucgen elde edip edemeyecegimiz sorusunun cevabı ozdesitir. Herhangibir koseyi alalım, guvercin yuvası ilkesi uyarınca, bu koseden cıkan uckosegen aynı renk olsun. Bundan sonraki herbir kosegende rengi, ucgenolusturmayacak bicimde secmeye calıssak bile mutlaka ortaya her defa-sında kenarları aynı renk olan bir ucgen ortaya cıkacaktır.

Page 138: MATEMATİK OLİMPİYATI ÇALIŞMA KİTAPÇIĞI · C»˜oz um.˜ Bu soruda »c˜oz ume daha kolay i»slemlerle ula»smak i»cin baz‡ de‚gi»s-˜ ken e‚gi»stirmeler yapmak yerinde

2.15. USTEL DIYOFANT DENKLEMLERI 137

4. Varsayalım d1, d2, d3, · · · , d11 sayımızın basamakları olsun. Eger bu sayı-nın basamaklarından her hangi biri 0, 1, 4 veya 9 ise problemin cozumuacıktır. Varsayalım sayımızın basamakları 2, 3, 5, 6, 7 ve 8 sayılarındanolussun. Eger x0 = 1 ve xi degeride d1, d2, · · · , di i = 1, · · · , 16 sayıları-nın carpımı olsun. Her bir xi = 2pi · 3qi · 5ri · 7si olacaktır. Burada ustdegerler zaten ya tektik yada cift sayıladır. Dolayısıyla 24 = 16 farklı du-rum elde edilebilir. Guvercin yuvası ilkesi uyarınca pi, qi, ri, si degerlerionyedi xi’lerin ikisi icin, varsayalım bu ikililerde xj ve xk olsun, teklikciftlik bakımından aynı pariteye sahip olacaklardır. Buna gore

dj+1 · dj+2 · · · · · dk =xkxj

tamkare olacaktır.

2.15 Ustel Dıyofant Denklemlerı

Ustel diyofant denklemlerinin diger diyofant denklemlerinden farkları de-giskenlerin genelde ustel durumda bulunmasıdır. Genel bir cozum yontemleriolmamasına ragmen bazı cebirsel oyunlarla cozum yolları gelistirilebilir. Asa-gıda verilen ornekleri incelediginiz zaman tam olarak ne demek istedigimizidaha iyi anlayacaksınız. Ilk olarak kolay bir ornekle baslayalım.

Ornek. (x, y, z) ∈ Z+ olmak uzere verilen 3x + 4y = 5z denkleminisaglayan tum ucluleri bulunuz.

Cozum I. Eger soruda verilen esitlige (mod4) altında bakarsak 3x ≡1 (mod4) dekligini elde ederiz. Buna gore x = 2x1 olmalıdır. Benzer bicimde5z ≡ 1 (mod3) oldugundan z = 2z1 alabiliriz. Bunları denklemde yerine ko-yarsak

4y = (5z1 + 3x1) (5z1 − 3x1)

olacaktır. Buradan 5z1 + 3x1 = 2s ve 5z1 − 3x1 = 2t olmalıdır. s + t = 2y ves > t olacagı acıktır. Eger bu iki denklemi cozersek 5z1 = 2t−1

(2s−t + 1

)ve

3x1 = 2t−1(2s−t − 1

)bulunur. Bu iki esitlikte de sol kısımlar birer tek sayıdır.

Oyleyse t = 1 olmalıdır. Eger s − t = u dersek 3x1 = 2u − 1 ve 5z1 = 2u + 1olacaktır. Eger birinci denkleme (mod3) altında bakarsak u = 2u1bulunur.

Page 139: MATEMATİK OLİMPİYATI ÇALIŞMA KİTAPÇIĞI · C»˜oz um.˜ Bu soruda »c˜oz ume daha kolay i»slemlerle ula»smak i»cin baz‡ de‚gi»s-˜ ken e‚gi»stirmeler yapmak yerinde

138 BOLUM 2. KONULAR

Sag tarafı carpanlara ayırırsak 2u1 + 1 = 3α ve 2u1 − 1 = 3β esitlikleri eldeedilir. Buradan 3α − 3β = 2 olduguna gore α = 1 ve β = 0 olacaktır. Buradanu1 = 1 ve u2 = 2 oldugundan cozumde x = y = z = 2 olarak bulunur.

Cozum II. Bu sefer farklı bir yonden probleme yaklasacagız. Birincicozumde oldugu uzere x = 2x1 ve z = 2z1 esitliklerini kullanırsak,

(3x1)2 + (2y)2 = (5z1)2

esitligini elde ederiz. Eger x = 3x1 , y = 2y, z = 5z1 olarak alırsak Pisagordenkleminden

x2 + y2 = z2

olur. Burada x, y, z degerleri aralarında asal ve y cift bir sayıdır. Pisagor uc-lulerini kullanırsak degerlerimiz

x = u2 − v2, y = 2uv, z = u2 + v2

olur. Bu esitliklerin ikincisinden u ve v ’nin 2 ’nin kuvvetleri oldukları acıktır.Buna gore v = 1 olmalıdır. Cunku birinci denklemden, 3’un kuvvetleri 2 ’ninkuvvetleri ile bolunemezler. Demek ki,3x1 = 2y − 1 ve 5z1 = 2y + 1 olacaktır.Ikinci denkleme (mod4) altında bakarsak y = 2y1 olur. Buradan

3x1 = (2y1 − 1) (2y1 + 1)

olacagından carpanlardan her ikisi de 3 ’un kuvveti olmalıdır. Buna gore y1 = 1ve x1 = z1 = 1 olmalıdır. Sonuc olarak, x = y = z = 2 olarak bulunur. Sıradakisorumuz 2005 Romanya Takım Secme Sınavı’ndan alınmıstır.

Ornek. (x, y) ∈ Z+ olmak uzere 3x = 2x ·y+1 esitligini saglayan degerleribulunuz.

Cozum. Denklemi yeniden yazarsak 3x − 1 = 2x · y olacaktır. Bunagore, (3x − 1) ifadesinin asal carpanlarından birisi ikinin kuvvetidir. Ve x de-geri bu kuvvetten kucuktur. Simdi bu kuvvetin ne kadar buyuk olabileceginiarastıralım. Ayrıca, eger n degeri tekse 3n − 1 ≡ 2 (mod4) ve eger n cift ise

Page 140: MATEMATİK OLİMPİYATI ÇALIŞMA KİTAPÇIĞI · C»˜oz um.˜ Bu soruda »c˜oz ume daha kolay i»slemlerle ula»smak i»cin baz‡ de‚gi»s-˜ ken e‚gi»stirmeler yapmak yerinde

2.15. USTEL DIYOFANT DENKLEMLERI 139

3n − 1 ≡ 0 (mod4) olacaktır. Buna gore x degiskenini 2m (2n+ 1) formundayazabiliriz. Buna gore,

3x − 1 = 32m(2n+1) − 1 =

(32n+1

)2m − 1

=(32n+1 − 1

) (32n+1 + 1

)m−1∏

k=1

((32n+1

)2k+ 1

)

olur. Eger bu esitlige (mod8) altında bakarsak ilk iki carpan 2 ve 4 olacaktır.Buna gore (3x − 1) icindeki 2’nin kuvvetleri (m + 2) tane olacaktır. Buradax ≤ m+ 2 ise

2m(2n+ 1) ≤ (m+ 2) ve 2m ≤ (m+ 2)

olacaktır. Burada m = 0, 1, 2 olabilir. Buna gore istenilen cozumler

(1, 1), (2, 2), (4, 5)

olacaktır.

2.15.1 Calısma Soruları

1. |3x − 2y| = 1 denklemini pozitif tamsayılarda cozunuz.

2. (x, y) pozitif tamsayılar olmak uzere verilen 3x − 2y = 7 denkleminicozunuz.

3. nx + ny + nz = nt esitligini saglayan tum (n, x, y, z, t) pozitif tamsayıbeslilerini bulunuz.

4. a. 3x − y3 = 1 denkleminin tum negatif olmayan tamsayı cozumlerinibulunuz.

b. p bir tek asal sayı olmak uzere verilen px − xp = 1 denkleminisaglayan tum negatif olmayan (x, y) tamsayı ikililerini bulunuz.

5. 1n+9n+10n = 5n+6n+11n esitligini saglayan tum n pozitif tamsayılarınıbulunuz.

Page 141: MATEMATİK OLİMPİYATI ÇALIŞMA KİTAPÇIĞI · C»˜oz um.˜ Bu soruda »c˜oz ume daha kolay i»slemlerle ula»smak i»cin baz‡ de‚gi»s-˜ ken e‚gi»stirmeler yapmak yerinde

140 BOLUM 2. KONULAR

2.15.2 Cozumler

1. 3x−2y = 1 icin x = 1, y = 1 ve x = 2, y = 3 birer cozumdur. Eger y ≥ 2ise x cift olmalıdır. x = 2z, z > 1 ve 3z = 2m+ 1, m > 1 alırsak

2y = (2m+ 1)2 − 1 = 4m(4m+ 1)

olacaktır ki, bu durum acık celiskidir.

3x−2y = −1 icin (mod8) altında bakarsak 3x+1 ifadesi 2 veya 4’e denkolacaktır. Buna gore y = 1 veya 2 olacaktır. Eger kontrol edilirse x = 1,y = 2 tek cozum olarak bulunacaktır.

2. Varsayalım y > 3 olsun. Buna gore

3x ≡ 7 (mod8)

olacaktır. Ancak 3x sayısı sadece 3 veya 1 sayılarına denk olabilir. Bunagore x = 1, 2 olabilir. x = 1 ise y /∈ Z ve x = 2, y = 1 oldugundan tekcozum ikilisi (2, 1) olarak bulunur.

3. Bu sorunun cozumunu iki farklı yoldan yapmayı deneyelim. Buna gore,

Cozum I. n = 1 durumun da denklemin saglanamayacagı acıktır. n > 1icin bakalım. Genelligi kaybetmeden x 6 y 6 z 6 t olarak alabiliriz.Esitligin iki tarafını nx ile bolersek

1 + ny−x + nz−x = nt−x

denklemini elde ederiz. Eger denkleme (modn) altında bakarsak y = xbuluruz. Buna gore a = z− x ve b = t− x alırsak 2+na = nb olur. Egera = 0 ise n = 3, b = 1 ve cozumlerimiz de

(n, x, y, z, t) = (3, x, x, x, x+ 1)

olur. Eger a > 0 ise (modn) altında yine baktıgımızda

(n, x, y, z, t) = (2, x, x, x+ 1, x+ 2) , x ∈ N olur.

Page 142: MATEMATİK OLİMPİYATI ÇALIŞMA KİTAPÇIĞI · C»˜oz um.˜ Bu soruda »c˜oz ume daha kolay i»slemlerle ula»smak i»cin baz‡ de‚gi»s-˜ ken e‚gi»stirmeler yapmak yerinde

2.15. USTEL DIYOFANT DENKLEMLERI 141

Cozum II. Denklemimizi

nx−t + ny−t + nz−t = 1

olarak yazalım. Esitligin sol kısmındaki tum terimler pozitif oldugunagore x, y, z < t olmalıdır. Boylece sol kısım en fazla 3/n olabilir. n = 2ve 3 icin bakarsak, n = 3 icin x = y = z = t− 1 olacaktır. n = 2 icin isesadece bir terim 1/n olabilir. Buna gore, x = y = t − 2 ve z = t − 1 vebu degerlerin permutasyonları bulunur.

4. a. Eger denklemde (0, 0) ikilisi alınırsa bunun bir cozum oldugunugormek zor olmayacaktır. Denklemi duzenlersek

3x = y3 + 1

olacagından esitligin sol tarafı

(y + 1)(y2 − y + 1

)

olacaktır. Burada iki carpanların ikisi de 3 ’un kuvveti olacaktır.Yani carpanlarımız

y + 1 = 3α ve y2 − y + 1 = 3β

olacaktır. Burada ilk denklemin karesini alıp ikinci denklemden cı-karırsak elde edecegimiz esitlik

3y = 3β(32α−β − 1

)

olacaktır. Soruda verilen denklemden y ’nin 3 ile bolunemediginibiliyoruz. Demek ki β = 1 olacaktır. Buna gore y2 − y + 1 = 3 ola-cagından y = 2 olarak bulunur. Buradan da x = 2 olarak bulunur.Demek ki ikililerimiz

(x, y) = (2, 2) , (0, 0)

olarak bulunacaktır.

Page 143: MATEMATİK OLİMPİYATI ÇALIŞMA KİTAPÇIĞI · C»˜oz um.˜ Bu soruda »c˜oz ume daha kolay i»slemlerle ula»smak i»cin baz‡ de‚gi»s-˜ ken e‚gi»stirmeler yapmak yerinde

142 BOLUM 2. KONULAR

b. Eger (x, y) ikilisi soruda verilen denklemin bir cozumu ise

px = yp + 1 = (y + 1)(yp−1 − yp−2 + · · ·+ y2 − y + 1

)

olacagından y + 1 = pn olacaktır. Eger n = 0 ise y = x = 0 ve pherhangi bir asal sayı olacaktır. Eger n > 1 ise

px = (pn − 1)p + 1

= pnp − ppn(p−1) +

(p

2

)pn(p−2) + · · ·+

(p

p− 2

)p2n + ppn

olacaktır. p bir asal sayı olduguna gore tum katsayılar p ile bolune-bilir. Dolayısıyla her bir terim pn+1 ile bolunebilecektir. Buna goreesitligin sag tarafını bolen, p’nin en buyuk kuvveti pn+1 olacaktır.Ve dolayısıyla x = n+ 1 olacaktır. Buradan

0 = pnp − ppn(p−1) +

(p

2

)pn(p−2) + · · ·+

(p

p− 2

)p2n

bulunur. p = 3 icin0 = 33n − 3 · 32n

ise n = 1 ve x = y = 2 bulunur. p ≥ 5 icin(

pp−2

)ifadesi p2 ile

bolunemez. Dolayısıyla esitligin sag kısmındaki son terim haricin-dekiler p2n+2 ile bolunebilir. Terimlerin toplamı sıfır oldugundan,bu durum imkansızdır. Dolayısıyla cozumler sadece x = y = 0, tump asalları icin ve x = y = 2, p = 3 olarak bulunur.

5. Eger her bir terimin birler basamagına bakarsak

1n, 5n, 6n, 10n ve 11n

ifadelerinin son basamakları sırasıyla 1, 5, 6, 0, 1 olacaktır. Esitligin sagın-daki terimlerin son basamakları toplamı 12 olduguna gore, 9n sayısınınbirler basamagı 1 olmalıdır. Dolayısıyla n ifadesi cift bir sayı olacaktır.Buradan n = 2 ve n = 4 birer cozum olacaktır. Eger n ≥ 6 ise

11n + 6n + 5n > 11n = (10 + 1)n

= 10n + n10n−1 + · · ·+ 1 ≥ 10n + 9n + 1n

Page 144: MATEMATİK OLİMPİYATI ÇALIŞMA KİTAPÇIĞI · C»˜oz um.˜ Bu soruda »c˜oz ume daha kolay i»slemlerle ula»smak i»cin baz‡ de‚gi»s-˜ ken e‚gi»stirmeler yapmak yerinde

2.16. KALAN SINIFLARI [RESIDUES] 143

olacaktır. Cunku

n · 10n−1 ≥ 6 · 104 · 10n−5 ≥ 95 · 10n−5 ≥ 9n

esitsizligi vardır. Buna gore cozumlerimiz n = 2 ve n = 4 olacaktır.

2.16 Kalan Sınıfları [Resıdues]

Olimpiyat sınavlarında sorulan soruları eger bir konu sınıflandırmasına tabitutmaya kalkıssak herhalde bu sınavların olmazsa olmaz soru gruplarındanbirisi de moduler aritmetik konusu olurdu. Ileriki satırlarda anlatılan orneklerive problemleri daha iyi kavrayabilmeniz icin bazı teoremleri bilmeniz gerekiyor.Bunun icin ilk olarak, bilmeniz gereken bu teoremlerden baslayacagız.

Teorem. [Cinli Kalanlar Teoremi] a1, a2, · · · , ar ikiserli olarak ara-larında aralarında asal sayılar pozitif tamsayılar olarak veriliyor. Buna gore(ai, aj) = 1 ve i 6= j icin

x ≡ k1 (moda1)

x ≡ k2 (moda2)

· · · · · · · · ·x ≡ r1 (modar)

· · · · · · · · ·

denklik sisteminin mod(a1 · a2 · · · · · ar)’ye gore tek cozumu vardır. Bu co-zumde

x =

(a

a1

)· b1 · k1 +

(a

a2

)· b2 · k2 + · · ·+

(a

ar

)· br · kr

olacaktır. Bu esitliktea = a1 · a2 · · · · · ar

ve (a

ai

)· bi ≡ (modai)

Page 145: MATEMATİK OLİMPİYATI ÇALIŞMA KİTAPÇIĞI · C»˜oz um.˜ Bu soruda »c˜oz ume daha kolay i»slemlerle ula»smak i»cin baz‡ de‚gi»s-˜ ken e‚gi»stirmeler yapmak yerinde

144 BOLUM 2. KONULAR

olacaktır.

Cinli kalanlar teoremini herhangi bir problem icerisinde nerede kullanaca-gınızı anlamanız oldukca kolaydır. Genelde sorulan tamsayının bir kac tamsayıdegeri ile bolumunden kalanlar verilir. Asagıdaki ornegimiz 2009 Tubitak Ma-tematik Olimpiyatlarında sorulmustu.

Ornek. 1 ≤ n ≤ 455 ve n3 ≡ 1 (mod455) kosullarını saglayan kac ntamsayı degeri vardır?

Cozum. 455 = 13 · 7 · 5 olduguna gore,

n3 ≡ 1 (mod5)

n3 ≡ 1 (mod7)

n3 ≡ 1 (mod13)

olduguna gore,

n ≡ 1 (mod5)

n ≡ 1, 2, 4 (mod7)

n ≡ 1, 3, 9 (mod13)

olduguna gore, 1 · 3 · 3 = 9 tane n tamsayı bulunur.

Teorem. [Kucuk Fermat Teoremi] p asal sayı ve p - a ise ap−1 ≡1 (modp) denkligi vardır.

Simdi bu teoremin ısıgında asagıda verilen ornegi ve cozumunu inceleyelim.

Ornek. a1 = 4, an = 4an−1 , n > 1 olduguna gore, a100 tamsayısının 7 ilebolumunden kalanı bulunuz.

Cozum. Kucuk Fermat Teoremine gore, 46 ≡ 1 (mod7) olacaktır. Bununyanında 4n ≡ 4 (mod6) ve 4n ≡ 4 + 6t oldugunu gormek zor degildir. Bunagore, soruda istenilen cevap

a100 ≡ 4a99 ≡ 44+6t ≡ 44 · (46)t ≡ 4 (mod7)

Page 146: MATEMATİK OLİMPİYATI ÇALIŞMA KİTAPÇIĞI · C»˜oz um.˜ Bu soruda »c˜oz ume daha kolay i»slemlerle ula»smak i»cin baz‡ de‚gi»s-˜ ken e‚gi»stirmeler yapmak yerinde

2.16. KALAN SINIFLARI [RESIDUES] 145

olacaktır.

Teorem. [Wilson Teoremi] p bir asal sayı olmak uzere, (p− 1)! ≡−1 (modp) denkligi vardır.

Teorem. [Euler-φ Teoremi] a ve n birer pozitif tamsayı olmak uzere,(a, n) = 1 ise aφ(n) ≡ 1 (modn) denkligi vardır.

Euler-φ Teoreminde adı gecen φ fonksiyonu herhangi bir n pozitif tamsayıdegeri icin, n sayısından kucuk esit ve n ile aralarında asal olan sayılarınsayısını vermektedir. Buna gore n = pa11 · pa22 · · · · · parr olarak asal carpanlarınaayrılırsa, φ(n) ifadesinin esiti

φ (n) = n ·(1− 1

p1

)·(1− 1

p2

)· · · · ·

(1− 1

pr

)

olacaktır.

Ornek. 771000

sayısının son iki basamagını bulunuz.

Cozum. Once φ(100) = φ(22) · φ(52) = (22 − 2)(52 − 5) = 40 degerinibululalım, cunku ileri basamaklarda kullanacagız. Buna gore, Euler Teore-minden 740 ≡ 1 (mod100) olacaktır. Benzer bicimde φ(40) = 16 olacagından716 ≡ 1 (mod40) olarak bulunur. 1000 = 16 · 62 + 8 olduguna gore,

71000 ≡ (716

)6278 ≡ 16278 ≡ (

74)2 ≡ 1 (mod40)

olacaktır. Buna gore,71000 = 1 + 40t

olacaktır. Simdi yaptıgımız bu islemleri ve sonucları kullanarak cozume git-meye calısırsak,

771000 ≡ 71+40t ≡ 7 · (740)t ≡ 7 (mod100)

cevabına ulasırız. Demek ki, sayının son iki basamagı 07 olacaktır.

Bu konu ile alakalı bilmeniz gereken teoremler ornekleriyle birlikte yukarıdaverildi. Eger teoremlerden ispatlarını merak ettikleriniz varsa, herhangi birsayılar teorisi kitabından kanıtlarını ogrenebilirsiniz. Simdi, calısma sorularınıcozmeye gecebilirsiniz.

Page 147: MATEMATİK OLİMPİYATI ÇALIŞMA KİTAPÇIĞI · C»˜oz um.˜ Bu soruda »c˜oz ume daha kolay i»slemlerle ula»smak i»cin baz‡ de‚gi»s-˜ ken e‚gi»stirmeler yapmak yerinde

146 BOLUM 2. KONULAR

2.16.1 Calısma Soruları

1. p bir asal sayı, w, n tamsayılar olmak uzere verilen 2p + 3p = wn denk-lemini saglayan tek n degerinin 1 oldugunu kanıtlayınız.

2. m, n pozitif tamsayılar olmak uzere√7 − m

n > 0 ise√7 − m

n > 1mn

oldugunu kanıtlayınız.

3. n ∈ Z olmak uzere, 2√28n2 + 1+ 2 bir tamsayı ise, bu tamsayının tam-

kare oldugunu kanıtlayınız.

4. {x2 + 6y2 = z2

6x2 + y2 = t2

denklem sisteminin (0, 0, 0, 0) dısında bir tamsayı dortlusu cozumu ol-madıgını kanıtlayınız.

5. Bir tamkare tamsayının basamakları toplamının alabilecegi olası tumdegerleri bulunuz.

6. x2 − y! = 2001 denklemini saglayan tum (x, y) pozitif tamsayı ikililerinibulunuz.

7. Rakamlarının yerleri degistirildiginde 2 nin kuvvetine esit olan kac tane2 nin kuvveti olan tamsayı vardır?

8. 44444444 sayısının basamakları toplamı A ve A sayısının basamaklarıtoplamı B ise B nin basamakları toplamı kactır?

9. y2 = x5 − 4 denkleminin tamsayılar kumesinde cozumunun olmadıgınıkanıtlayınız.

10. 1919 sayısının bir tamsayı kup ve bir tamsayı dorduncu kuvvetin topla-mına esit olamayacagını kanıtlayınız.

11. x2 + 3xy − 2y2 = 122 denklemini saglayan bir (x, y) tamsayı ikilisininolmadıgını kanıtlayınız.

12. n > 1 olmak uzere,

Page 148: MATEMATİK OLİMPİYATI ÇALIŞMA KİTAPÇIĞI · C»˜oz um.˜ Bu soruda »c˜oz ume daha kolay i»slemlerle ula»smak i»cin baz‡ de‚gi»s-˜ ken e‚gi»stirmeler yapmak yerinde

2.16. KALAN SINIFLARI [RESIDUES] 147

a. 1! + 2! + 3! + · · ·+ n! toplamının ancak ve ancak n = 3 oldugundabir tamsayının tam kuvvetine esit olabilecegini kanıtlayınız.

b. (1!)3 + (2!)3 + (3!)3 + · · ·+ (n!)3 toplamının ancak ve ancak n = 3oldugunda bir tamsayının tam kuvvetine esit olabilecegini gosteri-niz.

13

4444

sayısının ondalık yazılımda, sondan ucuncu yani yuzler basamagı kactır?

2.16.2 Cozumler

1. Eger p = 2 ise 22 + 32 = 13 ve n = 1 olacaktır. Eger p ≥ 2 ise p tek sayıolacagından (2 + 3)

(2p−1 − 2p−23 + · · ·+ 3p−1

)ifadesinden 5| (2p + 3p)

diyebiliriz. Buna gore 5|w olacaktır. Eger n > 1 ise 25|wn diyebiliriz.Dolayısıyla

2p + 3p

2 + 3= 2p−1 − 2p−2 · 3 + · · ·+ 3p−1

ifadesi de 5 ile bolunecektir. (−1)k · (2)p−k−1 · 3k carpımı (mod5) altında2p−1 olduguna gore ustte verilen toplam (mod5) altında p·2p−1 olacaktır.Ancak, p · 2p−1 ifadesi sadece p = 5 oldugunda 5 ile bolunebilir. Bunagore, 25 + 35 = 2751 olacagından n = 1 olacaktır.

2. Soruda verilen esitsizligi duzenlersek,√7− m

n > 0 ise√7 > m

n ve 7n2 −m2 > 0 olacaktır. Pozitif bir tam kare ifadenin 7 ile bolumunden kalanlarsadece 0, 1, 2 veya 4 olabilir. 7n2−m2 > 0 olduguna gore 7n2−m2 ≥ 7−4 = 3 olacaktır. Buradan

√7n ≥ √

m2 + 3 elde edilir. Bizim ispatlamayacalıstıgımız esitsizlikte artık m+ 1

m ≤ √7n olacaktır. Buradan,

m+1

m≤

√m2 + 3

olacaktır. Simdi bu esitsizligi kanıtlayalım. Aslında

m2 + 3 ≥ m2 + 2 +1

m2=

(m+

1

m

)2

oldugu acıktır. Kanıt tamamlanır.

Page 149: MATEMATİK OLİMPİYATI ÇALIŞMA KİTAPÇIĞI · C»˜oz um.˜ Bu soruda »c˜oz ume daha kolay i»slemlerle ula»smak i»cin baz‡ de‚gi»s-˜ ken e‚gi»stirmeler yapmak yerinde

148 BOLUM 2. KONULAR

3. Eger 2√28n2 + 1+2 ifadesi bir tamsayı ise 28n2+1 = (2m+ 1)2, m ≥ 0

olacaktır. Buna gore

28n2 + 1 = 4m2 + 4m+ 1

olduguna gore,7n2 = m (m+ 1)

olacaktır. m ve m + 1 aralarında asal olduklarına gore m = 7s2 vem + 1 = t2 veya m = u2ve m + 1 = 7v2 olacaktır. Burada ikinci du-rumun olması imkansızdır. Cunku (mod7)altında incelenirse kolaylıklagorulebilir. Demek ki m+ 1 = t2 olacagından,

2√

28n2 + 1 + 2 = 2 (2m+ 1) + 2 = 4m+ 4 = 4 (m+ 1) = 4t2 = (2t)2

olacaktır.

4. Varsayalım soruda verilen dortlu dısında da bir cozumumuz olsun. Ayrıcax, y, z, t sayılarının da ortak boleni olmasın. Buna gore soruda verilensistemdeki iki denklemi toplarsak

7(x2 + y2

)= z2 + t2

olacaktır. Tam kare bir sayının 7 ile bolumunden kalanlar yalnızca 0, 1, 2ve 4 olabilir. z2 + t2 ≡ 0 (mod7) ise z ve t sayılarının sırasıyla 7z0 ve 7t0formunda olacagı acıktır. Buna gore,

7(x2 + y2

)= (7z0)

2 + (7t0)2

olacagından x2+y2 = 7(z20 + t20

)olacagından bu seferde x ve y sayıları 7

ile kalansız bolunecektir. Buna gore, (x, y, z, t) = 7 olur. Ancak bu durumceliskilidir. Cunku biz cozumun en basında ortak bolenlerinin olmadıgınıkabul etmistik. Demek ki sistemin yegane cozumu olacaktır.

5. Bir tamsayının basamakları toplamı (mod9) altında sayının kendisinedenktir. Dolayısıyla eger kontrol edilirse tam kare bir sayı (mod9) al-tında 0, 1, 4 veya 7 olacaktır. Buna gore elimizdeki sayımızın basamak-ları toplamı 9m, 9m + 1, 9m + 4, 9m + 7 formlarından biri olacaktır.Simdi bu formları tek tek inceleyelim. Eger n = 9m ise (10m − 1)2 =

Page 150: MATEMATİK OLİMPİYATI ÇALIŞMA KİTAPÇIĞI · C»˜oz um.˜ Bu soruda »c˜oz ume daha kolay i»slemlerle ula»smak i»cin baz‡ de‚gi»s-˜ ken e‚gi»stirmeler yapmak yerinde

2.16. KALAN SINIFLARI [RESIDUES] 149

102m − 2 · 10m + 1 sayısının basamakları toplamı n olacaktır ve sayımız9 · · · 980 · · · 01 formunda olacaktır. Eger n = 9m + 1 ise (10m − 2)2 =102m − 4 · 10m + 4 ve 9 · · · 960 · · · 04 olacaktır. Eger m = 9m + 4 ise(10m − 3)2 = 102m − 6 · 10m + 9 ve 9 · · · 940 · · · 09 olacaktır. Eger n =9m−2 ise (10m − 5)2 = 102m−10 ·10m+25 ve 9 · · · 900 · · · 025 olacaktır.

6. y > 5 icin y! sayısı 9 ile tam bolunur. Dolayısıyla y! + 2001 toplamı(mod9) altında 3 sayısına denktir. Ancak tamkare bir sayının 9 ile bolu-munden kalanlar sadece 0, 1, 4, 7 olabilir. Demek ki, y = 1, 2, 3, 4 deger-lerinden birini alabilir. Eger kontrol edilirse (x, y) = (45, 4) ikilisi cozumolarak bulunur.

7. m < n icin 2m sayısı 2nsayısının rakamlarının permute edilmesiyle olus-sun. Bu iki sayının da aynı sayıda basamaklarının oldugu acıktır. Dola-yısıyla 2n < 10 · 2m olacaktır. Buna gore, n − m ≤ 3 olacaktır. Digertaraftan 2n ve 2m sayıları (mod9) altında denktir. Dolayısıyla

2n − 2m = 2m(2n−m − 1

)

sayısı 9 ile bolunur. Fakat 2m ve 9 aralarında asaldır ve n −m ≤ 3 icin2n−m − 1 ≤ 7 oldugundan, bu durum imkansızdır. Demek ki, bu sekildesayılar yoktur.

8. Once B sayısının basamakları toplamının kucuk bir sayı oldugunu gos-terelim. Buna gore,

44444444 < 100005000

esitsizliginden, 44444444 sayısı 20000 basamaktan azdır. Dolayısıyla, A <9 · 20000 = 180000 olacaktır. 180000 sayısından kucuk sayılar arasındabasamakları toplamı en buyuk olan sayı 99999 dur ve basamakları top-lamı 45 tir. Dolayısıyla B ≤ 45 olacaktır. Buna gore B sayısının basa-makları toplamı 39 sayısından 12 olur. Demek ki, bizim aradıgımız sayı12 den kucuk olacaktır. Diger taraftan, tum sayılar (mod9) altında ba-samakları toplamına denktir. Demek ki, bizim aradıgımız sayı 44444444

sayısına (mod9)altında denktir. Buna gore,

44444444 ≡ 74444 ≡ (78)555 · 74 ≡ 49 · 49 ≡ 13 · 13 ≡ 7 (mod9)

Page 151: MATEMATİK OLİMPİYATI ÇALIŞMA KİTAPÇIĞI · C»˜oz um.˜ Bu soruda »c˜oz ume daha kolay i»slemlerle ula»smak i»cin baz‡ de‚gi»s-˜ ken e‚gi»stirmeler yapmak yerinde

150 BOLUM 2. KONULAR

olacaktır. Demek ki B sayısının basamakları toplamı da (mod9) altında7 olacaktır. 12 sayısından kucuk ve (mod9) altında 7 sayısına denk olantek sayı 7 olduguna gore, istenilen cevap 7 olur.

9. Denklemimize (mod11) altında bakalım. Eger (x, 11) = 1 ise

(x5

)2= x10 ≡ 0 veya 1 (mod11)

olacaktır. Buna gore, x5 ≡ 0,−1 veya 1 (mod11) olacaktır. Demek ki,esitligin sag kısmı (mod11) altında 6, 7 veya 8 olacaktır. Ancak y2 ≡0, 1, 3, 4, 5, 9 (mod11) olabilir. Demek ki, bu esitligi saglayan (x, y) tam-sayı degerleri yoktur.

10. Varsayalım soruda verilen esitlik saglansın ve x, y tamsayılar olmak uzere,

x3 + y4 = 1919

olsun. Simdi toplamı olusturan bilesenleri ayrı ayrı inceleyelim.

x3 ≡ 0, 1, 5, 8, 12 (mod13) ve y4 ≡ 0, 1, 3, 9 (mod13)

olduguna gore 1919 ≡ 619 ≡ 612 · 67 ≡ 7 (mod13) olarak bulunur. Bunagore,

x3 + y4 ≡ 7 (mod13)

denkligini saglayan (x, y) ikilisi olmadıgından denklemin cozumu yoktur.

11. Denklemin her iki tarafını 4 ile carpıp duzenlersek,

(2x+ 3y)2 − 17y2 = 488

esitligini elde ederiz. Bu denklemin her iki tarafını (mod17) altında in-celersek

(2x+ 3y)2 ≡ 12 (mod17)

olacaktır. Ancak bir tam kare (mod17) altında sadece 0, 1, 4, 9, 16, 8, 2, 15, 13olabilir ancak 12 olamaz. Demek ki denklemi saglayan (x, y) tam sayılarıyoktur.

12. Soruda oldugu gibi cozumude sıklar halinde inceleyelim.

Page 152: MATEMATİK OLİMPİYATI ÇALIŞMA KİTAPÇIĞI · C»˜oz um.˜ Bu soruda »c˜oz ume daha kolay i»slemlerle ula»smak i»cin baz‡ de‚gi»s-˜ ken e‚gi»stirmeler yapmak yerinde

2.17. VIETE TEOREMI 151

a. n = 3 icin 1! + 2! + 3! = 32 oldugunu gormek zor degildir. Simdi budurumun tek oldugunu ve baska degeri bulunmadıgını kanıtlayalım.Varsayalım 1! + 2! + 3! + · · · + n! = km, m > 1 olsun. Buradan degerinin 9 dan buyuk yada esit olabilecegini kolaylıkla kontroledebiliriz. Eger n ≥ 5 ise 1!+2!+· · ·+n! sayısının son basamagı 3 ilebiter dolayısıyla m = 2 olamaz. Cunku tam kare sayılar 0, 1, 4, 5, 6veya 9 ile biter. Demek ki, m ≥ 3 olmalıdır. Bunun yanında n ≥ 9ise 1!+2!+ · · ·+n! toplamı 3 ile bolunebilir. Dolayısıyla k sayısı da3 ile bolunebilir. Buna gore, km sayısı da 27 ile bolunebilir. Demekki, faktoriyellerin toplamı da 27 ile bolunebilir. Fakat, 27| a!, a ≥ 9oldugundan, toplam 1! + 2! + · · · + 8! toplamı (mod27) altında 9sayısına denk olacaktır. Bu durum acık celiskidir. Demek ki n sadece3 olmalıdır.

b. Eger kontrol edilirse (1!)3+(2!)3+(3!)3 = 152 oldugunu gormek zordegildir. Ayrıca n = 2, 4, 5 ve 6 icin (1!)3+(2!)3+ · · ·+(n!)3toplamıbir tamsayının tam kuvvetine esit olamaz. n ≥ 7 icin bu toplam

(1!)3 + (2!)3 + (3!)3 + (4!)3 + (5!)3 + (6!)3 ≡ 7 (mod49)

olacaktır. Ancak bu kalan bir tam kuvvetin kalanı olmaz.

13. Bu sorunun cozumu okuyucuya bırakılmıstır. Euler-φ foksiyonu ve Ku-cuk Fermat Teoremlerini uygulayarak kolaylıkla cozume ulasabilirsiniz.

2.17 Vıete Teoremi

Viete teoremi (Viyet diye okunur) genel olarak bir polinomun kokleri ilekatsayıları arasındaki iliskiyi acıklar. Bu teoremi reel katsayılı polinomlardauygulayabildigimiz gibi benzer bicimde complex katsayılı polinomlarda da kul-lanabiliriz.

Buna gore, varsayalım x1 ve x2 degerleri P (x) = ax2+bx+c polinomununkokleri olsun. Buna gore,

ax2 + bx+ c = a (x− x1) (x− x2) = ax2 − a (x1 + x2)x+ ax1x2

Page 153: MATEMATİK OLİMPİYATI ÇALIŞMA KİTAPÇIĞI · C»˜oz um.˜ Bu soruda »c˜oz ume daha kolay i»slemlerle ula»smak i»cin baz‡ de‚gi»s-˜ ken e‚gi»stirmeler yapmak yerinde

152 BOLUM 2. KONULAR

olacagından, x1+x2 = − ba ve x1 ·x2 = c

a olacaktır. Benzer bicimde eger x1, x2ve x3 degerleri P (x) = ax3 + bx2 + cx + d polinomunun kokleri ise, koklerarasında ki iliski asagıdaki gibi olacaktır.

x1 + x2 + x3 = − b

a

x1x2 + x2x3 + x3x1 =c

a

x1x2x3 = −d

aEger yukarıda verdigimiz durumu genellestirirsek, kokleri x1, x2, · · · , xn olanbir p (x) = anx

n + an−1xn−1 + an−2x

n−2 + · · ·+ a2x2 + a1x+ a0 polinomunun

kokleri ve katsayıları arasındaki iliski asagıdaki gibi olacaktır.

x1 + x2 + · · ·+ xn = −an−1

an

x1x2 + x1x3 + · · ·+ xn−1xn =an−2

an...

x1x2x3 · · ·xn = (−1)na0an

Eger yukarıda yazdıgımız esitlikleri yeniden duzenlersek en genel form olan,

x1x2 · · ·xi + x1x2x4 · · ·xi+1 + · · ·+ xn−i+1xn−i+2 · · ·xn = (−1)ian−i

an

genel forma ulasılır.

Viete teoeremi her ne kadar bu ders notunun genel baslıgı olsada bazı denk-lem cozumlerinde oldukca isimize yarayacak olan Newton-Girard Formulu’nude verecegiz. Bu formulun kullanımı her nekadar elzem olmasa da, cozumlerindaha pratik olması acısından onemlidir.

2.17.1 Newton-Girard Formulæ

Newton-Girard Formulæ. Eger xn+a1xn−1+ · · ·+an = 0 denkleminin

kokleri α1, α2, · · · , αnve bu koklerin kuvvetleri toplamı

Sk = αk1 + αk

2 + αk3 + · · ·+ αk

n, k ∈ N+

Page 154: MATEMATİK OLİMPİYATI ÇALIŞMA KİTAPÇIĞI · C»˜oz um.˜ Bu soruda »c˜oz ume daha kolay i»slemlerle ula»smak i»cin baz‡ de‚gi»s-˜ ken e‚gi»stirmeler yapmak yerinde

2.17. VIETE TEOREMI 153

olarak verilirse, denklemin katsayıları ve k. kuvvetten koklerinin toplamlarıarasındaki iliski asagıdaki gibidir.

S1 + a1 = 0,

S2 + a1S1 + 2a2 = 0,

S3 + a1S2 + a2S1 + 3a3 = 0,

· · · · · · · · · · · · · · · · · · · · · · · · · · · = 0,

Sn + a1Sn−1 + · · ·+ an−1S1 + nan = 0

Buna gore, genel denklem

Sk + a1Sk−1 + · · ·+ anSk−n = 0, k > n

biciminde olacaktır. Simdi bu denklemi kullanarak problemleri cozmeye calı-salım.

Ornek. a, b, c sayıları x3 − x2 + 2 = 0 denkleminin kokleri ise

a2 + b2 + c2

a3 + b3 + c3

a4 + b4 + c4

toplamlarının esitlerini bulunuz.

Cozum. Her ne kadar bu sorunun cozumu ustte verdigimiz formullecozulebiliyor olsa da bu yontemi sizin kullanacagınızı dusunerek cozume dahada elementer bir yontemle gidelim. Buna gore,

a2 + b2 + c2 = (a+ b+ c)2 − 2 (ab+ ac+ bc) = 12 − 2 (0) = 1

olacaktır. Ikinci denklemin esiti ise, x3 = x2 − 2 olduguna gore,

a3 + b3 + c3 = a2 − 2 + b2 − 2 + c2 − 2 = a2 + b2 + c2 − 6 = 1− 6 = −5

olacaktır. Ucuncu toplamın esitini bulmak icinde x3 = x2 − 2 esitligin deesitligin her iki tarafını da x ile carparsak, x4 = x3 − 2x ise

a4+b4+c4 = a3−2a+b3−2b+c3−2c = a3+b3+c3−2 (a+ b+ c) = −5−2 (1) = −7

Page 155: MATEMATİK OLİMPİYATI ÇALIŞMA KİTAPÇIĞI · C»˜oz um.˜ Bu soruda »c˜oz ume daha kolay i»slemlerle ula»smak i»cin baz‡ de‚gi»s-˜ ken e‚gi»stirmeler yapmak yerinde

154 BOLUM 2. KONULAR

olarak sonuc bulunur.

Ornek. Asagıda verilen denklem sisteminin reel veya karmasık tum co-zum uclulerini bulunuz.

x+ y + z = 3,

x2 + y2 + z2 = 3,

x3 + y3 + z3 = 3.

Cozum. Varsayalım x, y, z degerleri bir P (t) polinomunun koku olsun.Buna gore,

p (t) = (t− x) (t− y) (t− z) = t3 − (x+ y + z) t2 + (xy + yz + zx) t− xyz

olacaktır. Burada

xy + yz + zx =(x+ y + z)2

2−

(x2 + y2 + z2

)

2=

9

2− 3

2= 3

esitligini elde ederiz. Buradan

x3 + y3 + z3 − 3xyz = (x+ y + z)(x2 + y2 + z2 − xy − yz − zx

)

esitligini kullanarak xyz = 1 olacaktır. Dolayısıyla

p (t) = t3 − 3t2 + 3t− 1 = (t− 1)3

olacaktır. Buna gore soruda verilen sistemin tek cozumu x = y = z = 1olacaktır.

Bu konuda ogrencilerin belkide bilmesi en onemli baslıklardan birisi deLagrange Interpolasyon Teknigi ’dir. Simdi bu teknigi ve bu teknik ile cozule-bilecek soruları inceleyelim.

Page 156: MATEMATİK OLİMPİYATI ÇALIŞMA KİTAPÇIĞI · C»˜oz um.˜ Bu soruda »c˜oz ume daha kolay i»slemlerle ula»smak i»cin baz‡ de‚gi»s-˜ ken e‚gi»stirmeler yapmak yerinde

2.17. VIETE TEOREMI 155

2.17.2 Lagrange Interpolasyon Teknıgı

Ornek. Kokleri 1, 2, 3 olan ve p(4) = 666 esitligini saglayan kubik poli-nomu bulunuz.

Cozum. Soruda istenen polinom ucuncu dereceden olduguna gore kesin-likle

p (x) = a (x− 1) (x− 2) (x− 3) , a ∈ Rformunda olmalıdır. Buna gore,

666 = p (4) = a (4− 1) (4− 2) (4− 3) = 6a, a = 111

bulunacagından polinom p (x) = 111 (x− 1) (x− 2) (x− 3) olacaktır.

Ornek. p (1) = 1, p (2) = 2, p (3) = 3, p (4) = 5 esitliklerini saglayan p (x)polinomunu bulunuz.

Cozum. Onceki ornekte cozume elementer yontemlerle ulasabilecegimizicin Lagrange Interpolasyon Teknigi’ni uygulamamıstık. Ancak bu soruda buteknigi hem uygulamasını ogrenecegiz hem de soruyu cozecegiz. Lagrange tek-nigine gore,

p (x) = a (x) + 2b (x) + 3c (x) + 5d (x) ,

polinomun da sırasıyla a (x) , b (x) , c (x) , d (x) birer kubik polinom olsun. Bunagore, a (1) = 1 ve a (x) polinomunun kokleri x = 2, 3, 4; b (2) = 1 ve b (x)polinomunun kokleri x = 1, 3, 4; c (3) = 1 ve c (x) polinomunun kokleri x =1, 2, 4; d (4) = 1 ve d (x) polinomunun kokleri x = 1, 2, 3 olacaktır.

Bir onceki ornekte uyguladıgımız teknigi yeniden burada uygularsak,

a (x) = −(x− 2) (x− 3) (x− 4)

6

b (x) =(x− 1) (x− 3) (x− 4)

2

c (x) = −(x− 1) (x− 2) (x− 4)

2

d (x) =(x− 1) (x− 2) (x− 3)

6

Page 157: MATEMATİK OLİMPİYATI ÇALIŞMA KİTAPÇIĞI · C»˜oz um.˜ Bu soruda »c˜oz ume daha kolay i»slemlerle ula»smak i»cin baz‡ de‚gi»s-˜ ken e‚gi»stirmeler yapmak yerinde

156 BOLUM 2. KONULAR

polinomlarını elde ederiz. Dolayısıyla, soruda istenilen polinom

p (x) = −1

6(x− 2) (x− 3) (x− 4)+(x− 1) (x− 3) (x− 4)−3

2(x− 1) (x− 2) (x− 4)+

5

6(x− 1) (x− 2) (x− 3)

olacaktır. isterseniz soruda verilen sartları saglayıp saglamadıgını kontrol ede-bilirsiniz.

Sırada ki ornegimize, Bilgi Universitesi Cahit Arf Matematik Gunleri VII-2008 sınavında sorulan bir polinom sorusu ile devam edelim.

Ornek. n > 1 bir tamsayı ve p (x) ise derecesi (n− 1) olan bir polinomolsun. Eger her k = 1, · · · , n icin p (k) = 1/k ise p (n+ 1) ifadesinin esitinibulunuz.

Cozum. Varsayalım elimizde p (x)− 1x seklinde bir polinom olsun. Ancak

1x ifadesi burada ki varsayımımızı bozacagından g (x) = x·p (x)−1 polinomunusecelim. p (x) polinomunun derecesi (n− 1) ise g (x) polinomunun derecesin. dereceden olacaktır. Ayrıca polinomunun kokleride olacaktır. Buna goreaslında g (x) polinomu

g (x) = c · (x− 1) · (x− 2) · · · · · (x− n)

olarak yazılabilir. Buradaki c katsayı sabitini bulabilmek icin once g (0) = −1oldugunu buluruz. Buna gore, x = 0

g (0) = c · (0− 1) · (0− 2) · · · · · (0− n)

= c · −1 · −2 · −3 · · · · · −n

= c · (−1)n · n!esitligini kullanırsak

c =−1

(−1)nn!=

(−1)1−n

n!

olacaktır. Bizden istenen p(n+ 1) ise

g (n+ 1) = (n+ 1) p (n+ 1)− 1

ve

g (n+ 1) =(−1)1−(n+1)

(n+ 1)!· (n+ 1− 1) · (n+ 1− 2) · · · · · (n+ 1− n)

Page 158: MATEMATİK OLİMPİYATI ÇALIŞMA KİTAPÇIĞI · C»˜oz um.˜ Bu soruda »c˜oz ume daha kolay i»slemlerle ula»smak i»cin baz‡ de‚gi»s-˜ ken e‚gi»stirmeler yapmak yerinde

2.17. VIETE TEOREMI 157

olacagından

(n+ 1) · p (n+ 1)− 1 =(−1)−n

(n+ 1)!· n!

olacagından

p (n+ 1) =

[(−1)n

n+ 1+ 1

]· 1

n+ 1=

(−1)n + (n+ 1)

(n+ 1)2

olacaktır.

Konu olarak Viete Teoremi ile cozulebilecek sorular, nispeten daha kolaysorulardır. Bu sebepten konu anlatımı icerisindeki sorularıda calısma sorula-rına ekledik. Soruların tamamı AMC ve AIME sınavlarından derlenmistir.

2.17.3 Calısma Soruları

1. ax2 + bx+ c = 0 polinomunun koklerinin tersleri toplamını bulunuz.

2. x1 ve x2 degerleri x2 − 3√2x− 3

√4 = 0 polinomunun kokleri ise

x1x2

+x2x1

toplamını bulunuz.

3. b ve c birer sabit sayı olmak uzere verilen

(x+ 2) (x+ b) = x2 + cx+ 6

esitligini saglayan c degerini bulunuz.

4. x1 6= x2 olmak uzere 3x21 − hx1 = b ve 3x22 − hx2 = b esitlikleri veriliyor.Buna gore x1 + x2 toplamını bulunuz.

5. P (x) = x3+ax2+bx+c polinomunun koklerinin aritmetik ortası, kokle-rin carpımına ve katsayılar toplamına esittir. P (x) polinomunun grafigiy eksenini 2 noktasında kestigine gore, b degerini bulunuz.

6. x2+mx+n = 0 denkleminin koklerinin kupleri x2+px+q = 0 oldugunagore, asagıda verilen esitliklerin ifadelerin dogrulugunu kontrol ediniz.

Page 159: MATEMATİK OLİMPİYATI ÇALIŞMA KİTAPÇIĞI · C»˜oz um.˜ Bu soruda »c˜oz ume daha kolay i»slemlerle ula»smak i»cin baz‡ de‚gi»s-˜ ken e‚gi»stirmeler yapmak yerinde

158 BOLUM 2. KONULAR

• p = m3 + 3mn

• p = m3 − 3mn

• p = 3mn−m3

• p+ q = m3

• (mn

)3= p

q

7. ax2+bx+c = 0 polinomunun kokleri r ve s olduguna gore, kokleri ar+bve as+ b olan ikinci dereceden denklemi yazınız.

8.

x2 − px+(p2 − 1

)/4 = 0

denkleminin kokler farkının mutlak degerini bulunuz.

9. x2 + kx + 6 = 0 polinomunun kokleri x1 ve x2 olarak verildiginde x2 −kx + 6 = 0 polinomunun kokleri sırasıyla x1 + 5 ve x2 + 5 olmaktadır.Buna gore, k degerini bulunuz.

10. x2 +mx+ n = 0denkleminin kokleri x2 + px+m = 0 denkleminin kok-lerinin iki katı olduguna gore, n/p degerini bulunuz.(n ve p degerlerininher biri sıfırdan farklıdır.)

11. a ve b birer reel sayı olmak uzere,

8x3 + 4ax2 + 2bx+ a = 0

denkleminin birbirinden farklı uc pozitif koku vardır. Koklerin log2 ta-banındaki toplamları 5 olduguna gore, a degerini bulunuz.

12. x2 − 63x + k = 0 denkleminin iki koku de birer asal sayıdır. Buna gorek degerinin alabilecegi kac farklı deger vardır.

13.4√x =

12

7− 4√x

denklemini saglayan degerler toplamını bulunuz.

Page 160: MATEMATİK OLİMPİYATI ÇALIŞMA KİTAPÇIĞI · C»˜oz um.˜ Bu soruda »c˜oz ume daha kolay i»slemlerle ula»smak i»cin baz‡ de‚gi»s-˜ ken e‚gi»stirmeler yapmak yerinde

2.17. VIETE TEOREMI 159

14.

x2 + 18x+ 30 = 2√x2 + 18x+ 45

denklemini saglayan degerler carpımını bulunuz.

15. p, q ve r degerleri x3 − x2 + x − 2 = 0 denklemini birer koku oldugunagore,

p3 + r3 + q3

toplamının esitini bulunuz.

16. x1000 − 10x + 10 = 0 denkleminin kokleri r1, r2, r3, · · · , r1000 oldugunagore,

r10001 + r10002 + r10003 + · · ·+ r10001000

degerini bulunuz.

17.

a+ 2b+ 4c = 12

ab+ 4bc+ 2ac = 22

abc = 6

denklem sistemini saglayan (a, b, c) uclulerini bulunuz.

2.17.4 Cozumler

1. ax2 + bx + c = 0 denkleminin koklerine ve diyelim. Buna gore sorudaistenilen toplam

1

x1+

1

x2

ifadesinin esiti olacagına gore,

1

x1+

1

x2=

x1 + x2x1 · x2 =

−b/a

c/a= −b

c

olarak istenilen toplam bulunur.

Page 161: MATEMATİK OLİMPİYATI ÇALIŞMA KİTAPÇIĞI · C»˜oz um.˜ Bu soruda »c˜oz ume daha kolay i»slemlerle ula»smak i»cin baz‡ de‚gi»s-˜ ken e‚gi»stirmeler yapmak yerinde

160 BOLUM 2. KONULAR

2. Onceki sorudaki benzer yontemleri kullanarak cozume gidelim. x1 ve x2denklemim birer koku olduguna gore,

x1x2

+x2x1

=x21 + x22x1 · x2 =

(x1 + x2)2 − 2x1 · x2

x1 · x2 = −3

olarak bulunacaktır.

3. Denklemin kokleri −2 ve −b olduguna gore kokler carpımı (−2 · −b) = 6ve kokler toplamı −b − 2 = c olacagından, soruda istenilen degerlerkolaylıkla bulunur.

4. Soruda verilen iki denklem birbirine esit olduguna gore,

3x21 − 3x22 = hx1 − hx2

olacaktır Burada x1 6= x2 olduguna gore, soruda istenilen ifade x1+x2 =h/3 olarak bulunacaktır.

5. Varsayalım denklem kokleri x1, x2, x3 olsun. Buna gore,

x1 + x2 + x33

= x1 · x2 · x3 = a+ b+ c+ 1

olacaktır. Soruda ayrıca P (0) = 2 olarak verildigine gore, c = 2 olacaktır.Buna gore yeni esitligimizi saglayan degerler a = 12 ve b = −17 olarakbulunur.

6. Varsayalım denklemlerimizin koklerini x31, x32 olarak alalım. Buna gore,

x31+x32 = (x1 + x2)(x21 + x22 − x1x2

)= −m

((x1 + x2)

2 − 3x1x2

)= −m

(m2 − 3mn

)

olacaktır. Buna gore,

x31 + x32 = −p = −m3 + 3mn

olacagından esitliklerden

m3 − 3mn = p

olanı dogrudur.

Page 162: MATEMATİK OLİMPİYATI ÇALIŞMA KİTAPÇIĞI · C»˜oz um.˜ Bu soruda »c˜oz ume daha kolay i»slemlerle ula»smak i»cin baz‡ de‚gi»s-˜ ken e‚gi»stirmeler yapmak yerinde

2.17. VIETE TEOREMI 161

7. Bu sorunun cozumu okuyucuya bırakılmıstır.

8. Denklemin kokler toplamı ve kokler carpımını kullanmaya calısalım. Bunagore,

x1 + x2 = p, ve x1 · x2 = p2 − 1

4

olacaktır. Diskriminantı kontrol edersek de

∆ = p2 − 4(p2 − 1

) 14= p2 − p2 + 1 = 1

ise denklemin kokleri,

x1 =p+ 1

2ve x2 =

p− 1

2

olacagına gore, soruda istenilen cevap,

|x1 − x2| = 2

olacaktır.

9. Bu sorunun cozumu okuyucuya bırakılmıstır.

10. Bu sorunun cozumu okuyucuya bırakılmıstır.

11. Bu sorunun cozumu okuyucuya bırakılmıstır.

12. Bu sorunun cozumu okuyucuya bırakılmıstır.

13. Soruda verilen denklemde 4√x = a olarak alınırsa, yeni denklem

a2 − 7a+ 12 = 0

olacaktır. Buradan a1 = 4 ve a2 = 3 olacagından istenen x degerlerix = 256 ve x = 81 olarak bulunur.

14. Bu sorunun cozumu okuyucuya bırakılmıstır.

Page 163: MATEMATİK OLİMPİYATI ÇALIŞMA KİTAPÇIĞI · C»˜oz um.˜ Bu soruda »c˜oz ume daha kolay i»slemlerle ula»smak i»cin baz‡ de‚gi»s-˜ ken e‚gi»stirmeler yapmak yerinde

162 BOLUM 2. KONULAR

15. p, q, r denklemin kokleri olduguna gore,

p3 + r3 + q3 = p2 + q2 + r2 − p− q − r + 6

= (p+ q + r)2 − 2 (pq + pr + qr)− (p+ q + r) + 6

= 1− 2 · 1 + 1 + 6 = 6

olarak cevap bulunur.

16. Her bir ri degeri denklemi sagladıgına gore

r1001 − 10r1 + 10 = 0

olacaktır. Eger her bir r1, r2, · · · , r100 degerini yerine koyalım buna gore,

r1001 = 10r1 − 10

r1001 = 10r2 − 10

· · · = · · ·r100100 = 10r100 − 10

esitliklerini elde ederiz. Bu esitlikleri alt alta toplarsak

= 10 (r1 + r2 + · · ·+ r100)− 10 · 1000= 10 · 10− 10 · 1000= 10 · (−990)

= −9900

cevabı elde edilir.

17. Eger a, 2b ve 4c degerlerini bir denklemin kokleri olarak alalım. Bunagore denklemimiz

0 = (x− a) (x− 2b) (x− 4c)

=(x2 − 2bx− ax+ 2ab

)(x− 4c)

=(x2 − x (2b+ a) + 2ab

)(x− 4c)

= x3 − x2 (a+ 2b+ 4c) + x (2ab+ 4ac+ 8bc)− 8abc

= x3 − 12x2 + 22x− 48

Page 164: MATEMATİK OLİMPİYATI ÇALIŞMA KİTAPÇIĞI · C»˜oz um.˜ Bu soruda »c˜oz ume daha kolay i»slemlerle ula»smak i»cin baz‡ de‚gi»s-˜ ken e‚gi»stirmeler yapmak yerinde

2.18. BAGINTI SAYILARI 163

olduguna gore gercektende a, 2b ve 4c degerleri 3. dereceden bir denkle-min kokleridir. Buna gore toplam da 3 · 3 · 2 = 6 tane uclu bulunabilir.

2.18 Bagıntı Sayıları

A kumesi bos kumeden farklı bir kume olmak uzere A× A kartezyen car-pım kumesinin tum alt kumelerine birer bagıntı denir. Varsayalım kumemizA = {a, b, c} olsun. Once kartezyen carpımı ve bagıntıyı tablo ile gosterelim.Tablo cizmekteki amacımız hem kartezyen carpımı hemde secebilecegimiz her-hangi bir bagıntıyı tablo uzerinde daha kolay acıklayabilmemiz olacaktır.

a b c

a ∗b

c ∗ ∗Yukarıdaki tabloda verilen bagıntının ikilileri, eger bu bagıntıya β dersek,

β = {(a, b), (c, a), (c, c)}olacaktır. Yıldızları tablodaki karelere rastgele koyarak, yada hicbir kareyekoymayarak, bagıntılar elde edebiliriz. Bu noktadan sonra karsılasabilecegimizsoruladan biriside, bu sekilde kac bagıntının yazılabilecegi olacaktır. Yukarıdacizdigimiz tablo uzerinden dusunurek baslayalım. Eger tablodaki her bir huc-reye, birer yıldız koyarsak toplamda 3× 3 = 9 yıldız olacaktır. Bagıntılarımızkartezyen carpımlarımızın birer alt kumesi olduguna gore, 29 tane bagıntımızolacaktır.

Bu durumu genellestirelim. Eger s(A) = n olarak alırsak, bagıntı sayımız

2n×n = 2n2

olacaktır. Simdi, bagıntı cesitlerini ve sayılarını inceleyelim.

Yansıyan Bagıntı. x ∈ A olmak uzere, ∀x ∈ A icin (x, x) ikilisi bagıntı-nın bir elemanı ise bagıntımız yansıyan olacaktır. Eger A = {a, b, c} ise,

Page 165: MATEMATİK OLİMPİYATI ÇALIŞMA KİTAPÇIĞI · C»˜oz um.˜ Bu soruda »c˜oz ume daha kolay i»slemlerle ula»smak i»cin baz‡ de‚gi»s-˜ ken e‚gi»stirmeler yapmak yerinde

164 BOLUM 2. KONULAR

a b c

a ∗b ∗c ∗

yukarıdaki tabloda da goruldugu uzere, en kucuk yansıyan bagıntımız

β = {(a, a), (b, b), (c, c)}olacaktır. Oyleyse, bu bagıntıya ekleyecegimiz her ikili icin bagıntı hala yansı-yan bagıntı olarak kalacaktır. Isaretli olmayan 6 kutu icin, kartezyen carpımın26 tane alt kumesi olacagından 64 tane yansıyan bagıntı yazılabilir. Eger Akumesinin n tane elemanı oldugunu varsayarsak, A × A kumesinin n2 taneelemanı olacaktır. Olusturulacak tablonun kosegeni uzerindeki n elemanı cı-karırsak kalan n2 − n tane elemanın alt kumelerini almamız yeterlidir. Yaniyansıyan bagıntı sayımız

2n2−n = 2n·(n−1)

olacaktır.

Ingilizce kaynaklarda Irreflexive Relation olarak gecen, bagıntıya biz Turkceolarak Yansımaz Bagıntı diyelim. Buna gore, eger yazılan bagıntının icindex ∈ A icin (x, x) /∈ β durumu varsa, bagıntımız bir yansımaz bagıntı olacaktır.Daha basit bir anlatımla cizilecek tablonun kosegeni uzerinde bagıntılar yazı-lırken hic yıldız bulunmayacak. Demek ki, yansıyan bagıntı sayısı ile yansımazbagıntı sayısı aynıdır. Ayrıca Yansımaz Bagıntı ile Yansıyan Olmayan Bagıntıbirbirlerinden farklıdır.

Simetrik Bagıntı. Bir bagıntının simetrik olması icin, (x, y) elemanı ba-gıntının icindeyse, (y, x) bagıntısınında bagıntının icinde olması sartı aranır.Eger tablo uzerinden dusunursek, tablonun ksegene gore simetrik olması gere-kir.

a b c

a ∗b ∗ ∗c ∗ ∗

Page 166: MATEMATİK OLİMPİYATI ÇALIŞMA KİTAPÇIĞI · C»˜oz um.˜ Bu soruda »c˜oz ume daha kolay i»slemlerle ula»smak i»cin baz‡ de‚gi»s-˜ ken e‚gi»stirmeler yapmak yerinde

2.18. BAGINTI SAYILARI 165

Ornegin yukarıda verilen tabloda verilen bagıntı simetriktir. Bagıntının ele-manları

{(a, b), (b, a), (b, c), (c, b), (a, a)}olacaktır. Simdi durumu genellestirelim. Eger s(A) = n olarak alırsak, simetrikbagıntı yazacagımız icin kosegen ustundeki ve uzerindeki noktaları secip bunoktaların olusturdugu kumenin alt kumelerini almamız yeterlidir. Durumugenellestirelim. Tum ikililerin sayısı n2, kosegen uzerindeki noktaların sayısı nise,

n2 − n

2+ n =

n · (n+ 1)

2istedigimiz ikililerin sayısını alt kumelerin sayısı, yani Simetrik Bagıntı Sayısı

2n·(n+1)

2

olacaktır.

Anti-Simetrik Bagıntı. x ve y birbirinden farklı elemanlar olmak uzere(x, y) ve (y, x) aynı anda bagıntının elemanı degilse bagıntı antisimetrik birbagıntı olur. Ornegin asagıda tablosu verilen bagıntı bir antisimetrik bagıntı-dır.

a b c

a ∗ ∗b

c ∗ ∗Bagıntının elemanları,

(a, a), (a, b), (c, a), (c, c)

olacaktır. Bagıntıda (a, b) varken (b, a) ve (c, a) varken (a, c) yoktur. Ama ko-segenler uzerindeki elemanlar istenildigi gibi secilebilir. Peki, kac tane simetrikbagıntı vardır? Bu sayıyı bulmak icin kosegen ustunde olma ve olmama du-rumlarını ayrı ayrı degerlendirelim. Eger s(A) = n olarak alırsak, kosegenuzerindeki, elemanlar icin 2n tane durum vardır. Kosegen uzerinde olmayanelemanlar icin ise uc durum vardır. Bunlar,

Page 167: MATEMATİK OLİMPİYATI ÇALIŞMA KİTAPÇIĞI · C»˜oz um.˜ Bu soruda »c˜oz ume daha kolay i»slemlerle ula»smak i»cin baz‡ de‚gi»s-˜ ken e‚gi»stirmeler yapmak yerinde

166 BOLUM 2. KONULAR

(x,y) (y,x)

icinde dısındadısında icindedısında icinde

seklinde olacaktır. (x, y) ve (y, x) ikilisinin ikiside zaten bagıntının icinde ola-maz. Kumemizde n tane eleman oldugunu varsayarsak,

C

(n

2

)=

n · (n− 1)

2

farklı durum vardır. Her ikili icin uc farklı durum olduguna gore,

3n·(n−1)/2

farklı durum olusacaktır. Buna gore toplam antisimetrik bagıntı sayısı

2n · 3n·(n−1)/2

olacaktır.

Ayrıca antisimetrik bagıntı ile simetrik olmayan bagıntılar birbirinden farklıdurumlardır. Ornegin,

{(a, b), (a, c), (c, a), (c, c)}bagıntısı simetrik degildir. Cunku, (a, b) bagıntının elemanı iken (b, a) bagıntı-nın elemanı degildir. Benzer bicimde bagıntı antisimetrik degildir cunku hem(a, c) hemde (c, a) bagıntının birer elemanıdır.

Anti-Simetrik Bagıntı ve Yansıyan Bagıntı. Eger bu durumu antisi-metrik bagıntı ile karsılastırırsak, aradaki tek fark, bu durumda her iki kosegenuzerindeki tum karelerde birer yıldız olacaktır. Ancak hala,

n · (n− 1)

2

eleman icin 3 farklı durum vardır. Bu sebepten de, yansıyan ve antisimetrikbagıntı sayısı

3n·(n−1)/2

olacaktır.

Page 168: MATEMATİK OLİMPİYATI ÇALIŞMA KİTAPÇIĞI · C»˜oz um.˜ Bu soruda »c˜oz ume daha kolay i»slemlerle ula»smak i»cin baz‡ de‚gi»s-˜ ken e‚gi»stirmeler yapmak yerinde

2.18. BAGINTI SAYILARI 167

2.18.1 Calısma Soruları

Asagıdaki calısma soruları Tubitak XVI. Bilgisayar Olimpiyatları sına-vında sorulmustur. Soruların cozumlerini once kendiniz yapmaya calısınız.Daha sonra cozumleri okuyunuz.

A = {0, 1, {1}, {1, {1}}} kume olarak tanımlanıyor.

1. A kumesi uzerinde kac farklı ikili bagıntı tanımlanabilir?

2. A kumesi uzerinde kac farklı simetrik olan ikili bagıntı tanımlanabilir?

3. A kumesi uzerinde kac farklı simetrik ve yansıma ozeligine sahip ikilibagıntı tanımlanabilir?

2.18.2 Cozumler

1. A kumesinin sorudada goruldugu uzere 4 tane elemanı vardır. Buna goreA kumesi uzerinde tanımlanabilecek bagıntı sayısı icin once kartezyencarpımın elemanlarını bulalım. s(A × A) = 4 · 4 = 16 olacaktır. Herbir bagıntı kartezyen kumenin alt kumesi olduguna gore, alt kumeleribulmamız yeterli olacaktır. Buna gore istenen cevap

24×4 = 216

olacaktır.

2. Simetrik bagıntı sayısı 2n·(n+1)

2 olduguna gore, n = 4 icin istenen cevap210 olacaktır.

3. Bagıntımız hem yansıyan hem simetrik olacagına gore, bagıntının icindemutlaka

{(a, a), (b, b), (c, c), (d, d)}ikilileri mutlaka bulunacaktır. Buna gore bu bagıntının icine eleman ola-rak ekleyecegimiz her ikili bir yansıyan bagıntı olacaktır. Bagıntımız egeraynı anda simetrik bagıntı ozeliginide tasıyacagına gore, bagıntının icinde

Page 169: MATEMATİK OLİMPİYATI ÇALIŞMA KİTAPÇIĞI · C»˜oz um.˜ Bu soruda »c˜oz ume daha kolay i»slemlerle ula»smak i»cin baz‡ de‚gi»s-˜ ken e‚gi»stirmeler yapmak yerinde

168 BOLUM 2. KONULAR

(a, b) eleman olarak bulunuyorsa, (b, a) ikilisi de eleman olarak bagın-tının icinde bulunacaktır. Geriye kalan 12 eleman icindeki her bir ikilisimetrikligi saglamak icin yanında bir ikiliyi daha tasıyacagından aslındabizim 6 elemanlı bir kumeden alt kumeler secmemi gerekir. Buna gore,istenen sayı 26 = 64 olacaktır.

2.19 Lıneer Denklemlerın Tamsayı Cozumlerı

Genel olarak tekrarlı permutasyon konusu altında verilen bu konuda asılamacımız

x1 + x2 + · · ·+ xr = n, n ∈ Z+ ∨ n ∈ Z+ ∪ {0}formunda verilen bir lineer denklemin pozitif tamsayı veya negatif olmayantamsayılardaki cozum sayılarını bulmak olacaktır. Once teoremleri ve cozumluornekleri dikkatli bir sekilde calısarak konu sonunda verilen calısma sorular ilekonuyu daha iyi kavramaya calısınız. Once bir teoremle baslayalım.

Teorem (De Moivre) . n pozitif bir tamsayı olmak uzere verilen

x1 + x2 + x3 + · · ·+ xr = n

denkleminin pozitif tamsayı cozumlerinin sayısı

(n− 1

r − 1

)

olacaktır.

Kanıt . n sayısını n = 1 + 1 + · · · + 1 + 1 olarak yazalım. Bu toplamdan tane 1 ve (n − 1) tane + isaretinin oldugu acıktır. Buna gore, n toplamınır tane parcaya bolmek icin bizim (r − 1) tane + isaretini secmemiz yeterliolacaktır. Buna gore secimimiz

(n− 1

r − 1

)

olacagından ispat tamamlanır. ¥

Page 170: MATEMATİK OLİMPİYATI ÇALIŞMA KİTAPÇIĞI · C»˜oz um.˜ Bu soruda »c˜oz ume daha kolay i»slemlerle ula»smak i»cin baz‡ de‚gi»s-˜ ken e‚gi»stirmeler yapmak yerinde

2.19. LINEER DENKLEMLERIN TAMSAYI COZUMLERI 169

Ornek. 9 rakamını uc pozitif tamsayının toplamı olarak kac farklı bicimdeyazabiliriz? Mesela, 1 + 1 + 7 ve 7 + 1 + 1 birbirinden farklı iki toplamdır.

Cozum. Soruyu eger denklem biciminde yazarsak, aslında sorulan soru

a+ b+ c = 9 a > 0, b > 0, c > 0

denkleminin cozum sayısı olacaktır. Oyleyse istenen cevap

(9− 1

3− 1

)=

(8

2

)= 28

olacaktır.

Sonuc . n pozitif bir tamsayı olmak uzere verilen

y1 + y2 + · · ·+ yr = n

denkleminin negatif olmayan tamsayılardaki cozum sayısı

(n+ r − 1

r − 1

)

olacaktır.

Kanıt . Denklemde yr = xr−1, xr ≥ 1 degisken degistirmesini yaparsak

x1 − 1 + x2 − 1 + · · ·+ xr − 1 = n

olacagından

x1 + x2 + · · ·+ xr = n+ r

olacaktır. Kanıtın bundan sonrası De Moivre teoreminin bir uygulamasınadonustugune gore (

n+ r − 1

r − 1

)

cozum sayısı olacaktır. ¥

Page 171: MATEMATİK OLİMPİYATI ÇALIŞMA KİTAPÇIĞI · C»˜oz um.˜ Bu soruda »c˜oz ume daha kolay i»slemlerle ula»smak i»cin baz‡ de‚gi»s-˜ ken e‚gi»stirmeler yapmak yerinde

170 BOLUM 2. KONULAR

Ornek.

a+ b+ c+ d = 100, a ≥ 30, b ≥ 21, c ≥ 1, d ≥ 1

durumlarını saglayan kac farklı (a, b, c, d) tamsayı dortlusu secilebilir?

Cozum. Once uygun degisken degistermeleri yapalım. a = a′ + 29, b =b′ + 20 olarak alırsak yeni denklemimiz

a′ + b′ + c+ d = 50

olacaktır. Bu denkleminde pozitif tamsayı cozumleride

(49

3

)= 18424

olacaktır.

Ornek. 8 katlı bir binanın asansorune binen 5 kisi asansorden katlarakac farklı bicimde dagılabilirler?

Cozum. Aslında soruda bulunması istenen sayı

x1 + x2 + · · ·+ x8 = 5

denkleminin negatif olmayan cozum sayısıdır. Buna gore cevap

(8 + 5− 1

8− 1

)=

(12

7

)= 792

olacaktır.

Ornek. a + b + c + d ≤ 2009 esitsizligini saglayan kac farklı negatifolmayan tamsayı (a, b, c, d) dortlusu vardır?

Cozum. a+b+c+d ≤ 2009 denkleminin negatif olmayan tamsayı cozumdortlulerinin sayısı

a+ b+ c+ d+ f = 2009, f ≥ 0

Page 172: MATEMATİK OLİMPİYATI ÇALIŞMA KİTAPÇIĞI · C»˜oz um.˜ Bu soruda »c˜oz ume daha kolay i»slemlerle ula»smak i»cin baz‡ de‚gi»s-˜ ken e‚gi»stirmeler yapmak yerinde

2.19. LINEER DENKLEMLERIN TAMSAYI COZUMLERI 171

denkleminin cozum dortlulerinin sayısına esittir. Benzer bicimde son yazdıgı-mız denklemin cozumlerinin sayısıda

a1 − 1 + b1 − 1 + c1 − 1 + d1 − 1 + f1 − 1 = 2009

denkleminin cozum sayısı ile aynı olacagına gore istenen cevap(2013

4

)

olacaktır.

Buraya kadar yaptıgımız orneklerde, degiskenlere dair yaptıgımız sınırla-malar hep tek yonluydu. Sıradaki ornegimizde durum biraz daha farklı.

Ornek.

a+ b+ c+ d = 100, 1 ≤ a ≤ 10, b ≥ 0, c ≥ 2, 20 ≤ d ≤ 30

olamak uzere verilen denklemin tum tamsayı cozum dortlulerinin sayısını bu-lunuz.

Cozum. Cozumu durum, durum inceleyerek surdurelim. Eger, a ≥ 1, b ≥0, c ≥ 2, d ≥ 20 olarak alırsak denklemimizin

(80

3

)= 82160

farklı cozumu olacaktır. Eger a ≥ 11, b ≥ 0, c ≥ 2, d ≥ 20 olarak alırsak cozumsayımız (

70

3

)

kadar olacaktır. Eger a ≥ 1, b ≥ 0, c ≥ 2, d ≥ 31 olarak alırsak cozum sayımız(69

3

)

kadar olacaktır. Bu iki durumun kesisimi ise(59

3

)

Page 173: MATEMATİK OLİMPİYATI ÇALIŞMA KİTAPÇIĞI · C»˜oz um.˜ Bu soruda »c˜oz ume daha kolay i»slemlerle ula»smak i»cin baz‡ de‚gi»s-˜ ken e‚gi»stirmeler yapmak yerinde

172 BOLUM 2. KONULAR

kadar olacagından birlesim kumesinin eleman sayısı(70

3

)+

(69

3

)−

(59

3

)= 74625

olacaktır. Buna gore istenen cozum sayısı(80

3

)−

(70

3

)−

(69

3

)+

(59

3

)= 7335

olacaktır.

Genel olarak karsınıza cıkabilecek lineer denklemler ve pozitif tamsayı co-zumleri orneklerle acıklandı. Simdi bir cogu Tubitak Bilgisayar Olimpiyatlarıbirinci asama sınavında cıkmıs calısma sorularını yapmaya calısarak konuyudaha iyi kavramaya calısınız.

2.19.1 Calısma Soruları

1. 1 ile 1000 arasındaki sayılardan kac tanesinin rakamlarının toplamı 7yapar?

2. (5a+ 8b+ 2c)15 acıldıgında kac terim elde edilir?

3. 0 ≤ x1, x2, x3, x4 ≤ 7 oldugunda x1 + x2 + x3 + x4 = 18 denkleminin kacfarklı tamsayı cozumu vardır?

4. 20 adet bos kartın her birinin uzerine kare, daire ve ucgen resmi cizi-lebilmektedir. Buna gore kac farklı sekilde 20 karttan olusan bir demetolusturulabilir?

5. 100 tane birbirinin aynısı top ve 5 adet birbirinden farklı kutu bulun-maktadır. Her bir kutuda en az 6 adet top bulunacak sekilde topları kacfarklı sekilde kutulara dagıtabiliri?

6. 100 tane birbirinin aynısı top ve 5 adet birbirinden farklı kutu bulun-maktadır. Her bir kutuda en fazla 40 adet top bulunacak bicimde kacfarklı sekilde kutulara dagıtabiliriz?

7. Kırmızı, beyaz ve mavi zarların ucu birden atıldıgında, kac farlı durumdagelen sayıların toplamı 10 yapar?

Page 174: MATEMATİK OLİMPİYATI ÇALIŞMA KİTAPÇIĞI · C»˜oz um.˜ Bu soruda »c˜oz ume daha kolay i»slemlerle ula»smak i»cin baz‡ de‚gi»s-˜ ken e‚gi»stirmeler yapmak yerinde

2.20. FONKSIYONEL DENKLEMLER 173

8. a + b + c + d = 98 esitligini saglayan kac (a, b, c, d) pozitif tek tamsayıdortlusu vardır? (AIME 1998)

2.19.2 Cozumler

Konu anlatımı icerisinde verilen cozumler, ornekleri cozmeniz icin yeterli-dir. Biraz ugrasark cozumlere ulasabilirsiniz.

2.20 Fonksıyonel Denklemler

Bilinmeyenlerinin birer fonksiyon oldugu denklemlere genel olarak fonk-siyonel denklem denilir. Benzer bicimde bilinmeyenlerin polinomlardan olus-tugu denklem sorularınıda bu baslık altında alabiliriz. Ancak bu tur sorularınne yazıkki genel bir cozum yontemi yoktur. Belkide bu sebepten matematikyarısmalarında sıklıkla sorulan sorular haline gelmislerdir. Bu baslık altındagenel olarak tek degiskenli fonksiyonel denklemlerin genel cozum teknikleri vecok degiskenli fonksiyonel denklemler konuları ele alınmıstır.

2.20.1 Tek Degiskenliler - Temel Teknikler

Tek degiskenlilerin cozumlerine uygun donusumler yapılarak ulasılır. Or-nekleri inceleyerek devap edelim.

Ornek. f (x+ 1) = x2 − 3x+ 2 fonksiyonel denklemini cozunuz.

Cozum. t = x+1 ise x = t−1 olacagından f(t) = (t−1)2−3(t−1)+2 =t2 − 5t+ 6 esitliginden f(x) = x2 − 5x+ 6 olarak bulunur.

Ornek.

f(x+ 1

x) =

x2 + 1

x2+

1

xfonksiyonel denklemini cozunuz.

Cozum. t = (x+ 1)/x olarak alırsak x = 1/(t− 1) olacagından

f(t) =( 1t−1)

2 + 1

( 1t−1)

2+

1

( 1t−1)

= t2 − t+ 1

Page 175: MATEMATİK OLİMPİYATI ÇALIŞMA KİTAPÇIĞI · C»˜oz um.˜ Bu soruda »c˜oz ume daha kolay i»slemlerle ula»smak i»cin baz‡ de‚gi»s-˜ ken e‚gi»stirmeler yapmak yerinde

174 BOLUM 2. KONULAR

bulunur. Buna gore f(x) = x2 − x+ 1 olarak bulunur.

Genel olarak cozumlerde uyguladıgımız teknik, f [ϕ(x)] = g(x) esitligini ficin cozmektir. Eger ϕ fonksiyonunun tersi varsa, t = ϕ(x) olarak alabiliriz.Dolayısıyla da

f(x) = g[ϕ−1(x)]

olur.

Ornek. f(ex) = x3 + sinx fonksiyonel denklemini cozunuz.

Cozum. t = ex degisken degistirmesini yaparsak, x = ln t olacagından

f(x) = (ln |x|)3 + sin(ln |x|)

olacaktır.

Ornek. a 6= ±1 olmak uzere verilen

f(x

x− 1) = af(x) + ϕ(x)

fonksiyonel denklemini cozunuz.

Cozum. t = x/(x − 1) degisken degistirmesini yaparsak, x = t/(t − 1)olacaktır. Buna gore,

f(t) = af(t

t− 1) + ϕ(

t

t− 1) = a(af(t) + ϕ(t)) + ϕ(

t

t− 1)

olacagından, istenilen fonksiyon

f(x) =aϕ(x) + ϕ( x

x−1)

1− a2

olacaktır.

Bir fonksiyonel denklem degiskenlere baglı cebirsel ifadeler ihtiva edebilir.Bu tur sorularda uygulanacak temel teknik eszamanlı fonksiyonlar olustur-maktır. Simdi asagıdaki ornekleri inceleyelim.

Page 176: MATEMATİK OLİMPİYATI ÇALIŞMA KİTAPÇIĞI · C»˜oz um.˜ Bu soruda »c˜oz ume daha kolay i»slemlerle ula»smak i»cin baz‡ de‚gi»s-˜ ken e‚gi»stirmeler yapmak yerinde

2.20. FONKSIYONEL DENKLEMLER 175

Ornek.

3f(x) + 2f(1/x) = 4x

fonksiyonel denklemini cozunuz.

Cozum. x degiskeni yerine 1/x alınırsa,

3f(1/x) + 2f(x) = 4/x

olacaktır. Soruda verilen fonksiyon −3/2 ile carpılıp son bulunan denklem iletoplanırsa, istenilen denklem

f(x) =12x2 − 8

5x

olarak bulunacaktır.

Ornek. x 6= 0 olmak uzere verilen

1

x· f(−x) + f(

1

x) = x

fonksiyonel denklemini cozunuz.

Cozum. Soruda verilen denklemde x yerine −x yazarsak,

−1

xf(x) + f(−1

x) = −x

bulunur. Eger x yerine 1x alınırsa

xf(−1

x) + f(x) =

1

x

denklemi elde edilecektir. Bu iki fonksiyonel denklemden, istenilen fonksiyon

f(x) =1

2(x2 +

1

x)

olarak elde edilir.

Page 177: MATEMATİK OLİMPİYATI ÇALIŞMA KİTAPÇIĞI · C»˜oz um.˜ Bu soruda »c˜oz ume daha kolay i»slemlerle ula»smak i»cin baz‡ de‚gi»s-˜ ken e‚gi»stirmeler yapmak yerinde

176 BOLUM 2. KONULAR

Fonksiyonel denklemlerin cozumu yapılırken eger fonksiyonun ozelikleridegoz onune alınırsa cozume ulasılması daha kolay olacaktır. Oyle ki fonksi-yonun surekliligi, monotonlugu, sınırlılıgı, turevlenebilir olması gibi bilgilerinbilinmesi cozumlerde faydalı olacaktır.

Ornek.

f(x+ 1) + f(x− 1) = 2x2 − 4x

fonksiyonel denklemini cozunuz.

Cozum. Iki fonksiyonun toplamı ikinci dereceden olduguna gore f(x)fonksiyonuda ikinci dereceden olacaktır. Buna gore, f(x) = ax2 + bx+ c ise

2ax2 + 2bx+ 2(a+ c) = 2x2 − 4x

esitliginden f(x) fonksiyonu f(x) = x2 − 2x− 1 olarak bulunur.

2.20.2 Cok Degiskenliler

Bazı cok degiskenli fonksiyonel denklemlerin cozumlerinde simetriden fay-dalanılarak denklemi tek degiskenliye indirgemek cozumu oldukca kolaylastırır.Asagıdaki ornekler ve cozumler bu durumla alakalıdır.

Ornek.

(x− y)f(x+ y)− (x+ y)f(x− y) = 4xy(x2 − y2)

fonksiyonel denklemini cozunuz.

Cozum. Eger soruda verilen esitligi duzenlersek

f(x+ y)

x+ y− f(x− y)

x− y= 4xy

olacaktır. Buna gore

g(x) =f(x)

x

Page 178: MATEMATİK OLİMPİYATI ÇALIŞMA KİTAPÇIĞI · C»˜oz um.˜ Bu soruda »c˜oz ume daha kolay i»slemlerle ula»smak i»cin baz‡ de‚gi»s-˜ ken e‚gi»stirmeler yapmak yerinde

2.20. FONKSIYONEL DENKLEMLER 177

fonksiyonunu alalım. Eger

g(x+ y)− g(x− y) = 4xy

ise

g(x+ y)− (x+ y)2 = g(x− y)− (x− y)2

olacagından g(x) − x2 = k, k ∈ R ve g(x) = x2 + k, f(x) = x3 + kx olarakbulunur.

Degisken sayısını dusurmenin bir diger yoluda uygun degerleri degiskenlereatamaktır. Asagıdaki ornek bu metodla cozulmustur.

Ornek. f(x) · f(y) − f(xy) = x + y fonksiyonel denklemini f(x) icincozunuz.

Cozum. Varsayalım y = 0 olsun. Buna gore, f(0)[f(x)−1] = x olacaktır.f(0) 6= 0 olacagına gore,

f(x) =x

f(0)+ 1

esitligi elde edilir. Benzer bicimde x = y = 0 alınırsa f(0)[f(0) − 1] = 0 isef(0) = 1 olacagından f(x) = x+ 1 olarak bulunur. Gercektende buldugumuzfonksiyon soruda verilen denklemi saglamaktadır.

Ornek. f fonksiyonu

f(x) + f(y) = f(x+ y)− xy − 1

esitligini saglamaktadır. Buna gore, eger f(1) = 1 ise f(n) = n esitliginisaglayan negatif tamsayı degerlerini bulunuz.

Cozum. x = 1 olarak alırsak, f(y + 1) − f(y) = y + 2 ve y = 0 alırsakf(0) = −1 olacaktır. n ≥ icin

f(n+1)+1 = f(n)−f(0) =n−1∑

y=0

(f(y+1)−f(y)) =n−1∑

y=0

(y+2) =(n+ 1)(n+ 2)

2−1

Page 179: MATEMATİK OLİMPİYATI ÇALIŞMA KİTAPÇIĞI · C»˜oz um.˜ Bu soruda »c˜oz ume daha kolay i»slemlerle ula»smak i»cin baz‡ de‚gi»s-˜ ken e‚gi»stirmeler yapmak yerinde

178 BOLUM 2. KONULAR

olacagından

f(n) =n2+3n+2

2− 2 =

n2 + 3n− 2

2

olacaktır. Eger x = n ve y = −n alırsak, f(n) + f(−n) = n2 − n ve

f(−n) = n2 − 2− f(n) =(−n)2 + 3(−n)− 2

2

olacaktır. Demekki,

f(n) =n2 + 3n− 2

2

esitligi negatif n degerleri icinde gecerlidir. Sorudaki f(n) = n durumu icin

n2 + 3n− 2

2= n

ise (n− 1)(n+ 2) = 0 olacagından n = 1 veya n = −2 olacaktır.

Fonksiyonel denklemlerin cozumleri yapılırken bazı kullanıslı sonucların bi-linmesinde fayda vardır. Ornegin asagıda verilenler bunlardan sadece birkacı-dır. Buna gore, f surekli bir fonksiyon olmak uzere

i. f(x+ y) = f(x) + f(y) ise f(x) = c,

ii. f(x+ y) = f(x) · f(y) ise f(x) = cx

iii. f(xy) = f(x) + f(y) ise f(x) = c lnx

iv. f(xy) = f(x) · f(y) ise f(x) = xc

olacaktır. Sıradaki ornegimizin cozumunde bu sonucları kullanabiliriz.

Ornek. (Jensen Fonksiyonel Denklemi) x, y ∈ R olmak uzere verilen

f

(x+ y

2

)=

f (x) + f (y)

2

esitligini saglayan tum surekli f fonksiyonlarını bulunuz.

Page 180: MATEMATİK OLİMPİYATI ÇALIŞMA KİTAPÇIĞI · C»˜oz um.˜ Bu soruda »c˜oz ume daha kolay i»slemlerle ula»smak i»cin baz‡ de‚gi»s-˜ ken e‚gi»stirmeler yapmak yerinde

2.20. FONKSIYONEL DENKLEMLER 179

Cozum. Eger y = 0 olarak alınırsa,

f(x2

)=

f (x) + f (0)

2

olacaktır. Buna gore,

f (x) + f (y)

2= f

(x+ y

2

)=

f (x+ y) + f (0)

2

olacaktır. Yani, f(x + y) = f(x) + f(y) − f(0) olarak bulunur. Varsayalımh(x) = f(x) − f(0) ise h(x + y) = h(x) + h(y) olacagından h(x) = cx vef(x) = cx+ f(0) olarak bulunur.

Ornek. f(x+y) = g(x)+h(y) esitligini saglayan tum surekli fonksiyonlarıbulunuz.

Cozum. Eger y = 0 olarak alırsak h(0) = b ise f(x) = g(x) + b ve egerx = 0 olarak alırsak benzer bicimde f(y) = a+ h(y) olacaktır. Buna gore,

f(x+ y) = [f(x)− b] + [f(y)− a] = f(x) + f(y)− (a+ b)

olacagından F (x) = f(x) − a − b ve F (x + y) = F (x) + F (y) olacaktır. Ffonksiyonu toplamsal surekli fonksiyon oldugundan F (x) = kx olacaktır. Sonucolarak f(x) = kx+ a+ b, g(x) = kx+ a ve h(x) = kx+ b olacaktır.

Olimpiyat sınavlarında ozellikle TST ve kamp sınavlarında sorulan soru-ların cozumleri oldukca zordur. Belli teknikler kullanarak cozumeye gitmeyecalısmak her ne kadar gerekli olsada cogu zaman yereli degildir. Ancak bazıyardımcı ozelikleri arastırmak sanırız cozum yolundaki en buyuk yardımcılar-dan olacaktır. Bu sebepten elimizdeki fonksiyonun

i. Birebir veya ortenligi,

ii. Periyodikligi veya tek, cift fonksiyon olma durumu,

iii. Artan yada azalan olma durumu,

iv. Simetrikligi

gibi ozelliklerinin olup olmadıgını bilmek, cogu zaman cozumu ciddi bicimdekolaylastıracaktır.

Page 181: MATEMATİK OLİMPİYATI ÇALIŞMA KİTAPÇIĞI · C»˜oz um.˜ Bu soruda »c˜oz ume daha kolay i»slemlerle ula»smak i»cin baz‡ de‚gi»s-˜ ken e‚gi»stirmeler yapmak yerinde

180 BOLUM 2. KONULAR

2.20.3 Calısma Soruları

1. f : R→ R olmak uzere verilen

f(f(x+ y)) = f(x+ y) + f(x)f(y)− xy x, y ∈ Resitligini saglayan tum f fonksiyonlarını bulunuz.

2. f : Z→ R ve x+ y toplamı 3’un tam katı olduguna gore,

f

(x+ y

3

)=

f(x) + f(y)

2

esitligini saglayan f− fonksiyonlarını bulunuz.

3. f : R→ R olmak uzere verilen

f(xf(y) + x) = xy + f(x)

esitligini saglayan tum f fonksiyonlarını bulunuz.

4. f : R→ R olmak uzere verilen

f(f(x) + y) = f(x2 − y) + 4f(x)y

esitligini saglayan f fonksiyonlarını bulunuz.

2.20.4 Cozumler

1. Eger y = 0 alırsak,f(f(x)) = [1 + f(0)]f(x)

esitligi elde edilir. Eger x yerine x+ y alırsak

[1 + f(0)]f(x+ y) = f(f(x+ y)) = f(x+ y) + f(x)f(y)− xy

esitligindenf(0)f(x+ y) = f(x)f(y)− xy

elde edilir. Eger y = 1 alınırsa,

f(0)f(x+ 1) = f(x)f(1)− x

Page 182: MATEMATİK OLİMPİYATI ÇALIŞMA KİTAPÇIĞI · C»˜oz um.˜ Bu soruda »c˜oz ume daha kolay i»slemlerle ula»smak i»cin baz‡ de‚gi»s-˜ ken e‚gi»stirmeler yapmak yerinde

2.20. FONKSIYONEL DENKLEMLER 181

esitligi elde edilir. Eger y = −1 ve x yerine x+ 1 alınırsa

f(0)f(x) = f(x+ 1)f(−1) = f(x+ 1)f(−1) + x+ 1

esitligi elde edilir. Eger son iki denklemden f(x+ 1)’i cekersek

[f2(0)− f(1)f(−1)]f(x) = [f(0)− f(−1)]x+ f(0)

esitligi elde edilir. Eger f2(0)− f(1)f(−1) 6= 0 ise f− fonksiyonu lineer-dir. Eger f2(0)−f(1)f(−1) = 0 ise son denklemde x = 0 alarak f(0) = 0olarak bulunur. Bu durumda fonksiyon f(x)f(y) = xy olacaktır. y = 1alırsak f(x)f(1) = x olacaktır. Dolayısıyla f(1) 6= 0 ve f(x) lineerdir.Sonuc olarak, f(x) = ax + b olarak soruda verilen denklemde yerinekoyarsak a = 1 ve b = 0 bulunur. Demek ki, fonksiyon f(x) = x’dir.

2. Butun n tamsayıları icin

f(0) + f(3n) = 2f(n) = f(n) + f(2n)

olacagından

f(n) = f(2n) =f(3n) + f(3n)

2= f(3n)

olacaktır. Demek ki f(n) = f(0) esitligi vardır. Dolayısıyla tum tumsabit fonksiyonlar birer cozum olacaktır.

3. Eger x = 1, y = −1− f(1) ve a = f(y) + 1 alırsak esitligimiz

f(a) = f(f(y) + 1) = y + f(1) = −1

olacaktır. Eger y = a ve b = f(a) olarak alırsak

b = f(xf(a) + x) = ax+ f(x)

vef(x) = −ax+ b

olacaktır. Eger sorudaki denklemde yerine koyarsak,

ax2y − abx− ax+ b = xy − ax+ b

esitligi bulunur. Polinom esitliginden a = ±1 ve b = 0 dolayısıyla f(x) =x veya f(x) = −x bulunur.

Page 183: MATEMATİK OLİMPİYATI ÇALIŞMA KİTAPÇIĞI · C»˜oz um.˜ Bu soruda »c˜oz ume daha kolay i»slemlerle ula»smak i»cin baz‡ de‚gi»s-˜ ken e‚gi»stirmeler yapmak yerinde

182 BOLUM 2. KONULAR

4. y = x2 olarak alırsak

f(f(x) + x2) = f(0) + 4x2f(x)

bulunur. y = −f(x) alınırsa

f(0) = f(f(x) + x2) + 4f(x)2

esitsizligi elde edilir. Buldugumuz bu iki esitligi karsılastırırsak, her birx degeri icin f(x) = 0 veya f(x) = x2 olmalıdır. Varsayalım f(a) = 0olsun. x = a olarak alırsak

f(y) = f(a2 − y)

bulunur. y 6= a2/2 iciny2 6= (a2 − y)2

ise f(y) = 0 olacaktır. Son olarak x = 2a ve y = a2/2 alırsak

f(a2/2) = f(7a2/2) = 0

olacaktır. Demek ki, f(x) = 0 veya f(x) = x2 olmalıdır.